NUMBER SYSTEMS - Numbers - How to Prepare for Quantitative Aptitude for CAT

How to Prepare for Quantitative Aptitude for CAT (2014)

Block I: Numbers

...BACK TO SCHOOL

  • Chapters in this Block: Number Systems and Progressions
  • Block Importance – 20–25%

The importance of this block can be gauged from the table below:

Year

% of Marks from Block 1

Qualifying Score (approx. score for 96 percentile)

2000

48%

35%

2001

36%

35%

2002

36%

35%

2003 (cancelled)

30%

32%

2003 (retest)

34%

35%

2004

32%

35%

2005

40%

35%

2006

32%

40%

2007

24%

32%

2008

40%

35%

Online CAT 2009–2013

15–30%

60% with no errors

As you can see from the table above, doing well in this block alone could give you a definite edge and take you a long way to qualifying the QA section. Although the online CAT has significantly varied the relative importance to this block, the importance of this block remains high. Besides, there is a good chance that once the IIMs get their act together in the context of the online CAT and its question databases—the pre eminence of this block of chapters might return.

Hence, understanding the concepts involved in these chapters properly and strengthening your problem solving experience could go a long way towards a good score.

Before we move into the individual chapters of this block, let us first organise our thinking by looking at the core concepts that we had learnt in school with respect to these chapters.

Pre-assessment Test

This test consists of 25 questions based on the chapters of BLOCK I (Number Systems and Progressions). Do your best in trying to solve each question.

The time limit to be followed for this test is 30 minutes. However, after the 30 minutes is over continue solving till you have spent enough time and paid sufficient attention to each question. After you finish thinking about each and every question of the test, check your scores. Then go through the SCORE INTERPRETATION ALGORITHM given at the end of the test to understand the way in which you need to approach the chapters inside this block.

1.

The number of integers n satisfying – n + 2 ≥ 0 and 2n ≥ 4 is

(a) 0

(b) 1

(c) 2

(d) 3

2.

The sum of two integers is 10 and the sum of their reciprocals is 5/12. Then the larger of these integers is

(a) 2

(b) 4

(c) 6

(d) 8

3.

If x is a positive integer such that 2x + 12 is perfectly divisible by x, then the number of possible values of x is

(a) 2

(b) 5

(c) 6

(d) 12

4.

Let K be a positive integer such that k + 4 is divisible by 7. Then the smallest positive integer n, greater than 2, such that k + 2n is divisible by 7 equals.

(a) 9

(b) 7

(c) 5

(d) 3

5.

273 – 272 – 271 is the same as

(a) 269

(b) 270

(c) 271

(d) 272

6.

Three times the first of three consecutive odd integers is 3 more than twice the third. What is the third integer?

(a) 15

(b) 9

(c) 11

(d) 5

7.

x, y and z are three positive integers such that x > y > z. Which of the following is closest to the product xyz?

(a) xy(z – 1)

(b) (x – 1)yz

(c) (x y)xy

(d) x(y + 1)z

8.

A positive integer is said to be a prime number if it is not divisible by any positive integer other than itself and 1. Let p be a prime number greater than 5, then (p2 – 1) is

(a)

never divisible by 6.

(b)

always divisible by 6, and may or may not be divisible by 12.

(c)

always divisible by 12, and may or may not be divisible by 24.

(d)

always divisible by 24.

9.

Iqbal dealt some cards to Mushtaq and himself from a full pack of playing cards and laid the rest aside. Iqbal then said to Mushtaq “If you give me a certain number of your cards, I will have four times as many cards as you will have. If I give you the same number of cards, I will have thrice as many cards as you will have”. Of the given choices, which could represent the number of cards with Iqbal?

(a) 9

(b) 31

(c) 12

(d) 35

10.

In Sivakasi, each boy’s quota of match sticks to fill into boxes is not more than 200 per session. If he reduces the number of sticks per box by 25, he can fill 3 more boxes with the total number of sticks assigned to him. Which of the following is the possible number of sticks assigned to each boy?

(a) 200

(b) 150

(c) 125

(d) 175

11.

A lord got an order from a garment manufacturer for 480 Denim Shirts. He bought 12 sewing machines and appointed some expert tailors to do the job. However, many didn’t report for duty. As a result, each of those who did, had to stitch 32 more shirts than originally planned by Alord, with equal distribution of work. How many tailors had been appointed earlier and how many had not reported for work?

(a) 12, 4

(b) 10, 3

(c) 10, 4

(d) None of these

12.

How many 3-digit even numbers can you form such that if one of the digits is 5, the following digit must be 7?

(a) 5

(b) 405

(c) 365

(d) 495

13.

To decide whether a number of n digits is divisible by 7, we can define a process by which its magnitude is reduced as follows: (i1, i2, i3…..in are the digits of the number, starting from the most significant digit).

i1i2in fi i1. 3n–1 + i2, 3n–2 + …. + In30.

e.g. 259 fi 2.32 + 5.31 + 9.30 = 18 + 15 + 9 = 42

Ultimately the resulting number will be seven after repeating the above process a certain number of times.

After how many such stages, does the number 203 reduce to 7?

(a) 2

(b) 3

(c) 4

(d) 1

14.

A teacher teaching students of third standard gave a simple multiplication exercise to the kids. But one kid reversed the digits of both the numbers and carried out the multiplication and found that the product was exactly the same as the one expected by the teacher. Only one of the following pairs of numbers will fit in the description of the exercise. Which one is that?

(a) 14, 22

(b) 13, 62

(c) 19, 33

(d) 42, 28

15.

If 8 + 12 = 2, 7 + 14 = 3 then 10 + 18 =?

(a) 10

(b) 4

(c) 6

(d) 18

16.

Find the minimum integral value of n such that the division 55n/124 leaves no remainder.

(a) 124

(b) 123

(c) 31

(d) 62

17.

What is the value of k for which the following system of equations has no solution:

2x – 8y = 3; and kx + 4y = 10.

(a) –2

(b) 1

(c) –1

(d) 2

18.

A positive integer is said to be a prime if it is not divisible by any positive integer other than itself and one. Let p be a prime number strictly greater than 3. Then, when p2 + 17 is divided by 12, the remainder is

(a) 6

(b) 1

(c) 0

(d) 8

19.

A man sells chocolates that come in boxes. Either full boxes or half a box of chocolates can be bought from him. A customer comes and buys half the number of boxes the seller has plus half a box. A second customer comes and buys half the remaining number of boxes plus half a box. After this, the seller is left with no chocolates box. How many chocolates boxes did the seller have before the first customer came?

(a) 2

(b) 3

(c) 4

(d) 3.5

20.

X and Y are playing a game. There are eleven 50 paise coins on the table and each player must pick up at least one coin but not more than five. The person picking up the last coin loses. X starts. How many should he pick up at the start to ensure a win no matter what strategy Y employs?

(a) 4

(b) 3

(c) 2

(d) 5

21.

If a < b, which of the following is always true?

(a)

a < (a + b) / 2 < b

(b)

a < ab/2 < b

(c)

a < b2a2 < b

(d)

a < ab < b

22.

The money order commission is calculated as follows. From ` X to be sent by money order, subtract 0.01 and divide by 10. Get the quotient and add 1 to it, if the result is Y, the money order commission is ` 0.5Y. If a person sends two money orders to Aurangabad and Bhatinda for ` 71 and ` 48 respectively, the total commission will be

(a) ` 7.00

(b) ` 6.50

(c) ` 6.00

(d) ` 7.50

23.

The auto fare in Ahmedabad has the following formula based upon the meter reading. The meter reading is rounded up to the next higher multiple of 4. For instance, if the meter reading is 37 paise, it is rounded up to 40 paise. The resultant is multiplied by 12. The final result is rounded off to nearest multiple of 25 paise. If 53 paise is the meter reading what will be the actual fare?

(a) ` 6.75

(b) ` 6.50

(c) ` 6.25

(d) ` 7.50

24.

Juhi and Bhagyashree were playing simple mathematical puzzles. Juhi wrote a two digit number and asked Bhayashree to guess it. Juhi also indicated that the number is exactly thrice the product of its digits. What was the number that Juhi wrote?

(a) 36

(b) 24

(c) 12

(d) 48

25.

It is desired to extract the maximum power of 3 from 24!, where n! = n.(n – 1) . (n – 2) … 3.2.1. What will be the exponent of 3?

(a) 8

(b) 9

(c) 11

(d) 10

ANSWER KEY

1. (b)

2. (c)

3. (c)

4. (a)

5. (c)

6. (a)

7. (b)

8. (d)

9. (b)

10. (b)

11. (c)

12. (c)

13. (a)

14. (b)

15. (a)

16. (a)

17. (c)

18. (a)

19. (b)

20. (a)

21. (a)

22. (b)

23. (a)

24. (b)

25. (d)

Solutions

1.The only value that will satisfy will be 2.

2.¼ + 1/6 will give you 5/12.

3.The possible values are 1, 2, 3, 4, 6 and 12. (i.e. the factors of 12)

4.k will be a number of the form 7n + 3. Hence, if you take the value of n as 9, k + 2n will become 7n + 3 + 18 = 7n + 21. This number will be divisible by 7. The numbers 3, 5 and 7 do not provide us with this solution.

5.273 – 272 – 271 = 271 (22 – 2 – 1) = 271(1). Hence option (c) is correct.

6.Solve through options.

7.The closest value will be option (b), since the percentage change will be lowest when the largest number is reduced by one.

8.This is a property of prime numbers greater than 5.

9.He could have dealt a total of 40 cards, in which case Mushtaq would get 9 cards. On getting one card from Mushtaq, the ratio would become 4:1, while on giving away one card to Mushtaq, the ratio would become 3:1.

10.Looking at the options you realise that the correct answer should be a multiple of 25 and 50 both. The option that satisfies the condition of increasing the number of boxes by 3 is 150. (This is found through trial and error.)

11.Trial and error gives you option 3 as the correct answer.

12.Given that the number must have a 57 in it and should be even at the same time, the only numbers possible are 570, 572, 574, 576 and 578. Also, if there is no 5 in the number, you will get 360 more numbers.

13.203 becomes → 2.32 + 0 + 3.30 = 21 → 2.31 + 1.30 = 7. Hence, clearly two steps are required.

14.Trial and error will give option (b) as the correct answer, since 13 × 62 = 26 × 31

15.The solutions are defined as the sum of digits of the answer. Hence, 10 is correct.

16.There are no common factors between 55 and 124. Hence the answer should be 124.

17.At k = –1, the two equations become inconsistent with respect to each other and there will then be no solution to the system of equations.

18.Try with 5, 7, 11. In each case the remainder is 6.

19.Trial and error gives you the answer as 3 Option (b) is correct.

20.Picking up 4 coins will ensure that he wins the game.

21.Option (a) is correct (since the average of any two numbers lies between the numbers.

22.8/2 + 5/2 = 6.5.

23.The answer will be 56 × 12 = 672 → 675. Hence, ` 6.75.

24.The given condition is satisfied only for 24.

25.The answer will be given by 8 + 2 = 10.

(This logic is explained in the Number Systems chapter)

SCORE INTERPRETATION ALGORITHM FOR PRE-ASSESSMENT TEST OF BLOCK I

(Use a similar process for Blocks II to VI on the basis of your performance).

If You Scored: < 7: (In Unlimited Time)

Step One

Go through the block one Back to School Section carefully. Grasp each of the concepts explained in that part carefully. In fact I would recommend that you go back to your Mathematics school books (ICSE/ CBSE) Class 8, 9 and 10 if you feel you need it.

Step Two

Move into the first chapter of the block, viz, Number Systems. When you do so, concentrate on clearly understanding each of the concepts explained in the chapter theory.

Then move onto the LOD 1 exercises. These exercises will provide you with the first level of challenge. Try to solve each and every question provided under LOD 1 of Number systems. While doing so do not think about the time requirement. Once you finish solving LOD 1, revise the questions and their solution processes.

Step Three

After finishing LOD 1 of number systems, move into Chapter 2 of this block—Progressions and repeat the process, viz: chapter theory comprehensively followed by solving LOD 1 questions.

Step Four

Go to the first and second review tests given at the end of the block and solve them. While doing so, first look at the score you get within the mentioned time limit. Then continue to solve the test further without a time limit and try to evaluate the improvement in your unlimited time score.

In case the growth in your score is not significant, go back to the theory of each chapter and review each of the LOD 1 questions for both the chapters.

Step Five

Move to LOD 2 and repeat the process that you followed in LOD 1—first in the chapter of Number Systems, then in the chapter on Progressions. Concentrate on understanding each and every question and its underlying concept.

Step Six

Go to the third to fifth review tests given at the end of the block and solve them. Again, while doing so measure your score within the provided time limit first and then continue to solve the test further without a time limit and try to evaluate the improvement that you have had in your score.

Step Seven

Move to LOD 3 only after you have solved and understood each of the questions in LOD 1 and LOD 2. Repeat the process that you followed in LOD 1—first in the chapter of Number Systems, then with the Chapter on Progressions.

If You Scored: 7–15 (In Unlimited Time)

Although you are better than the person following the instructions above, obviously there is a lot of scope for the development of your score. You will need to work both on your concepts as well as speed. Initially emphasize more on the concept development aspect of your preparations, then move your emphasis onto speed development. The following process is recommended for you:

Step One

Go through the block one Back to School Section carefully. Revise each of the concepts explained in that part. Going through your VIIIth, IXth and Xth standard books will be an optional exercise for you. It will be recommended in case you scored in single digits, while if your score is in two digits, I leave the choice to you.

Step Two

Move into the first chapter of the block. Viz Number Systems. When you do so, concentrate on clearly understanding each of the concepts explained in the chapter theory.

Then move onto the LOD 1 exercises. These exercises will provide you with the first level of challenge. Try to solve each and every question provided under LOD 1 of Number Systems. Once you finish solving LOD 1, revise the questions and their solution processes.

Step Three

After finishing LOD 1 of number systems, move into Chapter 2 of this block—Progressions and repeat the process, viz: Chapter theory comprehensively followed by solving LOD 1 questions.

Step Four

Go to the first and second review tests given at the end of the block and solve them. While doing so, first look at the score you get within the time limit mentioned. Then continue to solve the test further without a time limit and try to evaluate the improvement in your score.

Step Five

Move to LOD 2 and repeat the process that you followed in LOD 1—first with the chapter on Number Systems, then with the chapter on Progressions.

Step Six

Go to the third to fifth review tests given at the end of the block and solve it. Again while doing so measure your score within the provided time limit first and then continue to solve the test further without a time limit and try to evaluate the improvement that you have had in your score.

In case the growth in your score is not significant, go back to the theory of both the chapters and re-solve LOD 1 and LOD 2 of both the chapters. While doing so concentrate more on the LOD 2 questions.

Step Seven

Move to LOD 3 and repeat the process that you followed in LOD 1—first with the chapter on Number Systems, then with the Chapter on Progressions.

If You Scored 15+ (In Unlimited Time)

Obviously you are much better than the first two categories of students. Hence unlike them, your focus should be on developing your speed by picking up the shorter processes explained in this book. Besides, you might also need to pick up concepts that might be hazy in your mind. The following process of development is recommended for you:

Step One

Quickly review the concepts given in the block one Back to School Section. Only go deeper into a concept in case you find it new. Going back to school level books is not required for you.

Step Two

Move into the first chapter of the block: Number Systems. Go through the theory explained there carefully. Concentrate specifically on clearly understanding the concepts which are new to you. Work out the short cuts and in fact try to expand your thinking by trying to think of alternative (and expanded) lines of questioning with respect to the concept you are studying.

Then move onto the LOD 1 exercises. Solve each and every question provided under LOD 1 of Number Systems. While doing so, try to think of variations that you can visualize in the same questions and how you would handle them.

Step Three

After finishing LOD 1 of number systems, move into Chapter 2 of this block, (Progressions) and repeat the process, viz: Chapter theory with emphasis on picking up things that you are unaware of, followed by solving LOD 1 questions and thinking about their possible variations.

Step Four

Move to LOD 2 and repeat the process that you followed in LOD 1—first in the chapter of Number Systems, then with the Chapter on Progressions.

Step Five

Go to the first to fifth review tests—given at the end of the block and solve it. While doing so, first look at the score you get within the time limit mentioned. Then continue to solve the test further without a time limit and try to evaluate the improvement in your score.

Step Six

Move to LOD 3 and repeat the process that you followed in LOD 1—first in the chapter of Number Systems, then with the Chapter on Progressions.

THE NUMBER LINE

Core Concepts

  1. The concept of the number line is one of the most crucial concepts in Basic Numeracy.

The number line is a line that starts from zero and goes towards positive infinity when it moves to the right and towards negative infinity when it moves to the left.

The difference between the values of any two points on the number line also gives the distance between the points.

Thus, for example if we look at the distance between the points + 3 and – 2, it will be given by their difference. 3 – (– 2) = 3 + 2 = 5.

  1. Types of numbers—

We will be looking at the types of numbers in detail again when we go into the chapter of number systems. Let us first work out in our minds the various types of numbers. While doing so do not fail to notice that most of these number types occur in pairs (i.e. the definition of one of them, defines the other automatically).

Note here that the number line is one of the most critical concepts in understanding and grasping numeracy and indeed mathematics.

Multiples on the Number Line

All tables and multiples of every number can be visualised on the number line. Thus, multiples of 7 on the number line would be seen as follows:

In order to visualize this you can imagine a frog jumping consistently 7 units every time. If it lands on –14, the next landing would be on –7, then on 0, then 7 and finally at 14. This is how you can visualise the table of 7 (in mathematical terms we can also refer to this as 7n – meaning the set of numbers which are multiples of 7 or in other words the set of numbers which are divisible by 7).

You can similarly visualise 5n, 4n, 8n and so on—practically all tables on the number line as above.

What do We Understand by 7n + 1 and Other such Notations & the Equivalence of 7n + 1 and 7n – 6

Look at the following figure closely:

Our 7n frog is made to first land on –13 and then asked to keep doing it’s stuff (jumping 7 to the right everytime). What is the result? It lands on –6, + 1, 8, 15 and it’s next landing would be on 22, 29 and so forth. These numbers cannot be described as 7n, but rather they all have a single property which is constant for all numbers.

They can be described as: “One more than a multiple of 7” and in mathematical terms such numbers are also called as 7n + 1. Alternately these numbers also have the property that they are “6 less than multiples of 7” and in mathematical terms such numbers can also be called as 7n – 6.

That is why in mathematics, we say that the set of numbers represented by 7n + 1 is the same as the set of numbers represented by 7n – 6.

The Implication in Terms of Remainders

This concept can also be talked about in the context of remainders.

When a number which can be described as 7n + 1 or 7n – 6 (like the numbers 8, 15, –6, –13, –20, –27, –34, –41…) is divided by 7, the remainder in every case is seen to be 1. For some people reconciling the fact that the remainder when –27 is divided by 7 the remainder is 1, seems difficult on the surface. Note that this needs to be done because about remainders we should know that remainders are always non-negative.

However, the following thinking would give you the remainder in every case:

27/7, remainder is 6.

–27/7, remainder is –6. In the context of dividing by 7, a remainder of –6 means a remainder of 7 – 6 = 1.

Let us look at another example:

What is the remainder when –29 is divided by 8.

First reaction 29/8 Æ remainder 5, –29/8 Æ remainder –5, hence actual remainder is 8 – 5 = 3.

The student is advised to practice more such situations and get comfortable in converting positive remainders to negative remainders and vice versa.

Even and Odd Numbers

The meaning of 2n and 2n + 1: 2n means a number which is a multiple of the number 2. Since, this can be visualised as a frog starting from the origin and jumping 2 units to the right in every jump, you can also say that this frog represents 2n.

(Note: Multiples of 2, are even numbers. Hence, 2n is also used to denote even numbers.)

So, what does 2n + 1 mean?

Well, simply put, if you place the above frog on the point represented by the number 1 on the number line then the frog will reach points such as 3, 5, 7, 9, 11 …..and so on. This essentially means that the points the frog now reaches are displaced by 1 unit to the right of the 2n frog. In mathematical terms, this is represented as 2n + 1.

In other words, 2n + 1 also represents numbers which leave a remainder of 1, when divided by 2. (Note: This is also the definition of an odd number. Hence, in Mathematics (2n + 1) is used to denote an odd number. Also note that taken together 2n and 2n + 1 denote the entire set of integers. i.e. all integers from – • to + • on the number line can be denoted by either 2n or 2n + 1. This happens because when we divide any integer by 2, there are only two results possible with respect to the remainder obtained, viz: A remainder of zero (2n) or a remainder of one (2n + 1).

This concept can be expanded to represent integers with respect to any number. Thus, in terms of 3, we can only have three types of integers 3n, 3n + 1 or 3n + 2 (depending on whether the integer leaves a remainder 0, 1 or 2 respectively when divided by 3.) Similarly, with respect to 4, we have 4 possibilities—4n, 4n + 1, 4n + 2 or 4n + 3.

Needless to say, from these representations above, the representations 2n and 2n + 1 (which can also be represented as 2n – 1) have great significance in Mathematics as they represent even and odd numbers respectively. Similarly, we use the concept of 4n to check whether a year is a leap year or not.

These representations can be seen on the number line as follows:

Representation of 2n and 2n + 1:

Representation of 3n, 3n + 1 and 3n + 2:

Representation of 4n, 4n + 1, 4n + 2 and 4n + 3:

One Particular Number can be a Multiple of more than 1 Number—The Concept of Common Multiples

Of course one of the things you should notice as you go through the above discussion is that individual numbers can indeed be multiples of more than 1 number—and actually often are.

Thus, for instance the number 14 is a multiple of both 7 and 2—hence 14 can be called as a common multiple of 2 and 7. Obviously, I think you can visualise more such numbers which can be classified as common multiples of 2 and 7?? 28,42,56 and in fact the list is infinite—i.e. the numbers never end. Thus, the common multiples of 2 and 7 can be represented by the infinite set:

{14,28,42,56,70…………1400……..14000……140000……and so on}

The LCM and It’s Significance

From the above list, the number 14 (which is the lowest number in the set of Common multiples of 2 and 7) has a lot of significance in Mathematics. It represents what is commonly known as the Least Common Multiple (LCM)of 2 and 7. It is the first number which is a multiple of both 2 and 7 and there are a variety of questions in numeracy which you would come across—not only when you solve questions based on number systems (where the LCM has it’s dedicated set of questions), but applications of the LCM are seen even in chapters like Time, Speed Distance, Time and Work etc.

So what did school teach us about the process of finding LCM?

Before you start to review/ relearn that process you first need to know about prime factors of a number.

I hope at this point you recognize the difference between finding factors of a number and prime factors of a number.

Simply put, finding factors of a number means finding the divisors of the number. Thus, for instance the factors of the number 80 would be the numbers 1, 2, 4, 5, 8, 10, 16, 20, 40 and 80. On the other hand finding the prime factors of the same number would mean writing the number 80 as 24 × 51. This form of writing the number is also called the STANDARD FORM or the CANONICAL FORM of the number.

The school process of writing down the standard form of a number:

Now this is something I would think most of you would remember and recognize:

Finding the prime factors of the number 80

2

80

2

40

2

20

2

10

5

5

1

The prime factors of the number 80 are: 2 × 2 × 2 × 2 × 5 = 24 × 51.

Exercise for Self-practice

Write down the standard form of the following numbers.

(a) 20

(b) 44

(c) 142

(d) 200

(e) 24

(f) 324

(g) 120

(h) 84

(i) 371

(j) 143

(k) 339

(l) 147

(m) 1715

Finding the LCM of two or more numbers: The school process

Step 1: Write down the prime factor form of all the numbers;

Let us say that you have 3 numbers whose standard forms are:

24 × 32 × 53 × 71

23 × 31 × 52 × 112

24 × 32 × 51 × 131

To write down the LCM of these numbers write down all the prime numbers and multiply them with their highest available powers. The resultant number would be the LCM of these numbers.

Thus, in the above case the LCM would be:

24 × 32 × 53 × 71 × 112 × 131

Note: Short cuts to a lot of these processes have been explained in the main chapters of the book.

Divisors (Factors) of a number

As we already mentioned, finding the factors or divisors of a number are one and the same thing. In order to find factors of a number, the key is to spot the factors below the square root of the number. Once you have found them, the factors above the square root would be automatically seen. Consider this for factors of the number 80:

Factors below the square root of 80 (8.xxx) Hence, factors up to and including 8

1

2

4

5

8

Once you can visualise the list on the left, the factors on the right would be seen automatically.

Factors below the square root of 80 (8.xxx) Hence, factors up to and including 8

Factors above the square root

1

80

2

40

4

20

5

16

8

10

Note that these will be seen automatically the moment you have the list on the left.

Common Divisors Between 2 Numbers

List of Common Divisors

When we write down the factors of two numbers, we can look for the common elements within the two lists.

For instance, the factors of 80 are (1, 2, 4, 5, 8, 10, 16, 20, 40 and 80) while the factors of 144 are (1, 2, 3, 4, 6, 8, 9, 12, 16, 18, 24, 36, 48, 72 and 144)

If you observe the two lists closely you will get the following list of common factors or common divisors between the two:

List of common factors of 80 and 144: (1, 2, 4, 8, 16)

The number 16 being the highest in this list of common factors (or divisors) is also called the Highest Common Factor or the Greatest Common Divisor. In short it is denoted as HCF or GCD. It has a lot of significance in terms of quantitative thinking—as the HCF is used in a multitude of problems and hence being able to spot the HCF of two or more numbers is one of the critical operations in Mathematics. You would be continuously seeing applications of the HCF in problems in the chapter of Number systems and right through the chapters of arithmetic (defined as word based problems in this book).

Rounding off and its use for approximation

One of the important things that we learnt in school was the use of approximation in order to calculate.

Thus, 72 × 53 can be approximated to 70 × 50 and hence seen as 3500.

Similarly, the addition of 48 + 53 + 61 + 89 can be taken as 50 + 50 + 60 + 90 = 250

Number Types You Should Know

Integers and Decimals: All numbers that do not have a decimal in them are called integers. Thus, –3, –17, + 4, + 13, + 1473, 0 etc are all integers.

Obviously, decimal numbers are numbers which have a decimal value attached to them. Thus, 1.3, 14.76, – 12.24 etc are all decimal numbers, since they have certain values after the decimal point.

Before we move ahead, let us pause a brief while, to further understand decimals. As you shall see, the concept of decimals is closely related to the concept of division and divisibility. Suppose, I have 4 pieces of bread which I want to divide equally between two people. It is easy for me to do this, since I can give two whole pieces to each of them.

However, if we alter the situation in such a way, that I now have 5 pieces of bread to distribute equally amongst 2 people. What do I do?

I give two whole pieces each, to each of them. The 5th piece has to be divided equally between the two. I can no longer do this, without in some way breaking the 5th piece into 2 parts. This is the elementary situation that gives rise to the need for decimals in mathematics.

Going back to the situation above, my only option is to divide the 5th piece into two equal parts (which in quants are called as halves).

This concept has huge implications for problem solving especially once you recognise that a half (i.e. a ‘.5’ in the decimal) only comes when you divide a whole into two parts.

Thus, in fact, all standards decimals emerge out of certain fixed divisors.

Hence, for example, the divisor 2, gives rise to the decimal · 5.

Similarly the divisor 3 gives rise to the decimals ·33333 and ·66666, etc.

Prime numbers and Composite numbers Amongst natural numbers, there are three broad divisions–

Unity It is representative of the number 1.

Prime numbers These are numbers which have no divisors/ factors apart from 1 and itself.

Composite numbers On the other hand, are numbers, which have at least one more divisor apart from 1 and itself.

Note: A brief word about factors/ division—A number X is said to divide Y (or is said to be a divisor or factor of Y) when the division of Y/X leaves no remainder.

All composite numbers have the property that they can be written as a product of their prime factors.

Thus, for instance, the number 40 can be represented as: 40 = 2 × 2 × 2 × 5 or 40 = 23 × 51

This form of writing is called as the standard form of the composite number.

The difference between Rational and Irrational numbers: This difference is one of the critical but unfortunately one of the less well understood differences in elementary Mathematics.

The definition of Rational numbers: Numbers which can be expressed in the form p/q where q π 0 are called rational numbers.

Obviously, numbers which cannot be represented in the form p/q are called as irrational numbers.

However, one of the less well understood issues in this regard is what does this mean?

The difference becomes clear when the values of decimals are examined in details:

Consider the following numbers.

(1)4.2,

(2)4.333….,

(3)4.1472576345…….

What is the difference between the decimal values of the three numbers above?

To put it simply, the first number has what can be described as a finite decimal value. Such numbers can be expressed in the form p/q easily. Since 4.2 can be first written as 42/10 and then converted to 21/5.

Similarly, numbers like 4.5732 can be represented as 45732/10000. Thus, numbers having a finite terminating decimal value are rational.

Now, let us consider the decimal value: 4.3333……..

Such decimal values will continue endlessly, i.e. they have no end. Hence, they are called infinite decimals (or non-terminating decimals).

But, we can easily see that the number 4.333.... can be represented as 13/3. Hence, this number is also rational. In fact, all numbers which have infinite decimal values, but have any recurring form within them can be represented in the p/q form.

For example the value of the number: 1.14814814814…. is 93/81.

(What I mean to say is that whenever you have any recurring decimal number, even if the value of ‘q’ might not be obvious, but it will always exist.)

Thus, we can conclude that all numbers whose decimal values are infinite (non-terminating) but which have a recurring pattern within them are rational numbers.

This leaves us with the third kind of decimal values, viz. Infinite non-recurring decimal values. These decimals neither have a recurring pattern, nor do they have an end—they go on endlessly. For such numbers it is not possible to find the value of a denominator ‘q’ which can be used in order to represent them as p/q. Hence, such numbers are called as irrational numbers.

In day-to-day mathematics, we come across numbers like , , , p, e, etc. which are irrational numbers since they do not have a p/q representation.

Note: can also be represented as 31/2, just as can be represented as 71/3.

An Important Tip:

Rational and Irrational numbers do not mix.: This means that in case you get a situation where an irrational number has appeared while solving a question, it will remain till the end of the solution. It can only be removed from the solution if it is multiplied or divided by the same irrational number.

Consider an example: The area of an equilateral triangle is given by the formula (/4) × a2 (where a is the side of the equilateral triangle). Since, is an irrational number, it remains in the answer till the end. Hence, the area of an equilateral triangle will always have a as part of the answer.

Before we move ahead we need to understand one final thing about recurring decimals.

As I have already mentioned, recurring decimals have the property of being able to be represented in the p/q form. The question that arises is—Is there any process to convert a recurring decimal into a proper fraction?

Yes, there is. In fact, in order to understand how this operates, you first need to understand that there are two kinds of recurring decimals. The process for converting an infinite recurring decimal into a fraction basically varies for both of these types. Let’s look at these one by one.

Type 1—Pure recurring decimals: These are recurring decimals where the recurrence starts immediately after the decimal point.

For example:

0.5555… =

3.242424… =

5.362362… =

The process for converting these decimals to fractions can be illustrated as:

0.5555 = 5/9

3.242424 = 3 + (24/99)

5.362362 = 5 + (362/999)

A little bit of introspection will tell you that what we have done is nothing but to put down the recurring part of the decimal as it is and dividing it by a group of 9’s. Also the number of 9’s in this group equals the number of digits in the recurring part of the decimal.

Thus, in the second case, the fraction is derived by dividing 24 by 99. (24 being the recurring part of the decimal and 99 having 2 nines because the number of digits in 24 is 2.)

Similarly, 0.43576254357625…. =

Type 2—Impure recurring decimals: Unlike pure recurring decimals, in these decimals, the recurrence occurs after a certain number of digits in the decimal. The process to convert these into a fraction is also best illustrated by an example:

Consider the decimal 0.435424242

=

The fractional value of the same will be given by: (43542 – 435)/99000. This can be understood in two steps.

Step 1: Subtract the non-recurring initial part of the decimal (in this case, it is 435) from the number formed by writing down the starting digits of the decimal value upto the digit where the recurring decimals are written for the first time;

Expanding the meaning—

Note: For 0.435424242, subtract 435 from 43542

Step 2: The number thus obtained, has to be divided by a number formed as follows; Write down as many 9’s as the number of digits in the recurring part of the decimal. (in this case, since the recurring part ‘42’ has 2 digits, we write down 2 9’s.) These nines have to be followed by as many zeroes as the number of digits in the non recurring part of the decimal value. (In this case, the non recurring part of the decimal value is ‘435’. Since, 435 has 3 digits, attach three zeroes to the two nines to get the number to divide the result of the first step.)

Hence divide 43542 – 435 by 99000 to get the fraction.

Similarly, for 3.436213213 we get

Mixed Fractions

A mixed number is a whole number plus a fraction. Here are a few mixed numbers:

In order to convert a mixed fraction to a proper fraction you do the following conversion process.

= (1 × 2 + 1)/2 = 3/2

= (2 × 3 + 1)/3 = 7/3

Similiarly, = 5/4

= 16/7

= 29/5 and so on.

i.e. multiply the whole number part of the mixed number by the denominator of the fractional part and add the resultant to the numerator of the fractional part to get the numerator of the proper fraction. The denominator of the proper fraction would be the same as the denominator of the mixed fraction.

OPERATIONS ON NUMBERS

Exponents and Powers

Exponents, or powers, are an important part of math as they are necessary to indicate that a number is multiplied by itself for a given number of times.

When a number is multiplied by itself it gives the ‘square of the number’.

Thus, n × n = n2 (for example 3 × 3 = 32)

If the same number is multiplied by itself twice we get the cube of the number.

Thus, n × n × n = n3 (for example 3 × 3 × 3 = 33)

n × n × n × n = n4 and so on.

With respect to powers of numbers, there are 5 basic rules which you should know:

For any number ‘n’ the following rules would apply:

Rule 1: na × nb = n(a + b). Thus, 43 × 45 = 48

Rule 2: na / nb =na b . Thus, 39/34 = 35

Rule 3: (na)b = nab. Thus, (32)4= 38.

Rule 4: n(–a)= 1/na. Thus, 3–4 = 1/34.

Rule 5: n0 = 1. Thus, 50 = 1.

General Form of Writing 2-3 Digit Numbers

In mathematics many a time we have to use algebraic equations in order to solve questions. In such cases an important concept is the way we represent two or three digit numbers in equation form.

For instance, suppose we have a 2 digit number with the digits ‘AB’.

In order to write this in the form of an equation, we have to use:

10A + B. This is because in the number ‘AB’ the digit A is occupying the tens place. Hence, in order to represent the value of the number ‘AB’ in the form of an equation- we can write 10A + B.

Thus, the number 29 = 2 × 10 + 9 × 1

Similarly, for a three digit number with the digits A, B and C respectively – the number ‘ABC ’ can be represented as below:

ABC = 100 A + 10 B + C.

Thus,243 = 2 × 100 + 4 × 10 + 3 × 1

The BODMAS Rule: It is used for the ordering of mathematical operations in a mathematical situation:

In any mathematical situation, the first thing to be considered is Brackets followed by Division, Multiplication, Addition and Subtraction in that order.

Thus 3 × 5 – 2 = 15 – 2 = 13

Also, 3 × 5 – 6 ∏ 3 = 15 – 2 = 13

Also, 3 × (5 – 6) ∏ 3 = 3 × (– 1) ∏ 3 = – 1.

Operations on Odd and Even numbers

ODD    SEVENS

ODDS & EVENS

Odd ∏ Even → Not divisible

SERIES OF NUMBERS

In many instances in Mathematics we are presented with a series of numbers formed simply when a group of numbers is written together. The following are examples of series:

1.3, 5, 8, 12, 17…

2.3, 7, 11, 15, 19…(Such series where the next term is derived by adding a certain fixed value to the previous number are called as Arithmetic Progressions).

3.5, 10, 20, 40 …..(Such series where the next term is derived by multiplying the previous term by a fixed value are called as Geometric Progressions).

(Note: You will study AP and GP in details in the chapter of progressions which is chapter 2 of this block.)

4.2, 7, 22, 67 ….

5.1/3, 1/5, 1/7, 1/9, 1/11…

6.1/12, 1/22 , 1/32, ¼2, 1/52

7.1/13, 1/33, 1/53

Remember the following points at this stage:

1.AP and GP are two specific instances of series. They are studied in details only because they have many applications and have defined rules.

2.Based on the behaviour of their sums, series can be classified as:

Divergent: These are series whose sum to ‘n’ terms keeps increasing and reaches infinity for infinite terms.

Convergent: Convergent series have the property that their sum tends to approach an upper limit/lower limit as you include more terms in the series. They have the additional property that even when infinite terms of the series are included they will only reach that value and not cross it.

For example consider the series;

1/12 + 1/32 + 1/52 + 1/72

It is evident that subsequent terms of this series keep getting smaller. Hence, their value becomes negligible after a few terms of the series are taken into account.

If taken to infinite terms, the sum of this series will reach a value which it will never cross. Such series are called convergent, because their sum converges to a limit and only reaches that limit for infinite terms.

Note: Questions on finding infinite sums of convergent series are very commonly asked in most aptitude exams including CAT and XAT.

NOTE TO THE READER: NOW THAT YOU ARE THROUGH WITH THE BACK TO SCHOOL SECTION, YOU ARE READY TO PROCEED INTO THE CHAPTERS OF THIS BLOCK. HAPPY SOLVING!!

Chapter 1. NUMBER SYSTEMS

INTRODUCTION

The chapter on Number Systems is amongst the most important chapters in the entire syllabus of Quantitative Aptitude for the CAT examination (and also for other parallel MBA entrance exams).Students are advised to go through this chapter with utmost care understanding each concept and question type on this topic. The CAT has consistently contained anything between 20–40% of the marks based on questions taken from this chapter. Naturally, this chapter becomes one of the most crucial as far as your quest to reach close to the qualification score in the section of Quantitative Aptitude and Data Interpretation is concerned.

Hence, going through this chapter and its concepts properly is imperative for you. It would be a good idea to first go through the basic definitions of all types of numbers. Also closely follow the solved examples based on various concepts discussed in the chapter. Also, the approach and attitude while solving questions on this chapter is to try to maximize your learning experience out of every question. Hence, do not just try to solve the questions but also try to think of alternative processes in order to solve the same question. Refer to hints or solutions only as a last resort.

To start off, the following pictorial representation of the types of numbers will help you improve your quality of comprehension of different types of numbers.

DEFINITION

Natural Numbers These are the numbers (1, 2, 3, etc.) that are used for counting. In other words, all positive integers are natural numbers.

There are infinite natural numbers and the number 1 is the least natural number.

Examples of natural numbers: 1, 2, 4, 8, 32, 23, 4321 and so on.

The following numbers are examples of numbers that are not natural: –2, –31, 2.38, 0 and so on.

Based on divisibility, there could be two types of natural numbers: Prime and Composite.

Prime Numbers A natural number larger than unity is a prime number if it does not have other divisors except for itself and unity.

Note: Unity (i.e. 1) is not a prime number.

Some Properties of Prime Numbers

■The lowest prime number is 2.

■2 is also the only even prime number.

■The lowest odd prime number is 3.

■The remainder when a prime number p ≥ 5 is divided by 6 is 1 or 5. However, if a number on being divided by 6 gives a remainder of 1 or 5 the number need not be prime. Thus, this can be referred to as a necessary but not sufficient condition.

■The remainder of the division of the square of a prime number p ≥ 5 divided by 24 is 1.

■For prime numbers p > 3, p2 – 1 is divisible by 24.

■Prime Numbers between 1 to 100 are: 2, 3, 5, 7, 11, 13, 17, 19, 23, 29, 31, 37, 41, 43, 47, 53, 59, 61, 67, 71, 73, 79, 83, 89, 97.

■Prime Numbers between 100 to 200 are: 101, 103, 107, 109, 113, 127, 131, 137, 139, 149, 151, 157, 163, 167, 173, 179, 181, 191, 193, 197, 199.

■If a and b are any two odd primes then a2b2 is composite. Also, a2 + b2 is composite.

■The remainder of the division of the square of a prime number p ≥ 5 divided by 12 is 1.

SHORT CUT PROCESS

To Check Whether a Number is Prime or Not

To check whether a number N is prime, adopt the following process.

(a)Take the square root of the number.

(b)Round of the square root to the immediately lower integer. Call this number z. For example if you have to check for 181, its square root will be 13. Hence, the value of z, in this case will be 13.

(c)Check for divisibility of the number N by all prime numbers below z. If there is no prime number below the value of z which divides N then the number N will be prime.

To illustrate :-

The value of lies between 15 to 16. Hence, take the value of z as 15.

Prime numbers less than 16 are 2, 3, 5, 7, 11 and 13, 239 is not divisible by any of these. Hence you can conclude that 239 is a prime number.

A Brief Look into why this Works?

Suppose you are asked to find the factors of the number 40.

An untrained mind will find the factors as : 1, 2, 4, 5, 8, 10, 20 and 40.

The same task will be performed by a trained mind as follows:

 1 × 40

 2 × 20

 4 × 10

and 5 × 8

i.e., The discovery of one factor will automatically yield the other factor. In other words, factors will appear in terms of what can be called as factor pairs. The locating of one factor, will automatically pinpoint the other one for you. Thus, in the example above, when you find 5 as a factor of 40, you will automatically get 8 too as a factor.

Now take a look again at the pairs in the example above. If you compare the values in each pair with the square root of 40 (i.e. 6. ) you will find that for each pair the number in the left column is lower than the square root of 40, while the number in the right column is higher than the square root of 40.

This is a property for all numbers and is always true.

Hence, we can now phrase this as: Whenever you have to find the factors of any number N, you will get the factors in pairs (i.e. factor pairs). Further, the factor pairs will be such that in each pair of factors, one of the factors will be lower than the square root of N while the other will be higher than the square root of N.

As a result of this fact one need not make any effort to find the factors of a number above the square root of the number. These come automatically. All you need to do is to find the factors below the square root of the number.

Extending this logic, we can say that if we are not able to find a factor of a number upto the value of its square root, we will not be able to find any factor above the square root and the number under consideration will be a prime number. This is the reason why when we need to check whether a number is prime, we have to check for factors only below the square root.

But, we have said that you need to check for divisibility only with the prime numbers below (and including) the square root of the number. What logic will explain this:

Let us look at an example to understand why you need to look only at prime numbers below the square root.

Uptil now, we have deduced that in order to check whether a number is prime, we just need to do a factor search below (and including) the square root.

Thus, for example, in order to find whether 181 is a prime number, we need to check with the numbers = 2, 3, 4, 5, 6, 7, 8, 9, 10, 11, 12, and 13.

The first thing you will realise, when you first look at the list above is that all even numbers will get eliminated automatically (since no even number can divide an odd number and of course you will check a number for being prime only if it is odd!)

This will leave you with the numbers 3, 5, 7, 9, 11 and 13 to check 181.

Why do we not need to check with composite numbers below the square root? This will again be understood best if explained in the context of the example above. The only composite number in the list above is 9. You do not need to check with 9, because when you checked N for divisibility with 3 you would get either of two cases:

Case I: If N is divisible by 3: In such a case, N will automatically become non-prime and you can stop your checking. Hence, you will not need to check for the divisibility of the number by 9.

Case II: N is not divisible by 3: If N is not divisible by 3, it is obvious that it will not be divisible by 9. Hence, you will not need to check for the divisibility of the number by 9.

Thus, in either case, checking for divisibility by a composite number (9 in this case) will become useless. This will be true for all composite numbers.

Hence, when we have to check whether a number N is prime or not, we need to only check for its divisibility by prime factors below the square root of N.

Finding Prime Numbers: The Short Cut

Using the logic that we have to look at only the prime numbers below the square root in order to check whether a number is prime, we can actually cut short the time for finding whether a number is prime drastically.

Before I start to explain this, you should perhaps realise that in an examination like the CAT, or any other aptitude test for that matter whenever you would need to be checking for whether a number is prime or not, you would typically be checking 2 digit or maximum 3 digit numbers in the range of 100 to 200.

Also, one would never really need to check with the prime number 5, because divisibility by 5 would automatically be visible and thus, there is no danger of anyone ever declaring a number like 35 to be prime. Hence, in the list of prime numbers below the square root we would never include 5 as a number to check with.

Checking Whether a Number is Prime (For Numbers below 49)

The only number you would need to check for divisibility with is the number 3. Thus, 47 is prime because it is not divisible by 3.

Checking Whether a Number is Prime (For Numbers above 49 and below 121)

Naturally you would need to check this with 3 and 7. But if you remember that 77, 91 and 119 are not prime, you would be able to spot the prime numbers below 121 by just checking for divisibility with the number 3.

Why? Well, the odd numbers between 49 and 121 which are divisible by 7 are 63, 77, 91, 105 and 119. Out of these perhaps 91 and 119 are the only numbers that you can mistakenly declare as prime. 77 and 105 are so obviously not-prime that you would never be in danger of declaring them prime.

Thus, for numbers between 49 and 121 you can find whether a number is prime or not by just dividing by 3 and checking for its divisibility.

For example:

61, is prime because it is not divisible by 3 and it is neither 91 nor 119.

Checking Whether a Number is Prime (For Numbers above 121 and below 169)

Naturally you would need to check this with 3, 7 and 11. But if you remember that 133,143 and 161 are not prime, you would be able to spot the prime numbers between 121 to 169 by just checking for divisibility with the number 3.

Why? The same logic as explained above. The odd numbers between 121 and 169 which are divisible by either 7 or 11 are 133,143,147,161 and 165. Out of these 133,143 and 161 are the only numbers that you can mistakenly declare as prime if you do not check for 7 or 11. The number 147 would be found to be not prime when you check its divisibility by 3 while the number 165 you would never need to check for, for obvious reasons.

Thus, for numbers between 121 and 169 you can find whether a number is prime or not by just dividing by 3 and checking for its divisibility.

For example:

149, is prime because it is not divisible by 3 and it is neither 133,143 nor 161.

Thus, we have been able to go all the way till 169 with just checking for divisibility with the number 3.

This logic can be represented on the number line as follows:

Integers A set which consists of natural numbers, negative integers (–1, –2, –3…–n…) and zero is known as the set of integers. The numbers belonging to this set are known as integers.

Integers can be visualised on the number line:

Note: Positive integers are the same thing as natural numbers.

The moment you define integers, you automatically define decimals.

Decimals

A decimal number is a number with a decimal point in it, like these: 1.5, 3.21, 4.173, 5.1 etc

The number to the left of the decimal is an ordinary whole number. The first number to the right of the decimal is the number of tenths (1/10’s). The second is the number of hundredths (1/100’s) and so on. So, for the number 5.1, this is a shorthand way of writing the mixed number . 3.27 is the same as 3 + 2/10 + 7/100.

A word on where decimals originate from

Consider the situation where there are 5 children and you have to distribute 10 chocolates between them in such a way that all the chocolates should be distributed and each child should get an equal number of chocolates? How would you do it? Well, simple—divide 10 by 5 to get 2 chocolates per child.

Now consider what if you had to do the same thing with 9 chocolates amongst 5 children? In such a case you would not be able to give an integral number of chocolates to each person. You would give 1 chocolate each to all the 5 and the ‘remainder’ 4 would have to be divided into 5 parts. 4 out of 5 would give rise to the decimal 0.8 and hence you would give 1.8 chocolates to each child. That is how the concept of decimals enters mathematics in the first place.

Taking this concept further, you can realize that the decimal value of any fraction essentially emerges out of the remainder when the numerator of the fraction is divided by the denominator. Also, since we know that each divisor has a few defined remainders possible, there would be a limited set of decimals that each denominator gives rise to.

Thus, for example the divisor 4 gives rise to only 4 remainders (viz. 0,1,2 and 3) and hence it would give rise to exactly 4 decimal values when it divides any integer. These values are:

0 (when the remainder is 0)

.25 (when the remainder is 1)

.50 (when the remainder is 2)

.75 (when the remainder is 3)

There would be similar connotations for all integral divisors—although the key is to know the decimals that the following divisors give you:

Primary list:

2, 3, 4, 5, 6, 7, 8, 9, 10, 11, 12, 15, 16

Secondary list:

18, 20, 24, 25, 30, 40, 50, 60, 80, 90, 120

Composite Numbers It is a natural number that has at least one divisor different from unity and itself.

Every composite number n can be factored into its prime factors. (This is sometimes called the canonical form of a number.)

In mathematical terms: n = p1m . p2npks, where p1, p2pk are prime numbers called factors and m, nk are natural numbers.

Thus, 24 = 23 ◊ 3, 84 = 7 ◊ 3 ◊ 22 etc.

This representation of a composite number is known as the standard form of a composite number. It is an extermely useful form of seeing a composite number as we shall see.

Whole Numbers The set of numbers that includes all natural numbers and the number zero are called whole numbers. Whole number are also called as Non-negative integers.

The Concept of the Number Line The number line is a straight line between negative infinity on the left to positive infinity to the right.

The distance between any two points on the number line is got by subtracting the lower value from the higher value. Alternately, we can also start with the lower number and find the required addition to reach the higher number.

For example: The distance between the points 7 and –4 will be 7 – (–4) = 11.

Real Numbers All numbers that can be represented on the number line are called real numbers. Every real number can be approximately replaced with a terminating decimal.

The following operations of addition, subtraction, multiplication and division are valid for both whole numbers and real numbers: [For any real or whole numbers a, b and c].

(a)Commutative property of addition: a + b = b + a.

(b)Associative property of addition: (a + b) + c = a + (b + c).

(c)Commutative property of multiplication: ab = ba.

(d)Associative property of multiplication: (ab) ◊ c = a ◊ (bc).

(e)Distributive property of multiplication with respect to addition: (a + b) c = ac + bc.

(f)Subtraction and division are defined as the inverse operations to addition and multiplication respectively.

Thus if a + b = c, then c b = a and if q = a/b then bq = a (where b π 0).

Division by zero is not possible since there is no number q for which b q equals a non zero number a.

Rational Numbers A rational number is defined as a number of the form a/b where a and b are integers and b π 0.

The set of rational numbers encloses the set of integers and fractions. The rules given above for addition, subtraction, multiplication and division also apply on rational numbers.

Rational numbers that are not integral will have decimal values. These values can be of two types:

(a)Terminating (or finite) decimal fractions: For example, 17/4 = 4.25, 21/5 = 4.2 and so forth.

(b)Non-terminating decimal fractions: Amongst non- terminating decimal fractions there are two types of decimal values:

(i)Non-terminating periodic fractions: These are non- terminating decimal fractions of the type x a1a2a3a4 ana1a2a3a4 ana1a2a3a4 an. For example = 5.3333, 15.23232323, 14.287628762 876 … and so on.

(ii)Non-terminating non-periodic fractions: These are of the form xb1b2b3b4bnc1c2c3cn. For example: 5.2731687143725186....

Of the above categories, terminating decimal and non-terminating periodic decimal fractions belong to the set of rational numbers.

Irrational Numbers Fractions, that are non-terminating, non-periodic fractions, are irrational numbers.

Some examples of irrational numbers are , etc. In other words, all square and cube roots of natural numbers that are not squares and cubes of natural numbers are irrational. Other irrational numbers include p, e and so on.

Every positive irrational number has a negative irrational number corresponding to it.

All operations of addition, subtraction, multiplication and division applicable to rational numbers are also applicable to irrational numbers.

As briefly stated in the Back to School section, whenever an expression contains a rational and an irrational number together, the two have to be carried together till the end. In other words, an irrational number once it appears in the solution of a question will continue to appear till the end of the question. This concept is particularly useful in Geometry For example: If you are asked to find the ratio of the area of a circle to that of an equilateral triangle, you can expect to see a p / in the answer. This is because the area of a circle will always have a p component in it, while that of an equilateral triangle will always have .

You should realise that once an irrational number appears in the solution of a question, it can only disappear if it is multiplied or divided by the same irrational number.

THE CONCEPT OF GCD (GREATEST COMMON DIVISOR OR HIGHEST COMMON FACTOR)

Consider two natural numbers n, and n2.

If the numbers n1 and n2 are exactly divisible by the same number x, then x is a common divisor of n1 and n2.

The highest of all the common divisors of n1 and n2 is called as the GCD or the HCF. This is denoted as GCD (n1, n2).

Rules for Finding the GCD of Two Numbers n1 and n2

(a)Find the standard form of the numbers n1 and n2.

(b)Write out all prime factors that are common to the standard forms of the numbers n1 and n2.

(c)Raise each of the common prime factors listed above to the lesser of the powers in which it appears in the standard forms of the numbers n1 and n2.

(d)The product of the results of the previous step will be the GCD of n1 and n2.

Illustration: Find the GCD of 150, 210, 375.

Step 1: Writing down the standard form of numbers

150 = 5 × 5 × 3 × 2

210 = 5 × 2 × 7 × 3

375 = 5 × 5 × 5 × 3

Step 2: Writing Prime factors common to all the three numbers is 51 × 31

Step 3: This will give the same result, i.e. 51 × 31

Step 4: Hence, the HCF will be 5 × 3 = 15

For practice, find the HCF of the following:

(a)78, 39, 195

(b)440, 140, 390

(c)198, 121, 1331

SHORTCUT FOR FINDING THE HCF

The above ‘school’ process of finding the HCF (or the GCD) of a set of numbers is however extremely cumbersome and time taking. Let us take a look at a much faster way of finding the HCF of a set of numbers.

Suppose you were required to find the HCF of 39,78 and 195 (first of the practice problems above)

Logic The HCF of these numbers would necessarily have to be a factor (divisor) of the difference between any pair of numbers from the above 3. i.e. the HCF has to be a factor of (78 – 39 = 39) as well as of (195 – 39 = 156) and (195 – 78 = 117). Why?

Well the logic is simple if you were to consider the tables of numbers on the number line.

For any two numbers on the number line, a common divisor would be one which divides both. However, for any number to be able to divide both the numbers, it can only do so if it is a factor of the difference between the two numbers. Got it??

Take an example:

Let us say we take the numbers 68 and 119. The difference between them being 51, it is not possible for any number outside the factor list of 51 to divide both 68 and 119. Thus, for example a number like 4, which divides 68 can never divide any number which is 51 away from 68- because 4 is not a factor of 51.

Only factors of 51, i.e. 51,17,3 and 1 ‘could’ divide both these numbers simultaneously.

Hence, getting back to the HCF problem we were trying to tackle—take the difference between any two numbers of the set—of course if you want to reduce your calculations in the situation, take the difference between the two closest numbers. In this case that would be the difference between 78 and 39 =39.

The HCF has then to be a factor of this number. In order to find the factors quickly remember to use the fact we learnt in the back to school section of this part of the book—that whenever we have to find the list of factors/divisors for any number we have to search the factors below the square root and the factors above the square root would be automatically visible)

A factor search of the number 39 yields the following factors:

1 × 39

3 × 13

Hence, one of these 4 numbers has to be the HCF of the numbers 39,78 and 195. Since we are trying to locate the Highest common factor—we would begin our search from the highest number (viz:39)

Check for divisibility by 39 Any one number out of 39 and 78 and also check the number 195 for divisibility by 39. You would find all the three numbers are divisible by 39 and hence 39 can be safely taken to be the correct answer for the HCF of 39,78 and 195.

Suppose the numbers were:

39, 78 and 182?

The HCF would still be a factor of 78–39=39. The probable candidates for the HCF’s value would still remain 1,3,13 and 39.

When you check for divisibility of all these numbers by 39, you would realize that 182 is not divisible and hence 39 would not be the HCF in this case.

The next check would be with the number 13. It can be seen that 13 divides 39 (hence would automatically divide 78- no need to check that) and also divides 182. Hence, 13 would be the required HCF of the three numbers.

Typical questions where HCF is used directly

Question 1: The sides of a hexagonal field are 216, 423, 1215, 1422, 2169 and 2223 meters. Find the greatest length of tape that would be able to exactly measure each of these sides without having to use fractions/parts of the tape?

In this question we are required to identify the HCF of the numbers 216,423,1215, 1422, 2169 and 2223.

In order to do that, we first find the smallest difference between any two of these numbers. It can be seen that the difference between 2223-2169 = 54. Thus, the required HCF would be a factor of the number 54.

The factors of 54 are:

1 × 54

2 × 27

3 × 18

6 × 9

One of these 8 numbers has to be the HCF of the 6 numbers. 54 cannot be the HCF because the numbers 423 and 2223 being odd numbers would not be divisible by any even number. Thus, we do not need to check any even numbers in the list.

27 does not divide 423 and hence cannot be the HCF. 18 can be skipped as it is even.

Checking for 9:

9 divides 216,423,1215,1422 and 2169. Hence, it would become the HCF. (Note: we do not need to check 2223 once we know that 2169 is divisible by 9)

Question 2: A nursery has 363,429 and 693 plants respectively of 3 distinct varieties. It is desired to place these plants in straight rows of plants of 1 variety only so that the number of rows required is the minimum. What is the size of each row and how many rows would be required?

The size of each row would be the HCF of 363, 429 and 693. Difference between 363 and 429 =66. Factors of 66 are 66, 33, 22, 11, 6, 3, 2, 1.

66 need not be checked as it is even and 363 is odd. 33 divides 363, hence would automatically divide 429 and also divides 693. Hence, 33 is the correct answer for the size of each row.

For how many rows would be required we need to follow the following process:

Minimum number of rows required = 363/33 + 429/33 + 693/33 = 11 + 13 + 21 = 45 rows.

THE CONCEPT OF LCM (LEAST COMMON MULTIPLE)

Let n1, and n2 be two natural numbers distinct from each other. The smallest natural number n that is exactly divisible by n1 and n2 is called the Least Common Multiple (LCM) of n1 and n2 and is designated as LCM (n1, n2).

Rule for Finding the LCM of two Numbers n1 and n2

(a)Find the standard form of the numbers n1 and n2.

(b)Write out all the prime factors, which are contained in the standard forms of either of the numbers.

(c)Raise each of the prime factors listed above to the highest of the powers in which it appears in the standard forms of the numbers n1 and n2.

(d)The product of results of the previous step will be the LCM of n1 and n2.

Illustration: Find the LCM of 150, 210, 375.

Step 1: Writing down the standard form of numbers

150 = 5 × 5 × 3 × 2

210 = 5 × 2 × 7 × 3

375 = 5 × 5 × 5 × 3

Step 2: Write down all the prime factors: that appear at least once in any of the numbers: 5, 3, 2, 7.

Step 3: Raise each of the prime factors to their highest available power (considering each to the numbers).

The LCM = 21 × 31 × 53 × 71 = 5250.

Important Rule:

GCD (n1, n2). LCM (n1, n2) = n1 n2

i.e. The product of the HCF and the LCM equals the product of the numbers.

SHORT CUT FOR FINDING THE LCM

The LCM (least common multiple) again has a much faster way of doing it than what we learnt in school.

The process has to do with the use of co-prime numbers.

Before we look at the process, let us take a fresh look at what co-prime numbers are:

Co-prime numbers are any two numbers which have an HCF of 1, i.e. when two numbers have no common prime factor apart from the number 1, they are called co-prime or relatively prime to each other.

Some Rules for Co-primes

2 Numbers being co-prime

(i)Two consecutive natural numbers are always co-prime (Example 5, 6; 82, 83; 749, 750 and so on)

(ii)Two consecutive odd numbers are always co-prime (Examples: 7, 9; 51, 53; 513, 515 and so on)

(iii)Two prime numbers are always co-prime (Examples: 13, 17; 53, 71 and so on)

(iv)One prime number and another composite number (such that the composite number is not a multiple of the prime number) are always co-prime (Examples: 17, 38; 23, 49 and so on, but note that 17 and 51 are not co-prime)

3 or more numbers being co-prime with each other means that all possible pairs of the numbers would be co-prime with each other.

Thus, 47, 49, 51 and 52 are co-prime since each of the 6 pairs (47,49); (47,51); (47,52); (49,51); (49,52) and (51,52) are co-prime.

Rules for Spotting three Co-prime Numbers

(i)Three consecutive odd numbers are always co-prime (Examples: 15, 17, 19; 51, 53, 55 and so on)

(ii)Three consecutive natural numbers with the first one being odd (Examples: 15, 16, 17; 21, 22, 23; 41, 42, 43 and so on). Note that 22, 23, 24 are not co-prime

(iii)Two consecutive natural numbers along-with the next odd number such that the first no. is even (examples: 22, 23, 25; 52, 53, 55; 68, 69, 71 and so on)

(iv)Three prime numbers (Examples: 17, 23, 29; 13, 31, 43 and so on)

So what do co-prime numbers have to do with LCMs?

By using the logic of co-prime numbers, you can actually bypass the need to take out the prime factors of the set of numbers for which you are trying to find the LCM. How?

The following process will make it clear:

Let us say that you were trying to find the LCM of 9,10,12 and 15.

The LCM can be directly written as: 9 × 10 × 2. The thinking that gives you the value of the LCM is as follows:

Step 1: If you can see a set of 2 or more co-prime numbers in the set of numbers for which you are finding the LCM- write them down by multiplying them.

So in the above situation, since we can see that 9 and 10 are co-prime to each other we can start off writing the LCM by writing 9 × 10 as the first step.

Step 2: For each of the other numbers, consider what part of them have already been taken into the answer and what part remains outside the answer. In case you see any part of the other numbers such that it is not a part of the value of the LCM you are writing—such a part would need to be taken into the answer of the LCM.

The process will be clear once you see what we do (and how we think) with the remaining 2 numbers in the above problem.

At this point when we have written down 9 × 10 we already have taken into account the numbers 9 and 10 leaving us to account for 12 and 15.

Thought about 12: 12 is 2 × 2 × 3

9 × 10 already has a 3 and one 2 in it prime factors. However, the number 12 has two 2’s. This means that one of the two 2’s of the number 12 is still not accounted for in our answer. Hence, we need to modify the LCM by multiplying the existing 9 × 10 by a 2. With this change the LCM now becomes:

9 × 10 × 2

Thought about 15: 15 is 5 × 3

9 × 10 × 2 already has a 5 and a 3. Hence, there is no need to add anything to the existing answer.

Thus, 9 × 10 × 2 would become the correct answer for the LCM of the numbers 9, 10, 12 and 15.

What if the numbers were: 9, 10, 12 and 25

Step 1: 9 and 10 are co-prime

Hence, the starting value is 9 × 10

Thought about 12: 12 is 2 × 2 × 3

9 × 10 already has a 3 and one 2 in it prime factors. However, the number 12 has two 2’s. This means that one of the two 2’s of the number 12 is still not accounted for in our answer. Hence, we need to modify the LCM by multiplying the existing 9 × 10 by a 2. With this change the LCM now becomes:

9 × 10 × 2

Thought about 25: 25 is 5 × 5

9 × 10 × 2 has only one 5. Hence, we need to add another 5 to the answer.

Thus, 9 × 10 × 2 × 5 would become the correct answer for the LCM of the numbers 9, 10, 12 and 25.

Rule for Finding out HCF and LCM of Fractions

(A)HCF of two or more fractions is given by:

(B)LCM of two or more fractions is given by:

Typical questions on LCMs

You would be able to see most of the standard questions on LCMs in the practice exercise on HCF and LCM given below.

Rules for HCF: If the HCF of x and y is G, then the HCF of

(i)x, (x + y) is also G

(ii)x, (xy) is also G

HCF and LCM

Practice Exercise

(Typical questions asked in Exams)

1.

Find the common factors for the numbers.

(a) 24 and 64

(b) 42, 294 and 882

(c) 60, 120 and 220

2.

Find the HCF of

(a) 420 and 1782

(b) 36 and 48

(c) 54, 72, 198

(d) 62, 186 and 279

3.

Find the LCM of

(a) 13, 23 and 48

(b) 24, 36, 44 and 62

(c) 22, 33, 45, and 72

(d) 13, 17, 21 and 33

4.

Find the series of common multiples of

(a) 54 and 36

(b) 33, 45 and 60

[Hint: Find the LCM and then create an Arithmetic progression with the first term as the LCM and the common difference also as the LCM.]

5.

The LCM of two numbers is 936. If their HCF is 4 and one of the numbers is 72, the other is:

(a) 42

(b) 52

(c) 62

(d) None of these

[Answer: (b). Use HCF × LCM = product of numbers.]

6.

Two alarm clocks ring their alarms at regular intervals of 50 seconds and 48 seconds. If they first beep together at 12 noon, at what time will they beep again for the first time?

(a) 12:10 P.M.

(b) 12:12 P.M.

(c) 12:11 P.M.

(d) None of these

[Answer: (d). The LCM of 50 and 48 being 1200, the two clocks will ring again after 1200 seconds.]

7.

4 Bells toll together at 9:00 A.M. They toll after 7, 8, 11 and 12 seconds respectively. How many times will they toll together again in the next 3 hours?

(a) 3

(b) 4

(c) 5

(d) 6

[Answer: (c). The LCM of 7, 8, 11 and 12 is 1848. Hence, the answer will be got by the quotient of the ratio (10800)/(1848) Æ 5.]

8.

On Ashok Marg three consecutive traffic lights change after 36, 42 and 72 seconds respectively. If the lights are first switched on at 9:00 A.M. sharp, at what time will they change simultaneously?

(a) 9 : 08 : 04

(b) 9 : 08 : 24

(c) 9 : 08 : 44

(d) None of these

[Answer (b). The LCM of 36,42 and 72 is 504. Hence, the lights will change simultaneously after 8 minutes and 24 seconds.]

9.

The HCF of 2472, 1284 and a third number ‘N’ is 12. If their LCM is 23 × 32 × 51 × 103 × 107, then the number ‘N’ is:

(a) 22 × 32 × 71

(b) 22 × 33 × 103

(c) 22 × 32 × 51

(d) None of these

[Answer: (c)]

10.

Two equilateral triangles have the sides of lengths 34 and 85 respectively.

(a) The greatest length of tape that can measure both of them exactly is:

[Answer: HCF of 34 and 85 is 17.]

(b) How many such equal parts can be measured?

[Answer: ]

11.

Two numbers are in the ratio 17:13. If their HCF is 15, what are the numbers?

(Answer: 17 × 15 and 13 × 15 i.e. 255 and 195 respectively.) [Note : This can be done when the numbers are co-prime.]

12.

A forester wants to plant 44 apple trees, 66 banana trees and 110 mango trees in equal rows (in terms of number of trees). Also, he wants to make distinct rows of trees (i.e. only one type of tree in one row). The number of rows (minimum) that are required are:

(a) 2

(b) 3

(c) 10

(d) 11

[Answer: (c) 44/22 + 66/22 + 110/22 (Since 22 is the HCF)]

13.

Three runners running around a circular track can complete one revolution in 2, 4 and 5.5 hours respectively. When will they meet at the starting point?

(a) 22

(b) 33

(c) 11

(d) 44

(The answer will be the LCM of 2, 4 and 11/2. This will give you 44 as the answer).

14.

The HCF and LCM of two numbers are 33 and 264 respectively. When the first number is divided by 2, the quotient is 33. The other number is?

(a) 66

(b) 132

(c) 198

(d) 99

[Answer: 33 × 264 = 66 × n. Hence, n = 132]

15.

The greatest number which will divide: 4003, 4126 and 4249:

(a) 43

(b) 41

(c) 45

(d) None of these

The answer will be the HCF of the three numbers. (1 in this case)

16.

Which of the following represents the largest 4 digit number which can be added to 7249 in order to make the derived number divisible by each of 12, 14, 21, 33, and 54.

(a) 9123

(b) 9383

(c) 8727

(d) None of these

[Answer: The LCM of the numbers 12, 14, 21, 33 and 54 is 8316. Hence, in order for the condition to be satisfied we need to get the number as:

7249 + n = 8316 × 2

Hence, n = 9383.]

17.

Find the greatest number of 5 digits, that will give us a remainder of 5, when divided by 8 and 9 respectively.

(a) 99931

(b) 99941

(c) 99725

(d) None of these

[Answer: The LCM of 8 and 9 is 72. The largest 5 digit multiple of 72 is 99936. Hence, the required answer is 99941.]

18.

The least perfect square number which is divisible by 3, 4, 6, 8, 10 and 11 is:

Solution: The number should have at least one 3, three 2’s, one 5 and one 11 for it to be divisible by 3, 4, 6, 8, 10 and 11.

Further, each of the prime factors should be having an even power in order to be a perfect square. Thus, the correct answer will be: 3 × 3 × 2 × 2 × 2 × 2 × 5 × 5 × 11 × 11

19.

Find the greatest number of four digits which when divided by 10, 11, 15 and 22 leaves 3, 4, 8 and 15 as remainders respectively.

(a) 9907

(b) 9903

(c) 9893

(d) None of these

[Answer: First find the greatest 4 digit multiple of the LCM of 10, 11, 15 and 22. (In this case it is 9900). Then, subtract 7 from it to give the answer.]

20.

Find the HCF of (3125 – 1) and (335 – 1).

[Answer: The solution of this question is based on the rule that:

The HCF of (am – 1) and (an – 1) is given by (aHCF of m, n – 1)

Thus, in this question the answer is: (35 – 1). Since 5 is the HCF of 35 and 125.]

21.

What will be the least possible number of the planks, if three pieces of timber 42 m, 49 m and 63 m long have to be divided into planks of the same length?

(a) 7

(b) 8

(c) 22

(d) None of these

22.

Find the greatest number, which will divide 215, 167 and 135 so as to leave the same remainder in each case.

(a) 64

(b) 32

(c) 24

(d) 16

23.

Find the L.C.M of 2.5, 0.5 and 0.175

(a) 2.5

(b) 5

(c) 7.5

(d) 17.5

24.

The L.C.M of 4.5; 0.009; and 0.18 = ?

(a) 4.5

(b) 45

(c) 0.225

(d) 2.25

25.

The L.C.M of two numbers is 1890 and their H.C.F is 30. If one of them is 270, the other will be

(a) 210

(b) 220

(c) 310

(d) 320

26.

What is the smallest number which when increased by 6 is divisible by 36, 63 and 108?

(a) 750

(b) 752

(c) 754

(d) 756

27.

The smallest square number, which is exactly divisible by 2, 3, 4, – 9, 6, 18, 36 and 60, is

(a) 900

(b) 1600

(c) 3600

(d) None of these

28.

The H.C.F of two numbers is 11, and their L.C.M is 616. If one of the numbers is 88, find the other.

(a) 77

(b) 87

(c) 97

(d) None of these

29.

What is the greatest possible rate at which a man can walk 51 km and 85 km in an exact number of minutes?

(a) 11 km/min

(b) 13 km/min

(c) 17 km/min

(d) None of these

30.

The HCF and LCM of two numbers are 12 and 144 respectively. If one of the numbers is 36, the other number is

(a) 4

(b) 48

(c) 72

(d) 432

ANSWER KEY

21. (c)

22. (d)

23. (d)

24. (a)

25. (a)

26. (a)

27. (a)

28. (a)

29. (c)

30. (b)

Solutions

4.

(a)

The first common multiple is also the LCM. The LCM of 36 and 54 would be 108. The next common multiple would be 216, 324 and so on. Thus, the required series would be 108, 216, 324, 432, 540, 648….

(b)

The LCM of 33, 45 and 60 = 60 × 3 × 11 = 1980. Thus, the required series is: 1980, 3960, 5940…

5.

LCM × HCF = 936 × 4 = N1 × N2 Æ

936 × 4 = 72 × N2 Æ N2 = 13 × 4 = 52. Option (b) is correct.

6.

The first bell toll will be after the time elapsed is the LCM of 50 and 48. The LCM of 50 and 48 is 50 × 24 = 1200. Hence, the first time they would toll together after 12 noon would be exactly 1200 seconds or 20 minutes later. Option (d) is correct.

7.

The LCM of 7, 8, 11 and 12 is given by 12 × 11 × 2 × 7 = 264 × 7 = 1848. 1848 seconds is 30 minutes 48 seconds. Hence, the 4 bells would toll together every 30 minutes 48 seconds.

The number of times they would toll together in the next 3 hours would be given by the quotient of the division:

3 × 60 × 60 /1848 Æ 5 times

Alternately, by thinking of 1848 seconds as 30 minutes 48 seconds you can also solve the same question by thinking as follows:

9.

2472 = 23 × 103 × 3; 1284 = 22 × 107 × 3. Since the HCF is 12, the number must have a component of 22 × 31 at the very least in it. Also, since the LCM is 23 × 32 × 51 × 103 × 107 we can see that the minimum requirement in the required number has to be 32 × 51. Combining these two requirements we get that the number should have 22 × 32 × 51 at the minimum and the power of 2 could also be 23 while we could also have either one of 1031 and/or 1071 as a part of the required number.

Thus, for instance, the number could also be 23 × 32 × 51 × 1031 × 1071. The question has asked us- what ‘could’ the number be?

Option (c) gives us a possible value of the number and is hence the correct answer.

21.

The least possible number of planks would occur when we divide each plank into a length equal to the HCF of 42, 49 and 63. The HCF of these numbers is clearly 7- and this should be the size of each plank. Number of planks in this case would be: 42/7 + 49/7 + 63/7 = 6 + 7 + 9 = 22 planks. Hence, option (c) is correct.

22.

Trial and error would give us that the number 16 would leave the same remainder of 7 in all the three cases. Hence, option (d) is correct.

23.

The numbers are 5/2, ½ and 175/1000 = 7/40. The LCM of three fractions is given by the formula:

LCM of numerators/HCF of denominators = (LCM of 5, 1 and 7)/(HCF of 2 and 40) = 35/2 = 17.5

24.

Use the same process as for question 23 for the numbers: 9/2; 9/1000 and 9/50.

(LCM of 9, 9, 9)/ (HCF of 2, 100, 50) = 9/2 = 4.5

25.

1890 × 30 = 270 × N2 Æ N2 = 210. Hence, option (a) is correct.

26.

The LCM of 36, 63 and 108 is 756. Hence, the required number is 750. Option (a) is correct.

27.

The LCM of the given numbers is 180. Hence, all multiples of 180 would be divisible by all of these numbers. Checking the series 180, 360, 540, 720, 900 we can see that 900 is the first perfect square in the list. Option (a) is correct.

28.

Using the property HCF × LCM = product of the numbers, we get:

616 × 11 = 88 × N2 Æ N2 = 77. Option (a) is correct.

29.

The answer would be given by the HCF of 51 and 85 – which is 17. Hence, option (c) is correct.

30.

Using the property HCF × LCM = product of the numbers, we get:

12 × 144 = 36 × N2 Æ N2 = 48. Option (b) is correct.

DIVISIBILITY

A number x is said to be divisible by another number ‘y’ if it is completely divisible by Y (i.e. it should leave no remainder).

In general it can be said that any integer I, when divided by a natural number N, there exist a unique pair of numbers Q and R which are called the quotient and Remainder respectively.

Thus, I = QN + R.

For any integer I and any natural number n there is a unique pair of numbers a and b such that:

I = QN + R

Where Q is an integer and N is a natural number or zero and 0 ≥ R < N (i.e. remainder has to be a whole number less than N.)

If the remainder is zero we say that the number I is divisible by N.

When R π 0, we say that the number I is divisible by N with a remainder.

Thus, 25/8 can be written as: 25 = 3 × 8 + 1 (3 is the quotient and 1 is the remainder)

While, –25/7 will be written as –25 = 7 × (–4) + 3 (–4 is the quotient and 3 is the remainder)

Note: An integer b π 0 is said to divide an integer a if there exists another integer c such that:

a = bc

It is important to explain at this point a couple of concepts with respect to the situation, when we divide a negative number by a natural number N.

Suppose, we divide – 32 by 7. Contrary to what you might expect, the remainder in this case is + 3 (and not – 4). This is because the remainder is always non negative.

Thus, –32/7 gives quotient as – 5 and remainder as + 3.

The relationship between the remainder and the decimal:

1.Suppose we divide 42 by 5. The result has a quotient of 8 and remainder of 2.

But 42/5 = 8.4. As you can see, the answer has an integer part and a decimal part. The integer part being 8 (equals the quotient), the decimal part is 0.4 (and is given by 2/5).

Since, we have also seen that for any divisor N, the set of remainders obeys the inequality 0 £ R < N, we should realise that any divisor N, will yield exactly N possible remainders. (For example If the divisor is 3, we have 3 possible remainders 0, 1 and 2. Further, when 3 is the divisor we can have only 3 possible decimal values .00, .333 & 0.666 corresponding to remainders of 0, 1 or 2. I would want you to remember this concept when you study the fraction to percentage conversion table in the chapter on percentages.

2.In the case of –42 being divided by 5, the value is – 8.4. In this case the interpretation should be thus:

The integer part is – 9 (which is also the quotient of this division) and the decimal part is 0.6 (corresponding to 3/5) Notice that since the remainder cannot be negative, the decimal too cannot be negative.

Theorems of Divisibility

(a)If a is divisible by b then ac is also divisible by b.

(b)If a is divisible by b and b is divisible by c then a is divisible by c.

(c)If a and b are natural numbers such that a is divisible by b and b is divisible by a then a = b.

(d)If n is divisible by d and m is divisible by d then (m + n) and (m n) are both divisible by d. This has an important implication. Suppose 28 and 742 are both divisible by 7. Then (742 + 28) as well as (742 – 28) are divisible by 7. (and in fact so is + 28 – 742).

(e)If a is divisible by b and c is divisible by d then ac is divisible by bd.

(f)The highest power of a prime number p, which divides n! exactly is given by

[n/p] + [n/p2] + [n/p3] +…

where [x] denotes the greatest integer less than or equal to x.

As we have already seen earlier –

Any composite number can be written down as a product of its prime factors. (Also called standard form)

Thus, for example the number 1240 can be written as 23 × 311 × 51.

The standard form of any number has a huge amount of information stored in it. The best way to understand the information stored in the standard form of a number is to look at concrete examples. As a reader I want you to understand each of the processes defined below and use them to solve similar questions given in the exercise that follows and beyond:

1.Using the standard form of a number to find the sum and the number of factors of the number:

(a)Sum of factors of a number:

Suppose, we have to find the sum of factors and the number of factors of 240.

240 = 24 × 31 × 51

The sum of factors will be given by:

(20 + 21 + 22 + 23 + 24) (30 + 31) (50 + 51)

= 31 × 4 × 6 = 744

Note: This is a standard process, wherein you create the same number of brackets as the number of distinct prime factors the number contains and then each bracket is filled with the sum of all the powers of the respective prime number starting from 0 to the highest power of that prime number contained in the standard form.

Thus, for 240, we create 3 brackets—one each for 2, 3 and 5. Further in the bracket corresponding to 2 we write (20 + 21 + 22 + 23 + 24).

Hence, for example for the number 40 = 23 × 51, the sum of factors will be given by: (20 + 21 + 22 + 23) (50 + 51) {2 brackets since 40 has 2 distinct prime factors 2 and 5}

(b)Number of factors of the number:

Let us explore the sum of factors of 40 in a different context.

(20 + 21 + 22 + 23) (50 + 51)

= 20 × 50 + 20 × 51 + 21 × 50 + 21 × 51 + 22 × 50 + 22

× 51 + 23 × 50 + 23 × 51

= 1 + 5 + 2 + 10 + 4 + 20 + 8 + 40 = 90

A clear look at the numbers above will make you realize that it is nothing but the addition of the factors of 40

Hence, we realise that the number of terms in the expansion of (20 + 21 + 22 + 23) (50 + 51) will give us the number of factors of 40. Hence, 40 has 4 × 2 = 8 factors.

Note: The moment you realise that 40 = 23 × 51 the answer for the number of factors can be got by (3 + 1) (1 + 1) = 8

2.Sum and Number of even and odd factors of a number.

Suppose, you are trying to find out the number of factors of a number represented in the standard form by: 23 × 34 × 52 × 7 3

As you are already aware the answer to the question is (3 + 1) (4 + 1) (2 + 1) (3 + 1) and is based on the logic that the number of terms will be the same as the number of terms in the expansion: (20 + 21 + 22 + 23) (30 + 31 + 32 + 33 + 34)(50 + 51 + 52) (70 + 71 + 72 + 73).

Now, suppose you have to find out the sum of the even factors of this number. The only change you need to do in this respect will be evident below. The answer will be given by:

(21 + 22 + 23)(30+ 31 + 32 + 33 + 34) (50 + 51 + 52) (70 + 71 + 72 + 73)

Note: We have eliminated 20 from the original answer. By eliminating 20 from the expression for the sum of all factors you are ensuring that you have only even numbers in the expansion of the expression.

Consequently, the number of even factors will be given by: (3) (4 + 1) (2 + 1) (3 + 1)

i.e. Since 20 is eliminated, we do not add 1 in the bracket corresponding to 2.

Let us now try to expand our thinking to try to think about the number of odd factors for a number.

In this case, we just have to do the opposite of what we did for even numbers. The following step will make it clear:

Odd factors of the number whose standard form is : 23 × 34 × 52 × 73

Sum of odd factors = (20) (30 + 31 + 32 + 33 + 34) (50 + 51 + 52) (70 + 71 + 72 + 73)

i.e.: Ignore all powers of 2. The result of the expansion of the above expression will be the complete set of odd factors of the number. Consequently, the number of odd factors for the number will be given by the number of terms in the expansion of the above expression.

Thus, the number of odd factors for the number 23 × 34 × 52 × 73 = 1 × (4 + 1) (2 + 1) (3 + 1).

3.Sum and number of factors satisfying other conditions for any composite number

These are best explained through examples:

(i)Find the sum and the number of factors of 1200 such that the factors are divisible by 15.

Solution : 1200 = 24 × 52 × 31.

For a factor to be divisible by 15 it should compulsorily have 31 and 51 in it. Thus, sum of factors divisible by 15 = (20 + 21 + 22 + 23 + 24) × (51 + 52) (31) and consequently the number of factors will be given by 5 × 2 × 1 = 10.

(What we have done is ensure that in every individual term of the expansion, there is a minimum of 31 × 51. This is done by removing powers of 3 and 5 which are below 1.)

Task for the student: Physically verify the answers to the question above and try to convert the logic into a mental algorithm.

NOTE FROM THE AUTHOR—The need for thought algorithms:

I have often observed that the key difference between understanding a concept and actually applying it under examination pressure, is the presence or absence of a mental thought algorithm which clarifies the concept to you in your mind. The thought algorithm is a personal representation of a concept—and any concept that you read/understand in this book (or elsewhere) will remain an external concept till it remains in someone else’s words. The moment the thought becomes internalised the concept becomes yours to apply and use.

Practice Exercise on Factors

For the number 2450 find.

1.

The sum and number of all factors.

2.

The sum and number of even factors.

3.

The sum and number of odd factors.

4.

The sum and number of factors divisible by 5

5.

The sum and number of factors divisible by 35.

6.

The sum and number of factors divisible by 245.

For the number 7200 find.

7.

The sum and number of all factors.

8.

The sum and number of even factors.

9.

The sum and number of odd factors.

10.

The sum and number of factors divisible by 25.

11.

The sum and number of factors divisible by 40.

12.

The sum and number of factors divisible by 150.

13.

The sum and number of factors not divisible by 75.

14.

The sum and number of factors not divisible by 24.

15.

Find the number of divisors of 1728.

(a) 18

(b) 30

(c) 28

(d) 20

16.

Find the number of divisors of 1080 excluding the divisors, which are perfect squares.

(a) 28

(b) 29

(c) 30

(d) 31

17.

Find the number of divisors of 544 excluding 1 and 544.

(a) 12

(b) 18

(c) 11

(d) 10

18.

Find the number of divisors 544 which are greater than 3.

(a) 15

(b) 10

(c) 12

(d) None of these.

19.

Find the sum of divisors of 544 excluding 1 and 544.

(a) 1089

(b) 545

(c) 589

(d) 1134

20.

Find the sum of divisors of 544 which are perfect squares.

(a) 32

(b) 64

(c) 42

(d) 21

21.

Find the sum of odd divisors of 544.

(a) 18

(b) 34

(c) 68

(d) 36

22.

Find the sum of even divisors of 4096.

(a) 8192

(b) 6144

(c) 8190

(d) 6142

23.

Find the sum the sums of divisors of 144 and 160.

(a) 589

(b) 781

(c) 735

(d) None of these

24.

Find the sum of the sum of even divisors of 96 and the sum of odd divisors of 3600.

(a) 639

(b) 735

(c) 651

(d) 589

ANSWER KEY

15. (c)

16. (a)

17. (d)

18. (b)

19. (c)

20. (d)

21. (a)

22. (c)

23. (b)

24. (c)

Solutions

Solutions to Questions 1 to 6:

2450 = 50 × 49 = 21 × 52 × 72

1.Sum and number of all factors:

Sum of factors = (20 + 21) (50 + 51 + 52) (70 + 71 + 72)

Number of factors = 2 × 3 × 3 = 18

2.Sum of all even factors:

(21) (50 + 51 + 52) (70 + 71 + 72)

Number of even factors = 1 × 3 × 3 = 9

3.Sum of all odd factors:

(20) (50 + 51 + 52) (70 + 71 + 72)

Number of odd factors = 1 × 3 × 3 = 9

4.Sum of factors divisible by 5:

(20 + 21) (51 + 52) (70 + 71 + 72)

Number of factors divisible by 5 = 2 × 2 × 3 = 12

5.Sum of factors divisible by 35:

(20 + 21) (51 + 52) (71 + 72)

Number of factors divisible by 35 = 2 × 2 × 2 = 8

6.Sum of all factors divisible by 245:

(20 + 21) (51 + 52) (72)

Number of factors divisible by 245 = 2 × 2 × 1 = 4

Solutions to Questions 7 to 14:

7200 = 72 × 100 = 12 × 6 × 100 = 25 × 32 × 52

7.Sum and number of all factors:

Sum of factors = (20 + 21+ 22 + 23 + 24 + 25) (30 + 31 + 32) (50 + 51 + 52)

Number of factors = 6 × 3 × 3 = 54

8.Sum and number of even factors:

Sum of even factors = (21 + 22 + 23 + 24 + 25) (30 + 31 + 32) (50 + 51 + 52)

Number of even factors = 5 × 3 × 3 = 45

9.Sum and number of odd factors:

Sum of odd factors = (20) (30 + 31 + 32) (50 + 51 + 52)

Number of odd factors = 1 × 3 × 3 = 9

10.Sum and number of factors divisible by 25:

Sum of factors divisible by 25= (20 + 21 + 22 + 23 + 24 + 25) (30 + 31 + 32) (52)

Number of factors divisible by 25 = 6 × 3 × 1 = 18

11.Sum and number of factors divisible by 40:

Sum of factors divisible by 40 = (23 + 24 + 25) (30 + 31 + 32) (51 + 52)

Number of factors = 3 × 3 × 2 = 18

12.Sum and number of factors divisible by 150:

Sum of factors divisible by 150 = (21 + 22 + 23 + 24 + 25) (31 + 32) (52)

Number of factors divisible by 150 = 5 × 2 × 1 = 10

13.Sum and number of factors not divisible by 75:

Sum of factors not divisible by 75 = Sum of all factors – Sum of factors divisible by 75 =

(20 + 21 + 22 + 23 + 24 + 25) (30 + 31 + 32) (50 + 51 + 52) – (20 + 21 + 22 + 23 + 24 + 25) (31 + 32) (52)

Number of factors not divisible by 75 = Number of factors of 7200 – Number of factors of 7200 which are divisible by 75 = 6 × 3 × 3 – 6 × 2 × 1 = 54 – 12 = 42

14.Sum and number of factors not divisible by 24:

Sum of factors not divisible by 24 = Sum of all factors – Sum of factors divisible by 24 =

(20 + 21 + 22 + 23 + 24 + 25) (30 + 31 + 32) (50 + 51 + 52) – (23 + 24 + 25) (31 + 32) (50 + 51 + 52)

Number of factors not divisible by 24 = Number of factors of 7200 – Number of factors of 7200 which are divisible by 24 = 6 × 3 × 3 – 3 × 2 × 3 = 54 – 18 = 36

15.Number of divisors of 1728

1728 = 4 × 432 = 16 × 108 = 64 × 27 = 26 × 33

Number of factors = 7 × 4 = 28. Option (a) is correct.

16.1080 = 108 × 10 = 27 × 4 × 10 = 33 × 23 × 51

Number of factors = 4 × 4 × 2 = 32.

In order to see the number of factors of 1080 which are perfect squares we need to visualize the structure for writing down the sum of perfect square factors of 1080.

This would be given by:

Sum of all perfect square factors of 1080 = (20 + 22) (30 + 32) (50).

From the above structure it is clear that the number of perfect square factors is going to be 2 × 2 × 1 = 4

Thus, the number of factors of 1080 which are not perfect squares are equal to 32 – 4 = 28.

Option (a) is correct.

17.544 = 171 × 25. Hence, the total number of factors of 544 is 2 × 6 =12. But we have to count factors excluding 1 and 544. Thus, we need to remove 2 factors from this. The required answer is 12 – 2 =10. Option (d) is correct.

18.Using the fact that 544 has a total of 12 factors and the numbers 1 and 2 are the two factors which are lower than 3, we would get a total of 10 factors greater than 3. Option (b) is correct.

19.The required answer would be given by: Sum of all factors of 544 – 1 – 544 = (20 + 21+ 22 + 23 + 24 + 25) (170 + 171) – 545 = 63 × 18 – 545 = 589. Option (c) is correct.

20.Sum of divisors of 544 which are perfect square is:

(20 + 22 + 24 ) (170) = 21. Option (d) is correct.

21.Sum of odd divisors of 544 =

(20) (170 + 171) = 18. Option (a) is correct.

22.4096 = 212.

Sum of even divisors = (21 + 22 + 23 + 24 + 25 + 26 + 27 + 28 + 29 + 210 + 211 + 212) = 213 – 2 = 8190

23.144 = 24 × 32 Æ Sum of divisors of 144 = (20 + 21 + 22 + 23 + 24 ) (30 + 31 + 32) = 31 × 13 = 403

160 = 25 × 51 Æ Sum of divisors of 160 = (20 + 21 + 22 + 23 + 24 + 25) (50 + 51) = 63 × 6 = 378.

Sum of the two = 403 + 378 = 781.

24.96 = 25 × 31. Sum of even divisors of 96 = (21 + 22 + 23 + 24 + 25) (30 + 31) = 62 × 4 = 248

3600 = 24 × 52 × 32. Sum of odd divisors of 3600 = (20) (30 + 31 + 32) (50 + 51 + 52) = 13 × 31 = 403

Sum of the two = 248 + 403 = 651.

Option (c) is correct.

NUMBER OF ZEROES IN AN EXPRESSION

Suppose you have to find the number of zeroes in a product: 24 × 32 × 17 × 23 × 19 = (23 × 31) × (25) × 171 × 23 × 19.

As you can notice, this product will have no zeroes because it has no 5 in it.

However, if you have an expression like: 8 × 15 × 23 × 17 × 25 × 22

The above expression can be rewritten in the standard form as:

23 × 31 × 51 × 23 × 17 × 52 × 21 × 111

Zeroes are formed by a combination of 2 × 5. Hence, the number of zeroes will depend on the number of pairs of 2’s and 5’s that can be formed.

In the above product, there are four twos and three fives. Hence, we shall be able to form only three pairs of (2 × 5). Hence, there will be 3 zeroes in the product.

Finding the Number of Zeroes in a Factorial Value

Suppose you had to find the number of zeroes in 6!.

6! = 6 × 5 × 4 × 3 × 2 × 1 = (3 × 2) × (5) × (2 × 2) × (3) × (2) × (1).

The above expression will have only one pair of 5 × 2, since there is only one 5 and an abundance of 2’s.

It is clear that in any factorial value, the number of 5’s will always be lesser than the number of 2’s. Hence, all we need to do is to count the number of 5’s. The process for this is explained in Solved Examples 1.1 to 1.3.

Exercise for Self-practice

Find the number of zeroes in the following cases:

1.47!

2.58!

3.13 × 15 × 22 × 125 × 44 × 35 × 11

4.12 × 15 × 5 × 24 × 13 × 17

5.173!

6.144! × 5 × 15 × 22 × 11 × 44 × 135

7.148!

8.1093!

9.1132!

10.1142! × 348! × 17!

Solutions

1.47/5 Æ Quotient 9. 9/5 Æ Quotient Æ 1. 9 + 1 = 10 zeroes.

2.58/5 Æ Quotient 11. 11/5 Æ Quotient Æ 2. 11 + 2 = 13 zeroes.

3.The given expression has five 5’s and three 2’s. Thus, there would be three zeroes in the expression.

4.The given expression has two 5’s and five 2’s. Thus, there would be two zeroes in the expression.

5.173/5 Æ Quotient 34. 34/5 Æ Quotient 6. 6/5 Æ Quotient 1. 34 + 6 + 1 = 41 zeroes.

6.144! Would have 28 + 5 + 1 = 34 zeroes and the remaining part of the expression would have three zeroes. A total of 34 + 3 = 37 zeroes.

7.148/5 Æ Quotient 29. 29/5 Æ Quotient 5. 5/5 Æ Quotient 1. 29 + 5 + 1 = 35 zeroes.

8.1093/5 Æ Quotient 218. 218/5 Æ Quotient 43. 43/5 Æ Quotient 8. 8/5 Æ Quotient 1. 218 + 43 + 8 + 1 = 270 zeroes.

9.1132/5 Æ Quotient 226. 226/5 Æ Quotient 45. 45/5 Æ Quotient 9. 9/5 Æ Quotient 1. 226 + 45 + 9 + 1 = 281 zeroes.

10.1142/5 Æ Quotient 228. 228/5 Æ Quotient 45. 45/5 Æ Quotient 9. 9/5 Æ Quotient 1. 228 + 45 + 9 + 1 = 284 zeroes.

348/5 Æ Quotient 69. 69/5 Æ Quotient 13. 13/5 Æ Quotient 2. 69 + 13 + 2 = 84 zeroes.

17/5 Æ Quotient 3 Æ 3 zeroes.

Thus, the total number of zeroes in the expression is: 284 + 84 + 3 = 371 zeroes.

A special implication: Suppose you were to find the number of zeroes in the value of the following factorial values:

45!, 46!, 47!, 48!, 49!

What do you notice? The number of zeroes in each of the cases will be equal to 10. Why does this happen? It is not difficult to understand that the number of fives in any of these factorials is equal to 10. The number of zeroes will only change at 50! (It will become 12).

In fact, this will be true for all factorial values between two consecutive products of 5.

Thus, 50!, 51!, 52!, 53! And 54! will have 12 zeroes (since they all have 12 fives).

Similarly, 55!, 56!, 57!, 58! And 59! will each have 13 zeroes.

Apart from this fact, did you notice another thing? That while there are 10 zeroes in 49! there are directly 12 zeroes in 50!. This means that there is no value of a factorial which will give 11 zeroes. This occurs because to get 50! we multiply the value of 49! by 50. When you do so, the result is that we introduce two 5’s in the product. Hence, the number of zeroes jumps by two (since we never had any paucity of twos.)

Note: at 124! you will get 24 + 4 fi 28 zeroes.

At 125! you will get 25 + 5 + 1 = 31 zeroes. (A jump of 3 zeroes.)

Exercise for Self-practice

1.n! has 23 zeroes. What is the maximum possible value of n?

2.n! has 13 zeroes. The highest and least values of n are?

3.Find the number of zeroes in the product 11 × 22 × 33 × 44 × 55 × 66 × …. × 4949

4.Find the number of zeroes in:

1001 × 992 × 983 × 974 × ………. × 1100

5.Find the number of zeroes in:

11! × 22! × 33! × 44! × 55! × ……..1010!

6.Find the number of zeroes in the value of:

22 × 54 × 46 × 108 × 610 × 1512 × 814 × 2016 × 1018 × 2520

7.What is the number of zeroes in the following:

(a)3200 + 1000 + 40000 + 32000 + 15000000

(b)3200 × 1000 × 40000 × 32000 × 16000000

Solutions

1.This can never happen because at 99! number of zeroes is 22 and at 100! the number of zeroes is 24.

2.59 and 55 respectively.

3.The fives will be less than the twos. Hence, we need to count only the fives.

Thus : 55 × 1010 × 1515 × 2020 × 2525 × 3030 × 3535 × 4040 × 4545

gives us: 5 + 10 + 15 + 20 + 25 + 25 + 30 + 35 + 40 + 45 fives. Thus, the product has 250 zeroes.

4.Again the key here is to count the number of fives. This can get done by:

1001 × 956 × 9011 × 8516 × 8021 × 7526 × …… 596

(1 + 6 + 11 + 16 + 21 + 26 + 31 + 36 + 41 + 46 + …….. + 96) + (1 + 26 + 51 + 76)

= 20 × 48.5 + 4 × 38.5 (Using sum of A.P. ex-plained in the next chapter.)

= 970 + 154 = 1124.

5.The answer will be the number of 5’s. Hence, it will be 5! + 10!

6.The number of fives is again lesser than the number of twos.

The number of 5’s will be given by the power of 5 in the product:

54 × 108 × 1512 × 2016 × 1018 × 2520

= 4 + 8 + 12 + 16 + 18 + 40 = 98.

7.A. The number of zeroes in the sum will be two, since:

3200

1000

40000

32000

15076200

15152400

Thus, in such cases the number of zeroes will be the least number of zeroes amongst the numbers.

Exception: 3200 + 1800 = 5000 (three zeroes, not two).

  1. The number of zeroes will be:

2 + 3 + 4 + 3 + 6 = 18.

An extension of the process for finding the number of zeroes.

Consider the following questions:

1.Find the highest power of 5 which is contained in the value of 127!

2.When 127! is divided by 5n the result is an integer. Find the highest possible value for n.

3.Find the number of zeroes in 127!

In each of the above cases, the value of the answer will be given by:

[127/5] + [127/25] + [127/125]

= 25 + 5 + 1 = 31

This process can be extended to questions related to other prime numbers. For example:

Find the highest power of:

1.3 which completely divides 38!

Solution: [38/3] + [38/32] + 38/33] = 12 + 4 + 1 = 17

2.7 which is contained in 57!

[57/7] + [57/72] = 8 + 1 = 9.

This process changes when the divisor is not a prime number. You are first advised to go through worked out problems 1.4, 1.5, 1.6 and 1.19.

Now try to solve the following exercise:

1.Find the highest power of 7 which divides 81!

2.Find the highest power of 42 which divides 122!

3.Find the highest power of 84 which divides 342!

4.Find the highest power of 175 which divides 344!

5.Find the highest power of 360 which divides 520!

Solutions

1.81/7 Æ Quotient 11. 11/7 Æ Quotient 1. Highest power of 7 in 81! = 11 + 1 =12.

2.In order to check for the highest power of 42, we need to realize that 42 is 2 × 3 × 7. In 122! The least power between 2,3 and 7 would obviously be for 7. Thus, we need to find the number of 7’s in 122! (or in other words- the highest power of 7 in 122!).

This can be done by:

122/7 Æ Quotient 17. 17/7 Æ Quotient 2. Highest power of 7 in 122! = 17 + 2 = 19.

3.84 = 2 × 2 × 3 × 7. This means we need to think of which amongst 22, 3 and 7 would appear the least number of times in 342! It is evident that there would be more 22s and 3’s than 7’s in any factorial value (Because if you write any factorial 1 × 2 × 3 × 4 × 5 × 6 × 7 × 8 × 9 × 10 × 11 × 12 × 13 × 14 × 15…you can clearly see that before a 7 or it’s multiple appears in the multiplication, there are at least two 2’s and one 3 which appear beforehand.)

Hence, in order to solve this question we just need to find the power of 7 in 342!

This can be done as:

342/7 Æ Quotient 48. 48/7 Æ Quotient 6. 6/7 Æ Quotient 0. Highest power of 7 in 342! = 48 + 6 = 54.

4.175 = 5 × 5 × 7. This means we need to think of which amongst 52 and 7 would appear the least number of times in 175! In this case it is not immediately evident that whether there would be more 52s or more 7’s in any factorial value (Because if you write any factorial 1 × 2 × 3 × 4 × 5 × 6 × 7 × 8 × 9 × 10 × 11 × 12 × 13 × 14 × 15…you can clearly see that although the 5’s appear more frequently than the 7’s it is not evident that we would have at least two fives before the 7 appears.) Hence, in this question we would need to check for both the number of 52s and the number of 7’s.

Number of 7’s in 344!

344/7 Æ Quotient 49. 49/7 Æ Quotient 7. 7/7 Æ Quotient 1. Highest power of 7 in 344! = 49 + 7 + 1 = 57.

In order to find the number of 52s in 344! We first need to find the number of 5’s in 344!

344/5 Æ Quotient 68. 68/5 Æ Quotient 13. 13/5 Æ Quotient 2. Number of 5’s in 344! = 68 + 13 + 2 = 83.

83 fives would obviously mean [83/2] = 41 52s

Thus, there are 41 52s and 57 7’s in 344!

Since, the number of 52s are lower, they would determine the highest power of 175 that would divide 344!

The answer is 41.

5.360 = 5 × 2 × 2 × 2 × 3 × 3. This means we need to think of which amongst 23, 32 and 5 would appear the least number of times in 520! In this case it is not immediately evident which of these three would appear least number of times. Hence, in this question we would need to check for all three – 23s 32s and 5s.

Number of 5’s in 520!

520/5 Æ Quotient 104. 104/5 Æ Quotient 20. 20/5 Æ Quotient 4. Highest power of 5 in 520! = 104 + 20 + 4 = 128.

In order to find the number of 32s in 520! we first need to find the number of 3’s in 520!

520/3 Æ Quotient 173. 173/3 Æ Quotient 57. 57/3 Æ Quotient 19. 19/3 Æ Quotient 6. 6/3 Æ Quotient 2. 2/3 Æ Quotient 0. Number of 3’s in 520! = 173 + 57 + 19 + 6 + 2 = 257.

257 threes would obviously mean [257/2]= 128 32s.

In order to find the number of 23s in 520! we first need to find the number of 2’s in 520!

520/2 Æ Quotient 260. 260/2 Æ Quotient 130. 130/2 Æ Quotient 65. 65/2 Æ Quotient 32. 32/2 Æ Quotient 16. 16/2 Æ Quotient 8. 8/2 Æ Quotient 4. 4/2 Æ Quotient 2. 2/2 Æ Quotient 1. 1/2 Æ Quotient 0.

Number of 2’s in 520! = 260 + 130 +65 + 32 +16 + 8 + 4 + 2 + 1 = 518. 518 twos would obviously mean [518/3]= 172 23s.

Thus, there are 128 32s, 128 5’s and 172 23’s in 520!

The highest power of 360 that would divide 520! would be the least of 128, 128 and 172.

The answer is 128.

Exercise for Self-practice

1.

Find the maximum value of n such that 157! is perfectly divisible by 10n.

(a) 37

(b) 38

(c) 16

(d) – 1.15

2.

Find the maximum value of n such that 157! is perfectly divisible by 12n.

(a) 77

(b) 76

(c) 75

(d) 78

3.

Find the maximum value of n such that 157! is perfectly divisible by 18n.

(a) 37

(b) 38

(c) 39

(d) 40

4.

Find the maximum value of n such that 50! is perfectly divisible by 2520n.

(a) 6

(b) 8

(c) 7

(d) None of these

5.

Find the maximum value of n such that 50! is perfectly divisible by 12600n.

(a) 7

(b) 6

(c) 8

(d) None of these

6.

Find the maximum value of n such that 77! is perfectly divisible by 720n.

(a) 35

(b) 18

(c) 17

(d) 36

7.

Find the maximum value of n such that

42 × 57 × 92 × 91 × 52 × 62 × 63 × 64 × 65 × 66 × 67 is perfectly divisible by 42n.

(a) 4

(b) 3

(c) 5

(d) 6

8.

Find the maximum value of n such that

570 × 60 × 30 × 90 × 100 × 500 × 700 × 343 × 720 × 81 is perfectly divisible by 30n.

(a) 12

(b) 11

(c) 14

(d) 13

9.

Find the maximum value of n such that

77 × 42 × 37 × 57 × 30 × 90 × 70 × 2400 × 2402 × 243 × 343 is perfectly divisible by 21n.

(a) 9

(b) 11

(c) 10

(d) 6

Find the number of consecutive zeroes at the end of the following numbers.

10.

72!

(a) 17

(b) 9

(c) 8

(d) 16

11.

77! × 42!

(a) 24

(b) 9

(c) 27

(d) 18

12.

100! + 200!

(a) 73

(b) 24

(c) 11

(d) 22

13.

57 × 60 × 30 × 15625 × 4096 × 625 × 875 × 975

(a) 6

(b) 16

(c) 17

(d) 15

14.

1! × 2! × 3! × 4! × 5! × - - - - - - - × 50!

(a) 235

(b) 12

(c) 262

(d) 105

15.

11 × 22 × 33 × 44 × 55 × 66 × 77 × 88 × 99 × 1010.

(a) 25

(b) 15

(c) 10

(d) 20

16.

100! × 200!

(a) 49

(b) 73

(c) 132

(d) 33

ANSWER KEY

1. (b)

2. (c)

3. (a)

4. (b)

5. (b)

6. (c)

7. (b)

8. (b)

9. (d)

10. (d)

11. (c)

12. (b)

13. (d)

14. (c)

15. (b)

16. (b)

Solutions

1.[157/5] = 31. [31/5] = 6. [6/5] = 1. 31 + 6 + 1 = 38. Option (b) is correct.

2.No of 2’s in 157! = [157/2] + [157/4] + [157/8]… + [157/128] = 78 + 39 + 19 + 9 + 4 + 2 + 1 = 152. Hence, the number of 22s would be [152/2] = 76.

Number of 3’s in 157! = 52 + 17 + 5 +1 = 75.

The answer would be given by the lower of these values. Hence, 75 (Option c) is correct.

3.From the above solution:

Number of 2’s in 157! = 152

Number of 32s in 157! = [75/2]=37.

Hence, option (a) is correct.

4.2520 = 7 × 32 × 23 × 5.

The value of n would be given by the value of the number of 7s in 50!

This value is equal to [50/7] + [50/49] = 7 + 1 = 8

Option (b) is correct.

5.12600 = 7 × 32 × 23 × 52

The value of ‘n’ would depend on which of number of 7s and number of 52s is lower in 50!.

Number of 7’s in 50! = 8. Note here that if we check for 7’s we do not need to check for 32s as there would be at least two 3’s before a 7 comes in every factorial’s value. Similarly, there would always be at least three 2’s before a 7 comes in any factorial’s value. Thus, the number of 32s and the number of 23s can never be lower than the number of 7s in any factorial’s value.

Number of 5s in 50! = 10 + 2 =12. Hence, the number of 52s in 50! = [12/2] = 6.

6 will be the answer as the number of 52s is lower than the number of 7’s.

Option (b) is correct.

6.720 = 24 × 51 × 32

In 77! there would be 38 + 19 + 9 + 4 + 2 + 1 = 73 twos Æ hence [73/4] = 18 24s

In 77! there would be 25 + 8 + 2 = 35 threes Æ hence [35/2] = 17 32s

In 77! there would be 15 + 3 = 18 fives

Since 17 is the least of these values, option (c) is correct.

7.In the expression given, there are three 7’s and more than three 2’s and 3’s. Thus, Option (b) is correct.

8.Checking for the number of 2’s, 3’s and 5’s in the given expression you can see that the minimum is for the number of 3’s (there are 11 of them while there are 12 5’s and more than 11 2’s) Hence, option (b) is correct.

9.The number of 7’s in the number is 6, while there are six 3’s too. Option (d) is correct.

10.The number of zeroes would depend on the number of 5’s in the value of the factorial.

72! Æ 14 + 2 = 16. Option (d) is correct.

11.The number of zeroes would depend on the number of 5’s in the value of the factorial.

77! × 42! Æ 15 + 3 = 18 (for 77!) and 8 + 1 = 9 (for 42!).

Thus, the total number of zeroes in the given expression would be 18 + 9 = 27. Option (c) is correct.

12.The number of zeroes would depend on the number of 5’s in the value of the factorial.

100! would end in 20 + 4 = 24 zeroes

200! Would end in 40 + 8 + 1 = 49 zeroes.

When you add the two numbers (one with 24 zeroes and the other with 49 zeroes at it’s end), the resultant total would end in 24 zeroes. Option (b) is correct.

13.The given expression has fifteen 2’s and seventeen 5’s. The number of zeroes would be 15 as the number of 2’s is lower in this case. Option (d) is correct.

14.1! to 4! would have no zeroes while 5! to 9! All the values would have 1 zero. Thus, a total of 5 zeroes till 9!. Going further 10! to 14! would have two zeroes each – so a total of 10 zeroes would come out of the product of 10! × 11! × 12! × 13! × 14!.

Continuing this line of thought further we get:

Number of zeroes between 15! × 16!... × 19! = 3 + 3 + 3 + 3 + 3 = 3 × 5 = 15

Number of zeroes between 20! × 21!... × 24! = 4 × 5 = 20

Number of zeroes between 25! × 26!... × 29! = 6 × 5 = 30

Number of zeroes between 30! × 31!... × 34! = 7 × 5 = 35

Number of zeroes between 35! × 36!... × 39! = 8 × 5 = 40

Number of zeroes between 40! × 41!... × 44! = 9 × 5 = 45

Number of zeroes between 45! × 46!... × 49! = 10 × 5 = 50

Number of zeroes for 50! = 12

Thus, the total number of zeroes for the expression 1! × 2! × 3! …. × 50! = 5 + 10 +15 + 20 + 30 + 35 + 40 + 45 + 50 + 12 = 262 zeroes. Option (c) is correct.

15.The number of 5’s is `15 while the number of 2’s is much more. Option (b) is correct.

16.The number of zeroes would depend on the number of 5’s in the value of the factorial.

100! would end in 20 + 4 = 24 zeroes

200! Would end in 40 + 8 + 1 = 49 zeroes.

When you multiply the two numbers (one with 24 zeroes and the other with 49 zeroes at it’s end), the resultant total would end in 24 + 49 = 73 zeroes. Option (b) is correct.

Co-Prime or Relatively Prime Numbers Two or more numbers that do not have a common factor are known as co-prime or relatively prime. In other words, these numbers have a highest common factor of unity.

If two numbers m and n are relatively prime and the natural number x is divisible by both m and n independently then the number x is also divisible by mn.

Key Concept 1: The spotting of two numbers as co-prime has a very important implication in the context of the two numbers being in the denominators of fractions.

The concept is again best understood through an example:

Suppose, you are doing an operation of the following format – M/8 + N/9 where M & N are integers.

What are the chances of the result being an integer, if M is not divisible by 8 and N is not divisible by 9? A little bit of thought will make you realise that the chances are zero. The reason for this is that 8 and 9 are co-prime and the decimals of co-prime numbers never match each other.

Note: this will not be the case in the case of:

M/3 + N/27.

In this case even if 3 and 27 are not dividing M and N respectively, there is a possibility of the values of M and N being such that you have an integral answer.

For instance: 5/3 + 36/27 = 81/27 = 3

The result will never be integral if the two denominators are co-prime.

Note: This holds true even for expressions of the nature A/7 – B/6 etc.

This has huge implications for problem solving especially in the case of solving linear equations related to word based problems. Students are advised to try to use these throughout Blocks I, II and III of this book.

Example: Find all five-digit numbers of the form 34 x 5Y that are divisible by 36.

Solution: 36 is a product of two co-primes 4 and 9. Hence, if 34 x 5y is divisible by 4 and 9, it will also be divisible by 36. Hence, for divisibility by 4, we have that the value of y can be 2 or 6. Also, if y is 2 the number becomes 34 x 52. For this to be divisible by 9, the addition of 3 + 4 + x + 5 + 2 should be divisible by 9. For this x can be 4.

Hence the number 34452 is divisible by 36.

Also for y = 6, the number 34 x 56 will be divisible by 36 when the addition of the digits is divisible by 9. This will happen when x is 0 or 9. Hence, the numbers 34056 and 34956 will be divisible by 36.

Exercise for Self-practice

Find all numbers of the form 56x3y that are divisible by 36.

Find all numbers of the form 72xy that are divisible by 45.

Find all numbers of the form 135xy that are divisible by 45.

Find all numbers of the form 517xy that are divisible by 89.

Divisibility Rules

Divisibility by 2 or 5: A number is divisible by 2 or 5 if the last digit is divisible by 2 or 5.

Divisibility by 3 (or 9): All such numbers the sum of whose digits are divisible by 3 (or 9) are divisible by 3 (or 9).

Divisibility by 4: A number is divisible by 4 if the last 2 digits are divisible by 4.

Divisibility by 6: A number is divisible by 6 if it is simultaneously divisible by 2 and 3.

Divisibility by 8: A number is divisible by 8 if the last 3 digits of the number are divisible by 8.

Divisibility by 11: A number is divisible by 11 if the difference of the sum of the digits in the odd places and the sum of the digits in the even places is zero or is divisible by 11.

Divisibility by 12: All numbers divisible by 3 and 4 are divisible by 12.

Divisibility by 7, 11 or 13: The integer n is divisible by 7, 11 or 13 if and only if the difference of the number of its thousands and the remainder of its division by 1000 is divisible by 7, 11 or 13.

For Example: 473312 is divisible by 7 since the difference between 473 – 312 = 161 is divisible by 7.

Even Numbers: All integers that are divisible by 2 are even numbers. They are also denoted by 2n.

Example: 2, 4, 6, 12, 122, –2, –4, –12.

Also note that zero is an even number.

2 is the lowest positive even number.

Odd Numbers: All integers that are not divisible by 2 are odd numbers. Odd number leave a remainder of 1 on being divided by 2. They are denoted by 2n + 1 or 2n – 1.

Lowest positive odd number is 1.

Example: –1, –3, –7, –35, 3, 11, etc.

Complex Numbers: The arithmetic combination of real numbers and imaginary numbers are called complex numbers.

Alternately: All numbers of the form a + ib, where i = ÷ – 1 are called complex number.

Twin Primes: A pair of prime numbers are said to be twin prime when they differ by 2.

Example: 3 and 5 are Twin Primes, so also are 11 and 13.

Perfect Numbers: A number n is said to be a perfect number if the sum of all the divisors of n (including n) is equal to 2n.

Example: 6 = 1 × 2 × 3 sum of the divisors = 1 + 2 + 3 + 6 = 12 = 2 × 6

28 = 1, 2, 4, 7, 14, 28, = 56 = 2 × 28

Task for student: Find all perfect numbers below 1000.

Mixed Numbers: A number that has both an integral and a fractional part is known as a mixed number.

Triangular Numbers: A number which can be represented as the sum of consecutive natural numbers starting with 1 are called as triangular numbers.

e.g.: 1 + 2 + 3 + 4 = 10. So, 10 is a triangular number.

Certain Rules

1.Of n consecutive whole numbers a, a + 1…a + n – 1, one and only one is divisible by n.

2.Mixed numbers: A number that has both the integral and fractional part is known as mixed number.

3.If a number n can be represented as the product of two numbers p and q, that is, n = p q, then we say that the number n is divisible by p and by q and each of the numbers p and q is a divisor of the number n. Also, each factor of p and q would be a divisor of n.

4.Any number n can be represented in the decimal system of numbers as

N = ak × 10k + ak – 1 × 10k – 1 + …+ ai × 10 + a0

Example: 2738 can be written as: 2 × 103 + 7 × 102 + 3 × 101 + 8 × 100.

5.3n will always have an even number of tens. (Example: 2 in 27,8 in 81,24 in 243,72 in 729 and so on.)

6.A sum of 5 consecutive whole numbers will always be divisible by 5.

7.The difference between 2 two digit numbers:

(xy) – (yx) will be divisible by 9

8.The square of an odd number when divided by 8 will always leave a remainder of 1.

9.The product of 3 consecutive natural numbers is divisible by 6.

10.The product of 3 consecutive natural numbers the first of which is even is divisible by 24.

11.Products:

Odd × odd = odd

Odd × even = even

Even × even = even

12.All numbers not divisible by 3 have the property that their square will have a remainder of 1 when divided by 3.

13.(a2 + b2)/(b2 + c2) = (a2/b2) if a/b = b/c.

14.The product of any r consecutive integers (numbers) is divisible by r!

15.If m and n are two integers then (m + n)! is divisible by m!n!

16.Difference between any number and the number obtained by writing the digits in reverse order is divisible by 9. (for any number of digits)

17.Any number written in the form 10n – 1 is divisible by 3 and 9.

18.If a numerical expression contains no parentheses, first the operations of the third stage (involution or raising a number to a power) are performed, then the operations of the second stage (multiplication and division) and, finally, the operations of the first stage (addition and subtraction) are performed. In this case the operations of one and the same stage are performed in the sequence indicated by the notation. If an expression contains parentheses, then the operation indicated in the parentheses are to be performed first and then all the remaining operations. In this case operations of the numbers in parentheses as well as standing without parentheses are performed in the order indicated above.

If a fractional expression is evaluated, then the operations indicated in the numerator and denominator of the function are performed and the first result is divided by the second.

19.(a)n/(a + 1) leaves a remainder of

a if n is odd

1 if n is even

20.(a + 1)n/a will always give a remainder of 1.

21.For any natural number n, n5 has the same units digit as n has.

22.For any natural number: n3n is divisible by 6.

23.The remainder of gives a remainder of (n –1)

THE REMAINDER THEOREM

Consider the following question:

17 × 23.

Suppose you have to find the remainder of this expression when divided by 12.

We can write this as:

17 × 23 = (12 + 5) × (12 + 11)

Which when expanded gives us:

12 × 12 + 12 × 11 + 5 × 12 + 5 × 11

You will realise that, when this expression is divided by 12, the remainder will only depend on the last term above:

Thus, gives the same remainder as

Hence, 7.

This is the remainder when 17 × 23 is divided by 12.

Learning Point: In order to find the remainder of 17 × 23 when divided by 12, you need to look at the individual remainders of 17 and 23 when divided by 12. The respective remainders (5 and 11) will give you the remainder of the original expression when divided by 12.

Mathematically, this can be written as:

The remainder of the expression [A × B × C + D × E]/M, will be the same as the remainder of the expression [AR × BR × CR + DR × ER]/M.

Where AR is the remainder when A is divided by M,

BR is the remainder when B is divided by M,

CR is the remainder when C is divided by M

DR is the remainder when D is divided by M and

ER is the remainder when E is divided by M,

We call this transformation as the remainder theorem transformation and denote it by the sign

Thus, the remainder of

1421 × 1423 × 1425 when divided by 12 can be given as:

= .

gives us a remainder of 3.

In the above question, we have used a series of remainder theorem transformations (denoted by ) and equality transformations to transform a difficult looking expression into a simple expression.

Try to solve the following questions on Remainder theorem:

Find the remainder in each of the following cases:

1.17 × 23 × 126 × 38 divided by 8.

2.243 × 245 × 247 × 249 × 251 divided by 12.

  1. + .

4..

  1. + .

USING NEGATIVE REMAINDERS

Consider the following question:

Find the remainder when: 14 × 15 is divided by 8.

The obvious approach in this case would be

= 2 (Answer).

However there is another option by which you can solve the same question:

When 14 is divided by 8, the remainder is normally seen as + 6. However, there might be times when using the negative value of the remainder might give us more convenience. Which is why you should know the following process:

Concept Note: Remainders by definition are always non-negative. Hence, even when we divide a number like – 27 by 5 we say that the remainder is 3 (and not – 2). However, looking at the negative value of the remainder—it has its own advantages in Mathematics as it results in reducing calculations.

Thus, when a number like 13 is divided by 8, the remainder being 5, the negative remainder is – 3.

Note: It is in this context that we mention numbers like 13, 21, 29, etc. as 8n + 5 or 8n – 3 numbers.

Thus will give us R Æ 2.

Consider the advantage this process will give you in the following question:

2.

(The alternative will involve long calculations. Hence, the principle is that you should use negative remainders wherever you can. They can make life much simpler!!!)

What if the Answer Comes Out Negative

For instance, R Æ .

But, we know that a remainder of –24, equals a remainder of 42 when divided by 66. Hence, the answer is 42.

Of course nothing stops you from using positive and negative remainders at the same time in order to solve the same question –

Thus R Æ –1 R Æ 8.

Dealing with large powers There are two tools which are effective in order to deal with large powers –

(A)If you can express the expression in the form , the remainder will become 1 directly. In such a case, no matter how large the value of the power n is, the remainder is 1.

For instance, 1. In such a case the value of the power does not matter.

(B). In such a case using –1 as the remainder it will be evident that the remainder will be +1 if n is even and it will be –1 (Hence a – 1) when n is odd.

e.g.: 7

ANOTHER IMPORTANT POINT

Suppose you were asked to find the remainder of 14 divided by 4. It is clearly visible that the answer should be 2.

But consider the following process:

14/4 = 7/2 1 (The answer has changed!!)

What has happened?

We have transformed 14/4 into 7/2 by dividing the numerator and the denominator by 2. The result is that the original remainder 2 is also divided by 2 giving us 1 as the remainder. In order to take care of this problem, we need to reverse the effect of the division of the remainder by 2. This is done by multiplying the final remainder by 2 to get the correct answer.

Note: In any question on remainder theorem, you should try to cancel out parts of the numerator and denominator as much as you can, since it directly reduces the calculations required.

AN APPLICATION OF REMAINDER THEOREM

Finding the last two digits of an expression:

Suppose you had to find the last 2 digits of the expression:

22 × 31 × 44 × 27 × 37 × 43

The remainder the above expression will give when it is divided by 100 is the answer to the above question.

Hence, to answer the question above find the remainder of the expression when it is divided by 100.

Solution:

= (on dividing by 4)

=

= 14

Thus the remainder being 14, (after division by 4). The actual remainder should be 56.

[Don’t forget to multiply by 4 !!]

Hence, the last 2 digits of the answer will be 56.

Using negative remainders here would have helped further.

Note: Similarly finding the last three digits of an expression means finding the remainder when the expression is divided by 1000.

Exercise for Self-practice

1.

Find the remainder when 73 + 75 + 78 + 57 + 197 is divided by 34.

(a) 32

(b) 4

(c) 15

(d) 28

2.

Find the remainder when 73 × 75 × 78 × 57 × 197 is divided by 34.

(a) 22

(b) 30

(c) 15

(d) 28

3.

Find the remainder when 73 × 75 × 78 × 57 × 197 × 37 is divided by 34.

(a) 32

(b) 30

(c) 15

(d) 28

4.

Find the remainder when 43197 is divided by 7.

(a) 2

(b) 4

(c) 6

(d) 1

5.

Find the remainder when 51203 is divided by 7.

(a) 4

(b) 2

(c) 1

(d) 6

6.

Find the remainder when 5928 is divided by 7.

(a) 2

(b) 4

(c) 6

(d) 1

7.

Find the remainder when 6799 is divided by 7.

(a) 2

(b) 4

(c) 6

(d) 1

8.

Find the remainder when 7580 is divided by 7.

(a) 4

(b) 3

(c) 2

(d) 6

9.

Find the remainder when 4177 is divided by 7.

(a) 2

(b) 1

(c) 6

(d) 4

10.

Find the remainder when 21875 is divided by 17.

(a) 8

(b) 13

(c) 16

(d) 9

11.

Find the remainder when 54124 is divided by 17.

(a)

4

(b) 5

(c) 13

(d) 15

12.

Find the remainder when 83261 is divided by 17.

(a) 13

(b) 9

(c) 8

(d) 2

13.

Find the remainder when 25102 is divided by 17.

(a) 13

(b) 15

(c) 4

(d) 2

ANSWER KEY

1. (b)

2. (a)

3. (a)

4. (d)

5. (a)

6. (b)

7. (d)

8. (a)

9. (c)

10. (b)

11. (a)

12. (d)

13. (c)

Solutions

1.The remainder would be given by: (5 + 7 + 10 + 23 + 27)/34 = 72/34 Æ remainder = 4. Option (b) is correct.

2.The remainder would be given by: (5 × 7 × 10 × 23 × 27)/34 Æ 35 × 230 × 27/ 34 Æ 1 × 26 × 27/34 = 702/34 Æ remainder = 22. Option (a) is correct.

3.The remainder would be given by: (5 × 7 × 10 × 23 × 27 × 3)/34 Æ 35 × 230 × 27 × 3/34 Æ 1 × 26 × 81/34 Æ 26 × 13/34 = 338/34 Æ remainder = 32. Option (a) is correct.

4.43197/7 Æ 1197/7 Æ remainder =1. Option (d) is correct.

5.51203/7 Æ 2203/7 = (23)67 × 22/7 = 867 × 4/7 Æ remainder = 4. Option (a) is correct.

6.5928/7 Æ 328/7 = (36)4 × 34/7 = 7296 × 81/7 Æ remainder = 4. Option (b) is correct.

7.6799/7 Æ 499/7 = (43)33/7 = 6433/7 Æ remainder = 1. Option (d) is correct.

8.7580/7 Æ 580/7 = (56)13 × 5 2/7 Æ 113 × 25/7 Æ remainder = 4. Option (a) is correct.

9.4177/7 Æ 677/7 Æ remainder = 6 (as the expression is in the form an/(a + 1). Option (c) is correct.

10.21875/17 Æ 4875/17 = (44)n × 43/17 = 256n × 64/17 Æ 1n × 13/17 Æ remainder =13. Option (b) is correct.

11.54124/17 Æ 3124/17. At this point, like in each of the other questions solved above, we need to plan the power of 3 which would give us a convenient remainder of either 1 or –1. As we start to look for remainders that powers of 3 would have when divided by 17, we get that at the power 36 the remainder is 15. If we convert this to –2 we will get that at the fourth power of 36, we should get a 16/17 situation (as –2 × – 2 × – 2 × – 2 = 16). This means that at a power of 324 we are getting a remainder of 16 or –1. Naturally then if we double the power to 348, the remainder would be 1.

With this thinking we can restart solving the problem:

3124/17 = 348 × 348 × 324 × 34/17 Æ 1 × 1 × 16 × 81/17 Æ 16 × 13/17 =208/17 Æ remainder = 4. Option (a) is correct.

(Note that if we are dividing a number by 17 and if we see the remainder as 15, we can logically say that the remainder is –2 – even though negative remainders are not allowed in mathematics)

12.Using the logic developed in Question 11 above, we have 83261/17 Æ 15261/17 Æ

(–2) 261/17 Æ (–24)65 × (–2) /17 Æ 1665 × (–2)/17 Æ (–1) × (–2)/17 Æ remainder = 2. Option (d) is correct.

13.25102/17 Æ 8102/17 = 2306/17 = (24)76 × 22/17 Æ 1676 × 4/17 Æ 1 × 4/17 Æ remainder = 4. Option (c) is correct.

BASE SYSTEM

All the work we carry out in our number system is called as the decimal system. In other words we work in the decimal system. Why is it called decimal?? It is because there are 10 digits in the system 0–9.

However, depending on the number of digits contained in the base system other number systems are also possible. Thus a number system with base 2 is called the binary number system and will have only two digits 0 and 1. Some of the most commonly used systems are: Binary (base 2), Octal (base 8), Hexadecimal (base 16).

Binary system has 2 digits : 0, 1. Octal has 8 digits : – 0, 1, 2, 3, ... 7.

Hexadecimal has 16 digits – 0, 1, 2, ... 9, A, B, C, D, E, F.

Where A has a value 10, B = 11 and so on.

Before coming to the questions asked under this category, let us first look at a few issues with regard to converting numbers between different base systems.

  1. Conversion from any base system into decimal:

Suppose you have to write the decimal equivalent of the base 8 number 1468.

In such a case, follow the following structure for conversion:

1468 = 1 × 82 + 4 × 81 + 6 × 80

= 64 + 32 + 6 = 102.

Note: If you remember the process, for writing the value of any random number, say 146, in our decimal system (base 10) we use: 1 × 102 + 4 × 101 + 6 × 100. All you need to change, in case you are trying to write the value of the number in base 8, is that you replace 10 with 8 in every power.

Try to write the decimal equivalents of the following numbers:

1435, 1436, 1437,1438,1439

12567, 12568, 12569.

  1. Conversion of a number in decimals into any base:

Suppose you have to find out the value of the decimal number 347 in base 6. The following process is to be adopted:

Step 1: Find the highest power of the base (6 in this case) that is contained in 347. In this case you will realise that the value of 63 = 216 is contained in 347, while the value of 64 = 1296 is not contained in 347. Hence, we realise that the highest power of 6 contained in 347 is 3. This should make you realise that the number has to be constructed by using the powers 60, 61, 62, 63 respectively. Hence, a 4-digit number.

Structure of number: - - - -

Step 2: We now need to investigate how many times each of the powers of 6 is contained in 347. For this we first start with the highest power as found above. Thus we can see that 63 (216) is contained in 347 once. Hence our number now becomes:

1 - - -

That is, we now know that the first digit of the number is 1. Besides, when we have written the number 1 in this place, we have accounted for a value of 216 out of 347. This leaves us with 131 to account for.

We now need to look for the number of times 62 is contained in 131. We can easily see that 62 = 36 is contained in 131 three times. Thus, we write 3 as the next digit of our number which will now look like:

1 3 - -

In other words we now know that the first two digits of the number are 13. Besides, when we have written the number 3 in this place, we have accounted for a value of 108 out of 131 which was left to be accounted for. This leaves us with 131–108 = 23 to account for.

We now need to look for the number of times 61 is contained in 23. We can easily see that 61 = 6 is contained in 23 three times. Thus, we write 3 as the next digit of our number which will now look like:

1 3 3 -

In other words we now know that the first three digits of the number are 133. Besides, when we have written the number 3 in this place, we have accounted for a value of 18 out of the 23 which was left to be accounted for. This leaves us with 23 – 18 = 5 to account for.

The last digit of the number corresponds to 60 = 1. In order to make a value of 5 in this place we will obviously need to use this power of 6, 5 times thus giving us the final digit as 5. Hence, our number is:

1 3 3 5.

A few points you should know about base systems:

(1)In single digits there is no difference between the value of the number—whichever base we take. For example, the equality 56 = 57 = 58 = 59 = 510.

(2)Suppose you have a number in base x. When you convert this number into its decimal value, the value should be such that when it is divided by x, the remainder should be equal to the units digit of the number in base x.

In other words, 3428 will be a number of the form 8n + 2 in base 10. You can use this principle for checking your conversion calculations.

The following table gives a list of decimal values and their binary, octal and hexadecimal equivalents:

Decimal

Binary

Octal

Hexadecimal

0

0

0

0

1

1

1

1

2

10

2

2

3

11

3

3

4

100

4

4

5

101

5

5

6

110

6

6

7

111

7

7

8

1000

10

8

9

1001

11

9

10

1010

12

A

11

1011

13

B

Illustrations

1.The number of x digit numbers in nth base system will be

(a)nx   (b) nx– 1

(c) nx n  (d) nx n(x – 1)

Solution Base Æ n, digit Æ x

So, required number of numbers = nx n(x – 1)

2.The number of 2 digit numbers in binary system is

(a) 2    (b) 90

(c) 10    (d) 4

Solution By using the formula, we get the required number of numbers = 22 – 21 = 2

fi Option (a)

3.The number of 5 digit numbers in binary system is

(a)48  (b) 16

(c)32  (d) 20

Solution Required number of numbers = 25 – 24 = 32 – 16 = 16

fi Option (b)

4.I celebrate my birthday on 12th January on earth. On which date would I have to celebrate my birthday if I were on a planet where binary system is being used for counting. (The number of days, months and years are same on both the planets.)

(a)11th Jan   (b) 111th Jan

(c)110th Jan   (d) 1100th Jan

Solution On earth (decimal system is used). 12th Jan fi 12th Jan

The 12th day on the planet where binary system is being used will be called

(12)10 = (?)2

=

i.e., 1100th day on that planet

So, 12th January on earth = 1100th January on that planet

fi Option (d)

5.My year of birth is 1982. What would the year have been instead of 1982 if base 12 were used (for counting) instead of decimal system?

(a) 1182   (b) 1022

(c) 2082   (d) 1192

Solution The required answer will equal to (1982)10 = (?)12.

= Æ

1 × 123 + 1 × 122 + 9 × 121 + 2 × 90 = 1728 + 144 + 108 + 2 = 1982

Hence, the number (1192)12 represents 1982 in our base system.

fi Option (d)

6.203 in base 5 when converted to base 8, becomes

(a) 61   (b) 53

(c)145   (d) 65

Solution (203)5 = (?)10

= 2 × 52 + 0 × 51 + 3 × 50

= 50 + 0 + 3 = 53

Now,

(53)10 = (?)8

=

= (203)5 = (65)8

fi Option (d)

  1. (52)7+ 468 = (?)10

(a)(75) 10   (b) (50)10

(c)(39) 39   (d) (28) 10

Solution (52)7 = (5 × 71 + 2 × 70)10 = (37)10

also, (46)8 = (4 × 81 + 6 × 80)10 = (38)10

sum = (75)10

fi Option (a)

8.(23)5 + (47) 9 = (?)8

(a)70   (b) 35

(c)64   (d) 18

Solution (23)5 = (2 × 51 + 3 × 50)10 = (13)10 = (1 × 81 + 5 × 80)8 = (15)8

also,(47)9 = (4 × 91 + 7 × 90) 10 = (43)10

= (5 × 81 + 3 × 80) 8 = (53)8

sum = (13)10 + (43)10 = (56)10 Æ (70)8

fi Option (a)

9.(11)2 + (22)3 + (33)4 + (44)5 + (55)6 + (66)7 + (77)8 + (88)9 = (?)10

(a) 396   (b) 276

(c)250   (d) 342

Solution (11)2 = (1 × 21 + 1 × 20)10 = (3) 10

(22)3 = (2 × 31 + 2 × 30)10 = (8)10

(33)4 = (3 × 41 + 3 × 40) 10 = (15)10

(44)5 = (4 × 51 + 4 × 50)10 = (24)10

(55)6 = (5 × 61 + 5 × 60)10 = (35)10

(66)7 = (6 × 71 + 6 × 70)10 = (48)10

(77)8 = (7 × 81 + 7 × 80)10 = (63)10

(88)9 = (8 × 91 + 8 × 90)10 = (80)10

sum = (276)10

fi Option (b)

10.(24)5 × (32)5 = (?)5

(a)1423  (b) 1422

(c)1420  (d) 1323

Solution (24)5 =1410 and 325 = 1710. Hence, the required answer can be got by 14 × 17 = 23810 = 1 × 53 + 4 × 52 + 2 × 51 + 3 × 50 Æ 1423 as the correct answer.

Alternately, you could multiply directly in base 5 as follows:

(2 4)

×

(3 2)

(1 4 2 3)

Unit’s digit of the answer would correspond to: 4 × 2 = 8 Æ 135. Hence, we write 3 in the units place and carry over 1.

(Note that in this process when we are doing 4 × 2 we are effectively multiplying individual digits of one number with individual digits of the other number. In such a case we can write 4 × 2 = 8 by assuming that both the numbers are in decimal system as the value of a single digit in any base is equal.)

The tens digit will be got by: 2 × 2 + 4 × 3 = 16 + 1 = 17 Æ 325

Hence, we write 2 in the tens place and carry over 3 to the hundreds place.

Where we get 3 × 2 + 3 = 9 Æ 14

Hence, the answer is 14.

fi Option (a)

11.In base 8, the greatest four digit perfect square is

(a)9801   (b) 1024

(c)8701   (d) 7601

Solution In base 10, the greatest 4 digit perfect square = 9801

In base 9, the greatest 4 digits perfect square = 8701

In base 8, the greatest 4 digits perfect square = 7601

Alternately, multiply (77)8 × (77)8 to get 7601 as the answer.

Unit’s Digit

(A)The unit’s digit of an expression will be got by getting the remainder when the expression is divided by 10.

Thus for example if we have to find the units digit of the expression:

17 × 22 × 36 × 54 × 27 × 31 × 63

We try to find the remainder –

= = 6.

Hence, the required answer is 6.

This could have been directly got by multiplying: 7 × 2 × 6 × 4 × 7 × 1 × 3 and only accounting for the units’ digit.

(B)Unit’s digits in the contexts of powers –

Study the following table carefully.

Unit’s digit when ‘N’ is raised to a power

Number Ending With

Value of power

1

2

3

4

5

6

7

8

9

1

1

1

1

1

1

1

1

1

1

2

2

4

8

6

2

4

8

6

2

3

3

9

7

1

3

9

7

1

3

4

4

6

4

6

4

6

4

6

4

5

5

5

5

5

5

5

5

5

5

6

6

6

6

6

6

6

6

6

6

7

7

9

3

1

7

9

3

1

7

8

8

4

2

6

8

4

2

6

8

9

9

1

9

1

9

1

9

1

9

0

0

0

0

0

0

0

0

0

0

In the table above, if you look at the columns corresponding to the power 5 or 9 you will realize that the unit’s digit for all numbers is repeated (i.e. it is 1 for 1, 2 for, 3 for 3….9 for 9.)

This means that whenever we have any number whose unit’s digit is ‘x’ and it is raised to a power of the form 4n + 1, the value of the unit’s digit of the answer will be the same as the original units digit.

Illustrations: (1273)101 will give a unit’s digit of 3. (1547)25 will give a units digit of 7 and so forth.

Thus, the above table can be modified into the form –

Value of power

Number ending in

If the value of the Power is

4n + 1

4n + 2

4n + 3

4n

1

1

1

1

1

2

2

4

8

6

3

3

9

7

1

4

4

6

4

6

5

5

5

5

5

6

6

6

6

6

7

7

9

3

1

8

8

4

2

6

9

9

1

9

1

[Remember, at this point that we had said (in the Back to School section of Part 1) that all natural numbers can be expressed in the form 4n + x. Hence, with the help of the logic that helps us build this table, we can easily derive the units digit of any number when it is raised to a power.)

A special Case

Question: What will be the unit’s digit of (1273)122!?

Solution: 122! is a number of the form 4n. Hence, the answer should be 1. [Note: 1 here is derived by thinking of it as 3 (for 4n + 1), 9 (for 4n + 2), 7 (for 4n + 3), 1(for 4n)]

Exercise for Self-practice

Find the Units digit in each of the following cases:

1.

22 × 44 × 66 × 88

2.

11 × 22 × 33 × 44 × 55 × 66 …. × 100100

3.

17 × 23 × 51 × 32 + 15 × 17 × 16 × 22

4.

13 × 17 × 22 × 34 + 12 × 6 × 4 × 3 – 13 × 33

5.

37123 × 43144 × 57226 × 32127 × 525!

6.

67 × 37 × 43 × 91 × 42 × 33 × 42

(a) 2

(b) 6

(c) 8

(d) 4

7.

67 × 35 × 43 × 91 × 47 × 33 × 49

(a) 1

(b) 9

(c) 5

(d) 6

8.

67 × 35 × 45 × 91 × 42 × 33 × 81

(a) 2

(b) 4

(c) 0

(d) 8

9.

67 × 35 × 45 + 91 × 42 × 33 × 82

(a) 8

(b) 7

(c) 0

(d) 5

10.

(52)97 × (43)72

(a) 2

(b) 6

(c) 8

(d) 4

11.

(55)75 × (93)175 × (107)275

(a) 7

(b) 3

(c) 5

(d) 0

12.

(173)45 × (152)77 × (777)999

(a) 2

(b) 4

(c) 8

(d) 6

13.

81 × 82 × 83 × 84 × 86 × 87 × 88 × 89

(a) 0

(b) 6

(c) 2

(d) 4

14.

8243 × 8344 × 8497 × 8698 × 87105 × 8894

(a) 2

(b) 6

(c) 4

(d) 8

15.

432 × 532 + 532 × 974 + 537 × 531 + 947 × 997

(a) 5

(b) 6

(c) 9

(d) 8

ANSWER KEY

6. (d)

7. (c)

8. (c)

9. (b)

10. (a)

11. (c)

12. (c)

13. (b)

14. (b)

15. (d)

Solutions

1.The units digit would be given by the units digit of the multiplication of 4 x 6 x 6 x6 = 4

2.0

3.7 × 3 × 1 × 2 + 0 Æ 2 + 0 = 2

4.8 + 4 – 9 Æ 3

5.3 × 1 × 9 × 8 × 6 = 6

6.7 × 7 × 3 × 1 × 2 × 3 × 2 = 4

7.Since we have a 5 multiplied with odd numbers, the units digit would naturally be 5.

8.5 × 2 Æ 0

9.5 + 2 Æ 7

  1. 2 × 1 Æ 2

11.5 × 7 × 3 Æ 5

12.3 × 2 × 3 Æ 8

13.2 × 3 × 4 × 6 × 7 × 8 × 9 Æ 6

14.8 × 1 × 4 × 6 × 7 × 4 Æ 6

15.4 + 8 + 7 + 9 Æ 8

WORKED-OUT PROBLEMS

Problem 1.1 Find the number of zeroes in the factorial of the number 18.

Solution 18! contains 15 and 5, which combined with one even number give zeroes. Also, 10 is also contained in 18!, which will give an additional zero. Hence, 18! contains 3 zeroes and the last digit will always be zero.

Problem 1.2 Find the numbers of zeroes in 27!

Solution 27! = 27 × 26 × 25 × … × 20 × … × 15 × … × 10 × … × 5 × … × 1.

A zero can be formed by combining any number containing 5 multiplied by any even number. Similarly, everytime a number ending in zero is found in the product, it will add an additional zero. For this problem, note that 25 = 5 × 5 will give 2 zeroes and zeroes will also be got by 20, 15, 10 and 5. Hence 27! will have 6 zeroes.

Short-cut method: Number of zeroes is 27! Æ [27/5] + [27/25]

where [x] indicates the integer just lower than the fraction

Hence, [27/5] = 5 and [27/52] = 1, 6 zeroes

Problem 1.3 Find the number of zeroes in 137!

Solution [137/5] + [137/52] + [137/53]

= 27 + 5 + 1 = 33 zeroes

(since the restriction on the number of zeroes is due to the number of fives.)

Exercise for Self-practice

Find the number of zeroes in

(a) 81!

(b) 100!

(c) 51!

Answers

(a) 19

(b) 24

(c) 12

Problem 1.4 What exact power of 5 divides 87!?

Solution [87/5] + [87/25] = 17 + 3 = 20

Problem 1.5 What power of 8 exactly divides 25!?

Solution If 8 were a prime number, the answer should be [25/8] = 3. But since 8 is not prime, use the following process.

The prime factors of 8 is 2 × 2 × 2. For divisibility by 8, we need three twos. So, everytime we can find 3 twos, we add one to the power of 8 that divides 25! To count how we get 3 twos, we do the following. All even numbers will give one ‘two’ at least [25/2] = 12

Also, all numbers in 25! divisible by 22 will give an additional two [25/22] = 6

Further, all numbers in 25! divisible by 23 will give a third two. Hence [25!/23] = 3

And all numbers in 25! divisible by 24 will give a fourth two. Hence [25!/24] = 1

Hence, total number of twos in 25! is 22. For a number to be divided by 8, we need three twos. Hence, since 25! has 22 twos, it will be divided by 8 seven times.

Problem 1.6 What power of 15 divides 87! exactly?

Solution 15 = 5 × 3. Hence, everytime we can form a pair of one 5 and one 3, we will count one.

87! contains – [87/5] + [87/52] = 17 + 3 = 20 fives

Also 87! contains – [87/3] + [87/32] + [87/33] + [87/34] = 29 +…(more than 20 threes).

Hence, 15 will divide 87! twenty times since the restriction on the power is because of the number of 5s and not the number of 3s.

In fact, it is not very difficult to see that in the case of all factors being prime, we just have to look for the highest prime number to provide the restriction for the power of the denominator.

Hence, in this case we did not need to check for anything but the number of 5s.

Exercise for Self-practice

(a) What power of 30 will exactly divide 128!

Hints: [128/5] + [128/52] + [128/53]

(b) What power of 210 will exactly divide 142!

Problem 1.7 Find the last digit in the expression (36472)123! × (34767)76!.

Solution If we try to formulate a pattern for 2 and its powers and their units digit, we see that the units digit for the powers of 2 goes as: 2, 4, 8, 6, 2, 4, 8, 6, 2, 4, 8, 6 and so on. The number 2 when raised to a power of 4n + 1 will always give a units digit of 2. This also means that the units digit for 24n will always end in 6. The power of 36472 is 123! . 123! can be written in the form 4n. Hence, (36472)123! will end in 6.

The second part of the expression is (34767)76!. The units digit depends on the power of 7. If we try to formulate a pattern for 7 and its powers and their units digit, we see that the units digit for the powers of 7 go as: 7 9 3 1 7 9 3 1 and so on. This means that the units digit of the expression 74n will always be 1.

Since 76! can be written as a multiple of 4 as 4n, we can conclude that the unit’s digit in (34767)76! is 1.

Hence the units digit of (36472)123! × (34767)76! will be 6.

Counting

Problem 1.8 Find the number of numbers between 100 to 200 if

(i)Both 100 and 200 are counted.

(ii)Only one of 100 and 200 is counted.

(iii)Neither 100 nor 200 is counted.

Solution

(i)Both ends included-Solution: 200 – 100 + 1 = 101

(ii)One end included-Solution: 200 – 100 = 100

(iii)Both ends excluded-Solution: 200 – 100 – 1 = 99.

Problem 1.9 Find the number of even numbers between 122 and 242 if:

(i)Both ends are included.

(ii)Only one end is included.

(iii)Neither end is included.

Solution

(i)Both ends included—Solution: (242 – 122)/2 = 60 + 1 = 61

(ii)One end included-Solution: (242 – 122)/2 = 60

(iii)Both ends excluded-Solution: (242 – 122)/2 – 1 = 59

Exercise for Self-practice

(a)Find the number of numbers between 140 to 259, both included, which are divisible by 7.

(b)Find the number of numbers between 100 to 200, that are divisible by 3.

Problem 1.10 Find the number of numbers between 300 to 400 (both included), that are not divisible by 2, 3, 4, and 5.

Solution Total numbers: 101

Step 1: Not divisible by 2 = All even numbers rejected: 51

Numbers left: 50.

Step 2: Of which: divisible by 3 = first number 300, last number 399. But even numbers have already been removed, hence count out only odd numbers between 300 and 400 divisible by 3. This gives us that:

First number 303, last number 399, common difference 6

So, remove: [(399 – 303)/6] + 1 = 17.

\ 50 – 17 = 33 numbers left.

We do not need to remove additional terms for divisibility by 4 since this would eliminate only even numbers (which have already been eliminated)

Step 3: Remove from 33 numbers left all odd numbers that are divisible by 5 and not divisible by 3.

Between 300 to 400, the first odd number divisible by 5 is 305 and the last is 395 (since both ends are counted, we have 10 such numbers as: [(395 – 305)/10 + 1 = 10].

However, some of these 10 numbers have already been removed to get to 33 numbers.

Operation left: Of these 10 numbers, 305, 315…395, reduce all numbers that are also divisible by 3. Quick perusal shows that the numbers start with 315 and have common difference 30.

Hence [(Last number – First number)/Difference + 1] = [(375 – 315)/30 + 1] = 3

These 3 numbers were already removed from the original 100. Hence, for numbers divisible by 5, we need to remove only those numbers that are odd, divisible by 5 but not by 3. There are 7 such numbers between 300 and 400.

So numbers left are: 33 – 7 = 26.

Exercise for Self-practice

Find the number of numbers between 100 to 400 which are divisible by either 2, 3, 5 and 7.

Problem 1.11 Find the number of zeroes in the following multiplication: 5 × 10 × 15 × 20 × 25 × 30 × 35 × 40 × 45 × 50.

Solution The number of zeroes depends on the number of fives and the number of twos. Here, close scrutiny shows that the number of twos is the constraint. The expression can be written as

5 × (5 × 2) × (5 × 3) × (5 × 2 × 2) × (5 × 5) × (5 × 2 × 3) × (5 × 7) × (5 × 2 × 2 × 2) × (5 × 3 × 3) × (5 × 5 × 2)

Number of 5s – 12, Number of 2s – 8.

Hence: 8 zeroes.

Problem 1.12 Find the remainder for [(73 × 79 × 81)/11].

Solution The remainder for the expression: [(73 × 79 × 81)/11] will be the same as the remainder for [(7 × 2 × 4)/11]

That is, 56/11 fi remainder = 1

Problem 1.13 Find the remainder for (3560/8).

Solution (3560/8) = [(32)280/8] = (9280/8)

= [9.9.9…(280 times)]/8

remainder for above expression = remainder for [1.1.1…(280 times)]/8 fi remainder = 1.

Problem 1.14 Find the remainder when (22225555 + 55552222)/7.

Solution This is of the form: [(22225555)/7 + (55552222)/7]

We now proceed to find the individual remainder of : (22225555)/7. Let the remainder be R1.

When 2222 is divided by 7, it leaves a remainder of 3.

Hence, for remainder purpose (22225555)/7 (35555/7) = (3.35554)/7 = [3(32)2777]/7 = [3.(7 + 2)2777]/7 (3.22777)/7 = (3.22 ◊ 22775)/7 = [3.22 ◊ (23)925]/7

= [3.22 ◊ (8)925] /7 (12/7) Remainder = 5.

Similarly, (55552222)/7 (42222)/7 = [(22)2222]/7 = (2)4444/7 = (2.24443)/7 = [2.(23)1481]/7 = [2.(8)1481]/7 [2.(1)1481]/7 Æ 2 (remainder).

Hence, (22225555)/7 + (55552222)/7 (5 + 2)/7 fi Remainder = 0

Problem 1.15 Find the GCD and the LCM of the numbers 126, 540 and 630.

Solution The standard forms of the numbers are:

126 Æ 3 × 3 × 7 × 2 Æ 32 × 7 × 2

540 Æ 3 × 3 × 3 × 2 × 2 × 5 Æ 22 × 33 × 5

630 Æ 3 × 3 × 5 × 2 × 7 Æ 2 × 32 × 5 × 7

For GCD we use Intersection of prime factors and the lowest power of all factors that appear in all three numbers. 2 × 32 = 18.

For LCM Æ Union of prime factors and highest power of all factors that appear in any one of the three numbers fi 22 × 33 × 5 × 7 = 3780.

Exercise for Self-practice

Find the GCD and the LCM of the following numbers:

(i) 360, 8400

(ii) 120, 144

(iii) 275, 180, 372, 156

(iv) 70, 112

(v) 75, 114

(vi) 544, 720

Problem 1.16 The ratio of the factorial of a number x to the square of the factorial of another number, which when increased by 50% gives the required number, is 1.25. Find the number x.

(a) 6

(b) 5

(c) 9

(d) None of these

Solution Solve through options: Check for the conditions mentioned. When we check for option (a) we get 6 ! = 720 and (4!)2 = 576 and we have 6 !/(4!)2 = 1.25, which is the required ratio.

Hence the answer is (a)

Problem 1.17 Three numbers A, B and C are such that the difference between the highest and the second highest two-digit numbers formed by using two of A, B and C is 5. Also, the smallest two two-digit numbers differ by 2. If B> A > C then what is the value of B?

(a) 1

(b) 6

(c) 7

(d) 8

Solution Since B is the largest digit, option (a) is rejected. Check for option (b).

If B is 6, then the two largest two-digit numbers are 65 and 60 (Since, their difference is 5) and we have B = 6, A = 5 and C = 0.

But with this solution we are unable to meet the second condition. Hence (b) is not the answer. We also realise here that C cannot be 0.

Check for option (c).

B is 7, then the nos. are 76 and 71 or 75 and 70. In both these cases, the smallest two two-digit numbers do not differ by 2.

Hence, the answer is not (c).

Hence, option (d) is the answer

[To confirm, put B = 8, then the solution A = 6 and C = 1 satisfies the 2nd condition.]

Problem 1.18 Find the remainder when 2851 × (2862)2 × (2873)3 is divided by 23.

Solution We use the remainder theorem to solve the problem. Using the theorem, we see that the following expressions have the same remainder.

fi

fi

fi fi

fi fi fi Remainder is 18.

Problem 1.19 For what maximum value of n will the expression be an integer?

Solution For to be a integer, we need to look at the prime factors of 504 Æ

504 = 32 × 7 × 8 = 23 × 32 × 7

We thus have to look for the number of 7s, the number of 23s and the number of 32s that are contained in 10200!. The lowest of these will be the constraint value for n.

To find the number of 23s we need to find the number of 2s as

+

+ + + + +

+ + +

where [ ] is the greatest integer function.

= 5100 + 2550 + 1275 + 637 + 318 + 159 + 79 + 39 + 19 + 9 + 4 + 2 + 1

Number of twos = 10192

Hence, number of 23 = 3397

Similarly, we find the number of 3s as

Number of threes =

+

= 3400 + 1133 + 377 + 125 + 41 + 13 + 4 + 1

Number of threes = 5094

\ Number of 32 = 2547

Similarly we find the number of 7s as

= 1457 + 208 + 29 + 4 = 1698.

Thus, we have, 1698 sevens, 2547 nines and 3397 eights contained in 10200!.

The required value of n will be given by the lowest of these three [The student is expected to explore why this happens]

Hence, answer = 1698.

Short cut We will look only for the number of 7s in this case. Reason: 7 > 3 × 2. So, the number of 7s must always be less than the number of 23.

And 7 > 2 × 3, so the number of 7s must be less than the number of 32.

Recollect that earlier we had talked about the finding of powers when the divisor only had prime factors. There we had seen that we needed to check only for the highest prime as the restriction had to lie there.

In cases of the divisors having composite factors, we have to be slightly careful in estimating the factor that will reflect the restriction. In the above example, we saw a case where even though 7 was the lowest factor (in relation to 8 and 9), the restriction was still placed by 7 rather than by 9 (as would be expected based on the previous process of taking the highest number).

Problem 1.20 Find the units digit of the expression: 785562 × 56256 × 971250.

Solution We can get the units digits in the expression by looking at the patterns followed by 78, 56 and 97 when they are raised to high powers.

In fact, for the last digit we just need to consider the units digit of each part of the product.

A number (like 78) having 8 as the units digit will yield units digit as

781 Æ 8

785 Æ 8

84n + 1 Æ 8

84n + 2 Æ 4

Hence 785562 will yield

four as the units digit

782 Æ 4

786 Æ 4

783 Æ 2

787 Æ 2

784 Æ 6

788 Æ 6

Similarly,

561 Æ 6  Æ 56256 will yield 6 as

562 Æ 6  the units digit.

563 Æ 6

Similarly,

971 Æ 7

74n + 1 Æ 7

74n + 2 Æ 9

Hence, 971250 will yield a units digit of 9.

972 Æ 9

973 Æ 3

974 Æ 1

Hence, the required units digit is given by 4 × 6 × 9 Æ 6 (answer).

Problem 1.21 Find the GCD and the LCM of the numbers P and Q where P = 23 × 53 × 72 and Q = 33 × 54.

Solution GCD or HCF is given by the lowest powers of the common factors.

Thus,GCD = 53.

LCM is given by the highest powers of all factors available.

Thus,LCM = 23 × 33 × 54 × 72

Problem 1.22 A school has 378 girl students and 675 boy students. The school is divided into strictly boys or strictly girls sections. All sections in the school have the same number of students. Given this information, what are the minimum number of sections in the school.

Solution The answer will be given by the HCF of 378 and 675.

378 = 2 × 33 × 7

675 = 33 × 52

Hence, HCF of the two is 33 = 27.

Hence, the number of sections is given by: = 14 + 25 = 39 sections.

Problem 1.23 The difference between the number of numbers from 2 to 100 which are not divisible by any other number except 1 and itself and the numbers which are divisible by at least one more number along with 1 and itself.

(a) 25

(b) 50

(c) 49

(d) can’t be determine

Solution From 2 to 100.

The number of numbers which are divisible by 1 and itself only = 25

Also, the number of numbers which are divisible by at least one more number except 1 and itself (i.e composite numbers) 99 – 25 = 74

So, required difference = 74 – 25 = 49

fi Option (c)

Problem 1.24 If the sum of (2n +1) prime numbers where n ŒN is an even number, then one of the prime numbers must be

(a) 2

(b) 3

(c) 5

(d) 7

Solution For any n ΠN, 2n + 1 is odd.

Also, it is given in the problem that the sum of an odd number of prime numbers = even. Since all prime numbers except 2 are odd, the above condition will only be fulfilled if we have an (odd + odd + even) structure of addition. Since, the sum of the three prime numbers is said to be even, we have to include one even prime number. Hence 2 being the only even prime number must be included.

If we add odd number of prime numbers, not including 2 (two), we will always get an odd number, because

fi Option (a)

Problem 1.25 What will be the difference between the largest and smallest four digit number made by using distinct single digit prime numbers?

(a) 1800

(b) 4499

(c) 4495

(d) 5175

Solution Required largest number Æ 7532

Required smallest number Æ 2357

Difference Æ 5175

fi Option (d)

Problem 1.26 The difference between the two three-digit numbers XYZ and ZYX will be equal to

(a) difference between X and Z i.e. |xz|

(b) sum of X and z i.e (X + Z)

(c) 9 × difference between X and Z

(d) 99 × difference between X and Z

Solution From the property of numbers, it is known that on reversing a three digit number, the difference (of both the numbers) will be divisible by 99. Also, it is known that this difference will be equal to 99 × difference between the units and hundreds digits of the three digit number. fi Option (d)

Problem 1.27 When the difference between the number 842 and its reverse is divided by 99, the remainder will be

(a) 0

(b) 1

(c) 74

(d) 17

Solution From the property (used in the above question) we can say that the difference will be divisible by 99

fi Remainder = 0 (zero)

fi Option (a)

Problem 1.28 When the difference between the number 783 and its reverse is divided by 99, the quotient will be

(a) 1

(b) 10

(c) 3

(d) 4

Solution The quotient will be the difference between extreme digits of 783, i.e. 7 – 3 = 4 (This again is a property which you should know.)

fi Option (d)

Problem 1.29 A long Part of wood of same length when cut into equal pieces each of 242 cms, leaves a small piece of length 98 cms. If this Part were cut into equal pieces each of 22 cms, the length of the leftover wood would be

(a) 76 cm

(b) 12 cm

(c) 11 cm

(d) 10 cm

Solution As 242 is divisible by 22, so the required length of left wood will be equal to the remainder when 98 is divided by 22:

Hence, 10 [98/22; remainder 10]

fi Option (d)

Problem 1.30 Find the number of numbers from 1 to 100 which are not divisible by 2.

(a) 51

(b) 50

(c) 49

(d) 48

Solution The 1st number from 1 to 100, not divisible by 2 is 1 and the last number from 1 to 100, not divisible by 2 is 99.

Every alternate number (i.e, at the gap of 2) will not be divisible by 2 from 1 to 99. (1, 2, 3, - - - - , 95, 97, 99)

So, the required number of nos = =

fi Option (b)

Alternate method

Total number of nos from 1 to 100 = 100(i)

Now, if we count number of numbers from 1 to 100 which are divisible by 2 and subtract that from the total number of numbers from 1 to 100, as a result we will find the number of numbers from 1 to 100 which are not divisible by 2.

To count the number of nos from 1 to 100 which are divisible by 2:

The 1st number which is divisible by 2 = 2

The last number which is divisible by 2 = 100

(2, 4, 6, - - - - , 96, 98, 100)

Gap/step between two consecutive numbers = 2

So, the number of numbers which are divisible by 2 = = = (ii)

So, from (i) & (ii)

Required number of numbers = 100 – 50 = 50

fi Option (b)

Problem 1.31 Find the number of numbers from 1 to 100 which are not divisible by any one of 2 & 3.

(a) 16

(b) 17

(c) 18

(d) 33

Solution From 1 to 100

Number of numbers not divisible by 2 & 3 = Total number of numbers–number of numbers divisible by either 2 or 3.

Now, total number of numbers = 100(ii)

For number of numbers divisible by either 2 or 3:

Number of numbers divisible by 2 = =

Now, the number of numbers divisible by 3 (but not by 2, as it has already been counted)

1st such no. = 3 and the gap will be 6. Hence 2nd such no. will be 9, 3rd no. would be 15 and the last number would be 99. Hence this series is 3, 9, 15, ..., 93, 99

So, the number of numbers divisible by 3 (but not by 2) =

Hence, the number of numbers divisible by either 2 or 3 = 50 + 17 = 67

So, from (i), (ii) & (iii) required number of numbers = 100 – 67 = 33

fi Option (d)

Problem 1.32 Find the number of numbers from 1 to 100 which are not divisible by any one of 2, 3, and 5.

(a) 26

(b) 27

(c) 29

(d) 32

Solution From the above question, we have found out that

From 1 to 100, number of numbers divisible by 2 = 50(i)

Number of numbers divisible by 3 ( but not by 2) = 17(ii)

Now, we have to find out the number of numbers which are divisible by 5 (but not by 2 and 3). Numbers which are divisible by 5

(5) 10 15 20 (25) 30 (35) 40 45 50 (55) 60 (65) 70 75 80 (85) 90 (95) 100

That is, there are 7 such numbers(iii)

Another way to find out the number of numbers that are divisible by 5 but not 2 and 3 is to first only consider odd multiples of 5.

You will get the series of 10 numbers: 5, 15, 25, 35, 45, 55, 65, 75, 85 and 95

From amongst these we need to exclude multiples of 3. In other words, we need to find the number of common elements between the above series and the series of odd multiples of 3, viz, 3, 9, 15, 21 …. 99.

This situation is the same as finding the number of common elements between the two series for which we need to first observe that the first such number is 15. Then the common terms between these two series will themselves form an arithmetic series and this series will have a common difference which is the LCM of the common differences of the two series. (In this case the common difference of the two series are 10 and 6 respectively and their LCM being 30, the series of common terms between the two series will be 15, 45 and 75.) Thus, there will be 3 terms out of the 10 terms of the series 5, 15, 25…95 which will be divisible by 3 and hence need to be excluded from the count of numbers which are divisible by 5 but not 2 or 3.

Hence, the required answer would be: 100 – 50 – 17 – 7 = 26

fi Option (a)

Problem 1.33 Find the number of numbers from 1 to 100 which are not divisible by any one of 2, 3, 5 & 7.

(a) 22

(b) 24

(c) 23

(d) 27

Solution From the above question we have seen that from 1 to 100.

number of numbers divisible by 2 = 50(i)

number of numbers divisible by 3 but not by 2 = 17(ii)

number of numbers divisible by 5 but not by 2 and 3 = 7(iii)

number of numbers divisible by 7 but not by 2, 3 & 5;

such nos are 7, 49, 77, 91 = 4 nos(iv)

Required number of numbers = Total number of numbers from 1 to 100 – {(i) + (ii) + (iii) + (iv)}

= 100 – (50 + 17 + 7 + 4)

= 22

fi Option (a)

Problem 1.34 What will be the remainder when – 34 is divided by 5?

(a) 1

(b) 4

(c) 2

(d) – 4

Solution – 34 = 5 × (– 6) + (– 4)

Remainder = – 4, but it is wrong because remainder cannot be negative.

So,–34 = 5 × (– 7) + 1

fi Option (a)

Alternately, when you see a remainder of – 4 when the number is divided by 5, the required remainder will be equal to 5 – 4 = 1.

Problem 1.35 What will be the remainder when – 24.8 is divided by 6?

(a) 0.8

(b) 5.2

(c) –0.8

(d) – 5.2

Solution – 24.8 = 6 × (– 4) + (– 0.8)

Negative remainder, so not correct – 24.8 = 6 × (– 5) + 5.2

Positive value of remainder, so correct

fi Option (b)

Problem 1.36 If p is divided by q, then the maximum possible difference between the minimum possible and maximum possible remainder can be?

(a) pq

(b) p – 1

(c) q – 1

(d) None of these

Solution minimum possible remainder = 0 (when q exactly divides P)

Maximum possible remainder = q – 1

So, required maximum possible difference = (q – 1) – 0 = (q – 1)

fi Option (c)

Problem 1.37 Find the remainder when 2256 is divided by 17.

(a) 0

(b) 1

(c) 3

(d) 5

Solution fi R = 1

Q; R = 1

when n Æ even

fi Option (b)

Problem 1.38 Find the difference between the remainders when 784 is divided by 342 & 344.

(a) 0

(b) 1

(c) 3

(d) 5

Solution fi R = 1

also, = fi R = 1

The required difference between the remainders = 1 – 1 = 0

fi Option (a)

Problem 1.39 What will be the value of x for ; the remainder = 0

(a) 3

(b) 6

(c) 9

(d) 8

Solution

10017 – 1 = = fi divisible by 9 fi R = 0

Since the first part of the expression is giving a remainder of 0, the second part should also give 0 as a remainder if the entire remainder of the expression has to be 0. Hence, we now evaluate the second part of the numerator.

1034 + x =

with x at the right most place. In order for this number to be divisible by 9, the sum of digits should be divisible by 9.

fi 1 + 0 + 0 ... + 0 + x should be divisible by 9.

fi 1 + x should be divisible by 9 fi x = 8

fi Option (d)

LEVEL OF DIFFICULTY (I)

1.

The last digit of the number obtained by multiplying the numbers 81 × 82 × 83 × 84 × 85 × 86 × 87 × 88 × 89 will be

(a) 0

(b) 9

(c) 7

(d) 2

2.

The sum of the digits of a two-digit number is 10, while when the digits are reversed, the number decreases by 54. Find the changed number.

(a) 28

(b) 19

(c) 37

(d) 46

3.

When we multiply a certain two-digit number by the sum of its digits, 405 is achieved. If you multiply the number written in reverse order of the same digits by the sum of the digits, we get 486. Find the number.

(a) 81

(b) 45

(c) 36

(d) 54

4.

The sum of two numbers is 15 and their geometric mean is 20% lower than their arithmetic mean. Find the numbers.

(a) 11, 4

(b) 12, 3

(c) 13, 2

(d) 10, 5

5.

The difference between two numbers is 48 and the difference between the arithmetic mean and the geometric mean is two more than half of 1/3 of 96. Find the numbers.

(a) 49, 1

(b) 12, 60

(c) 50, 2

(d) 36, 84

6.

If A381 is divisible by 11, find the value of the smallest natural number A.

(a) 5

(b) 6

(c) 7

(d) 9

7.

If 381A is divisible by 9, find the value of smallest natural number A.

(a) 5

(b) 5

(c) 7

(d) 6

8.

What will be the remainder obtained when (96 + 1) will be divided by 8?

(a) 0

(b) 3

(c) 7

(d) 2

9.

Find the ratio between the LCM and HCF of 5, 15 and 20.

(a) 8 : 1

(b) 14 : 3

(c) 12 : 2

(d) 12 : 1

10.

Find the LCM of 5/2, 8/9, 11/14.

(a) 280

(b) 360

(c) 420

(d) None of these

11.

If the number A is even, which of the following will be true?

(a) 3A will always be divisible by 6

(b) 3A + 5 will always be divisible by 11

(c) (A2 + 3)/4 will be divisible by 7

(d) All of these

12.

A five-digit number is taken. Sum of the first four digits (excluding the number at the units digit) equals sum of all the five digits. Which of the following will not divide this number necessarily?

(a) 10

(b) 2

(c) 4

(d) 5

13.

A number 15B is divisible by 6. Which of these will be true about the positive integer B?

(a) B will be even

(b) B will be odd

(c) B will be divisble by 6

(d) Both (a) and (c)

14.

Two numbers P = 23.310.5 and Q = 25.31.71 are given. Find the GCD of P and Q.

(a) 2.3.5.7

(b) 3. 22

(c) 22.32

(d) 23.3

15.

Find the units digit of the expression 256251 + 36528 + 7354.

(a) 4

(b) 0

(c) 6

(d) 5

16.

Find the units digit of the expression 55725 + 735810 + 22853.

(a) 4

(b) 0

(c) 6

(d) 5

17.

Find the units digit of the expression 111 + 122 + 133 + 144 + 155 + 166.

(a) 1

(b) 9

(c) 7

(d) 0

18.

Find the units digit of the expression 111.122.133. 144.155.166.

(a) 4

(b) 3

(c) 7

(d) 0

19.

Find the number of zeroes at the end of 1090!

(a) 270

(b) 268

(c) 269

(d) 271

20.

If 146! is divisible by 5n, then find the maximum value of n.

(a) 34

(b) 35

(c) 36

(d) 37

21.

Find the number of divisors of 1420.

(a) 14

(b) 15

(c) 13

(d) 12

22.

Find the HCF and LCM of the polynomials (x2 – 5x + 6) and (x2 – 7x + 10).

(a) (x - 2), (x – 2) (x – 3) (x – 5)

(b) (x – 2), (x – 2)(x – 3)

(c) (x – 3), (x – 2) (x – 3) (x – 5)

(d) (x - 2), (x – 2) (x – 3) (x – 5)2

Directions for Questions 23 to 25: Given two different prime numbers P and Q, find the number of divisors of the following:

23.

P.Q

(a) 2

(b) 4

(c) 6

(d) 8

24.

P2Q

(a) 2

(b) 4

(c) 6

(d) 8

25.

P3Q2

(a) 2

(b) 4

(c) 6

(d) 12

26.

The sides of a pentagonal field (not regular) are 1737 metres, 2160 metres, 2358 metres, 1422 metres and 2214 metres respectively. Find the greatest length of the tape by which the five sides may be measured completely.

(a) 7

(b) 13

(c) 11

(d) 9

27.

There are 576 boys and 448 girls in a school that are to be divided into equal sections of either boys or girls alone. Find the minimum total number of sections thus formed.

(a) 24

(b) 32

(c) 16

(d) 20

28.

A milkman has three different qualities of milk. 403 gallons of 1st quality, 465 gallons of 2nd quality and 496 gallons of 3rd quality. Find the least possible number of bottles of equal size in which different milk of different qualities can be filled without mixing.

(a) 34

(b) 46

(c) 26

(d) 44

29.

What is the greatest number of 4 digits that when divided by any of the numbers 6, 9, 12, 17 leaves a remainder of 1?

(a) 9997

(b) 9793

(c) 9895

(d) 9487

30.

Find the least number that when divided by 16, 18 and 20 leaves a remainder 4 in each case, but is completely divisible by 7.

(a) 364

(b) 2254

(c) 2964

(d) 2884

31.

Four bells ring at the intervals of 6, 8, 12 and 18 seconds. They start ringing together at 12’O’ clock. After how many seconds will they ring together again?

(a) 72

(b) 84

(c) 60

(d) 48

32.

For Question 31, find how many times will they ring together during the next 12 minutes. (including the 12 minute mark)

(a) 9

(b) 10

(c) 11

(d) 12

33.

The units digit of the expression 125813 × 5533703 × 4532828 is

(a) 4

(b) 2

(c) 0

(d) 5

34.

Which of the following is not a perfect square?

(a) 1,00,856

(b) 3,25,137

(c) 9,45,729

(d) All of these

35.

Which of the following can never be in the ending of a perfect square?

(a) 6

(b) 00

(c) x 000 where x is a natural number

(d) 1

36.

The LCM of 5, 8,12, 20 will not be a multiple of

(a) 3

(b) 9

(c) 8

(d) 5

37.

Find the number of divisors of 720 (including 1 and 720).

(a) 25

(b) 28

(c) 29

(d) 30

38.

The LCM of (16 – x2) and (x2 + x – 6) is

(a) (x – 3) (x + 3) (4 – x2)

(b) 4 (4 – x2) (x + 3)

(c) (4 – x2) (x – 3)

(d) None of these

39.

GCD of x2 – 4 and x2 + x – 6 is

(a) x + 2

(b) x – 2

(c) x2 – 2

(d) x2 + 2

40.

The number A is not divisible by 3. Which of the following will not be divisible by 3?

(a) 9 × A

(b) 2 × A

(c) 18 × A

(d) 24 × A

41.

Find the remainder when the number 9100 is divided by 8.

(a) 1

(b) 2

(c) 0

(d) 4

42.

Find the remainder of 21000 when divided by 3.

(a) 1

(b) 2

(c) 4

(d) 6

43.

Decompose the number 20 into two terms such that their product is the greatest.

(a) x1 = x2 = 10

(b) x1 = 5, x2 = 15

(c) x1 = 16, x2 = 4

(d) x1 = 8, x2 = 12

44.

Find the number of zeroes at the end of 50!

(a) 13

(b) 11

(c) 5

(d) 12

45.

Which of the following can be a number divisible by 24?

(a) 4,32,15,604

(b) 25,61,284

(c) 13,62,480

(d) All of these

46.

For a number to be divisible by 88, it should be

(a) Divisible by 22 and 8

(b) Divisible by 11 and 8

(c) Divisible by 11 and thrice by 2

(d) All of these

47.

Find the number of divisors of 10800.

(a) 57

(b) 60

(c) 72

(d) 64

48.

Find the GCD of the polynomials (x + 3)2 (x – 2)(x + 1)2 and (x + 1)3 (x + 3) (x + 4).

(a) (x + 3)3 (x + 1)2 (x – 2) (x + 4)

(b) (x + 3) (x – 2) (x + 1)(x + 4)

(c) (x + 3) (x + 1)2

(d) (x + 1) (x + 3)2

49.

Find the LCM of (x + 3) (6x2 + 5x + 4) and (2x2 + 7x + 3) (x + 3)

(a) (2x + 1)(x + 3) (3x + 4)

(b) (4x2 – 1) (x + 3)2 (3x + 4)

(c) (4x2 – 1)(x + 3) (3x + 4)

(d) (2x – 1) (x + 3) (3x + 4)

50.

The product of three consecutive natural numbers, the first of which is an even number, is always divisible by

(a) 12

(b) 24

(c) 6

(d) All of these

51.

Some birds settled on the branches of a tree. First, they sat one to a branch and there was one bird too many. Next they sat two to a branch and there was one branch too many. How many branches were there?

(a) 3

(b) 4

(c) 5

(d) 6

52.

The square of a number greater than 1000 that is not divisible by three, when divided by three, leaves a remainder of

(a) 1 always

(b) 2 always

(c) 0

(d) either 1 or 2

53.

The value of the expression (153 ◊ 212)/(352 34) is

(a) 3

(b) 15

(c) 21

(d) 12

54.

If A = , B = , C = (0.3)2, D = (–1.2)2 then

(a) A > B > C > D

(b) D > A > B > C

(c) D > B > C > A

(d) D > C > A > B

55.

If 2 < x < 4 and 1 < y < 3, then find the ratio of the upper limit for x + y and the lower limit of x y.

(a) 6

(b) 7

(c) 8

(d) None of these

56.

The sum of the squares of the digits constituting a positive two-digit number is 13. If we subtract 9 from that number, we shall get a number written by the same digits in the reverse order. Find the number.

(a) 12

(b) 32

(c) 42

(d) 52

57.

The product of a natural number by the number written by the same digits in the reverse order is 2430. Find the numbers.

(a) 54 and 45

(b) 56 and 65

(c) 53 and 35

(d) 85 and 58

58.

Find two natural numbers whose difference is 66 and the least common multiple is 360.

(a) 120 and 54

(b) 90 and 24

(c) 180 and 114

(d) 130 and 64

59.

Find the pairs of natural numbers whose least common multiple is 78 and the greatest common divisor is 13.

(a) 58 and 13 or 16 and 29

(b) 68 and 23 or 36 and 49

(c) 18 and 73 or 56 and 93

(d) 78 and 13 or 26 and 39

60.

Find two natural numbers whose sum is 85 and the least common multiple is 102.

(a) 30 and 55

(b) 17 and 68

(c) 35 and 55

(d) 51 and 34

61.

Find the pairs of natural numbers the difference of whose squares is 55.

(a) 28 and 27 or 8 and 3

(b) 18 and 17 or 18 and 13

(c) 8 and 27 or 8 and 33

(d) 9 and 18 or 8 and 27

62.

Which of these is greater?

(a) 544 or 2112

(b) (0.4)4 or (0.8)3

63.

Is it possible for a common fraction whose numerator is less than the denominator to be equal to a fraction whose numerator is greater than the denominator?

(a) Yes

(b) No

64.

What digits should be put in place of c in 38c to make it divisible by

(1) 2

(2) 3

(3) 4

(4) 5

(5) 6

(6) 9

(7) 10

65.

Find the LCM and HCF of the following numbers: (54, 81, 135 and 189), (156, 195) and (1950, 5670 and 3900)

66.

The last digit in the expansions of the three digit number (34x)43 and (34x)44 are 7 and 1 respectively. What can be said about the value of x?

(a) x = 5

(b) x = 3

(c) x = 6

(d) x = 2

Directions for Questions 67 and 68: Amitesh buys a pen, a pencil and an eraser for ` 41. If the least cost of any of the three items is ` 12 and it is known that a pen costs less than a pencil and an eraser costs more than a pencil, answer the following questions:

67.

What is the cost of the pen?

(a) 12

(b) 13

(c) 14

(d) 15

68.

If it is known that the eraser’s cost is not divisible by 4, the cost of the pencil could be:

(a) 12

(b) 13

(c) 14

(d) 15

69.

A naughty boy Amrit watches an innings of Sachin Tendulkar and acts according to the number of runs he sees Sachin scoring. The details of these are given below.

1 run

Place an orange in the basket

2 runs

Place a mango in the basket

3 runs

Place a pear in the basket

4 runs

Remove a pear and a mango from the basket

One fine day, at the start of the match, the basket is empty. The sequence of runs scored by Sachin in that innings are given as 11232411234232341121314. At the end of the above innings, how many more oranges were there compared to mangoes inside the basket? (The Basket was empty initially).

(a) 4

(b) 5

(c) 6

(d) 7

70.

In the famous Bel Air Apartments in Ranchi, there are three watchmen meant to protect the precious fruits in the campus. However, one day a thief got in without being noticed and stole some precious mangoes. On the way out however, he was confronted by the three watchmen, the first two of whom asked him to part with 1/3rd of the fruits and one more. The last asked him to part with 1/5th of the mangoes and 4 more. As a result he had no mangoes left. What was the number of mangoes he had stolen?

(a) 12

(b) 13

(c) 15

(d) None of these

71.

A hundred and twenty digit number is formed by writing the first x natural numbers in front of each other as 12345678910111213… Find the remainder when this number is divided by 8.

(a) 6

(b) 7

(c) 2

(d) 0

72.

A test has 80 questions. There is one mark for a correct answer, while there is a negative penalty of –1/2 for a wrong answer and –1/4 for an unattempted question. What is the number of questions answered correctly, if the student has scored a net total of 34.5 marks?

(a) 45

(b) 48

(c) 54

(d) Cannot be determined

73.

For Question 72, if it is known that he has left 10 questions unanswered, the number of correct answers are:

(a) 45

(b) 48

(c) 54

(d) Cannot be determined

74.

Three mangoes, four guavas and five watermelons cost ` 750. Ten watermelons, six mangoes and 9 guavas cost `1580. What is the cost of six mangoes, ten watermelons and 4 guavas?

(a) 1280

(b) 1180

(c) 1080

(d) Cannot be determined

75.

From a number M subtract 1. Take the reciprocal of the result to get the value of ‘N’. Then which of the following is necessarily true?

(a) 0 < MN < 2

(b) MN > 3

(c) 1< MN < 3

(d) 1< MN < 5

76.

The cost of four mangoes, six guavas and sixteen watermelons is ` 500, while the cost of seven magoes, nine guavas and nineteen watermelons is ` 620. What is the cost of one mango, one guava and one watermelon?

(a) 120

(b) 40

(c) 150

(d) Cannot be determined

77.

For the question above, what is the cost of a mango?

(a) 20

(b) 14

(c) 15

(d) Cannot be determined

78.

The following is known about three real numbers, x, y and z.

– 4 < x < 4 , – 8 < y < 2 and – 8 < z < 2. Then the range of values that M = xz/y can take is best represented by:

(a) – • < x <

(b) – 16 < x < 8

(c) – 8 < x < 8

(d) – 16 < x < 16

79.

A man sold 38 pieces of clothing (combined in the form of shirts, trousers and ties). If he sold at least 11 pieces of each item and he sold more shirts than trousers and more trousers than ties, then the number of ties that he must have sold is:

(a) Exactly 11

(b) At least 11

(c) At least 12

(d) Cannot be determined

80.

For Question 79, find the number of shirts he must have sold.

(a) At least 13

(b) At least 14

(c) At least 15

(d) At most 16.

81.

Find the least number which when divided by 12, 15, 18 or 20 leaves in each case a remainder 4.

(a) 124

(b) 364

(c) 184

(d) None of these

82.

What is the least number by which 2800 should be multiplied so that the product may be a perfect square?

(a) 2

(b) 7

(c) 14

(d) None of these

83.

The least number of 4 digits which is a perfect square is:

(a) 1064

(b) 1040

(c) 1024

(d) 1012

84.

The least multiple of 7 which leaves a remainder of 4 when divided by 6, 9, 15 and 18 is

(a) 94

(b) 184

(c) 364

(d) 74

85.

What is the least 3 digit number that when divided by 2, 3, 4, 5 or 6 leaves a remainder of 1?

(a) 131

(b) 161

(c) 121

(d) None of these

86.

The highest common factor of 70 and 245 is equal to

(a) 35

(b) 45

(c) 55

(d) 65

87.

Find the least number, which must be subtracted from 7147 to make it a perfect square.

(a) 86

(b) 89

(c) 91

(d) 93

88.

Find the least square number which is divisible by 6, 8 and 15

(a) 2500

(b) 3600

(c) 4900

(d) 4500

89.

Find the least number by which 30492 must be multiplied or divided so as to make it a

perfect square.

(a) 11

(b) 7

(c) 3

(d) 2

90.

The greatest 4-digit number exactly divisible by 88 is

(a) 8888

(b) 9768

(c) 9944

(d) 9988

91.

By how much is three fourth of 116 greater than four fifth of 45?

(a) 31

(b) 41

(c) 46

(d) None of these

92.

If 5625 plants are to be arranged in such a way that there are as many rows as there are plants in a row, the number of rows will be:

(a) 95

(b) 85

(c) 65

(d) None of these

93.

A boy took a seven digit number ending in 9 and raised it to an even power greater than 2000. He then took the number 17 and raised it to a power which leaves the remainder 1 when divided by 4. If he now multiples both the numbers, what will be the unit’s digit of the number he so obtains?

(a) 7

(b) 9

(c) 3

(d) Cannot be determined

94.

Two friends were discussing their marks in an examination. While doing so they realized that both the numbers had the same prime factors, although Raveesh got a score which had two more factors than Harish. If their marks are represented by one of the options as given below, which of the following options would correctly represent the number of marks they got?

(a) 30,60

(b) 20,80

(c) 40,80

(d) 20,60

95.

A number is such that when divided by 3, 5, 6, or 7 it leaves the remainder 1, 3, 4, or 5 respectively. Which is the largest number below 4000 that satisfies this property?

(a) 3358

(b) 3988

(c) 3778

(d) 2938

96.

A number when divided by 2,3 and 4 leaves a remainder of 1. Find the least number (after 1) that satisfies this requirement.

(a) 25

(b) 13

(c) 37

(d) 17

97.

A number when divided by 2, 3 and 4 leaves a remainder of 1. Find the second lowest number (not counting 1) that satisfies this requirement.

(a) 25

(b) 13

(c) 37

(d) 17

98.

A number when divided by 2, 3 and 4 leaves a remainder of 1. Find the highest 2 digit number that satisfies this requirement.

(a) 91

(b) 93

(c) 97

(d) 95

99.

A number when divided by 2,3 and 4 leaves a remainder of 1. Find the highest 3 digit number that satisfies this requirement.

(a) 991

(b) 993

(c) 997

(d) 995

100.

A frog is sitting on vertex A of a square ABCD. It starts jumping to the immediately adjacent vertex on either side in random fashion and stops when it reaches point C. In how many ways can it reach point C if it makes exactly 7 jumps?

(a) 1

(b) 3

(c) 5

(d) 0

101.

Three bells ring at intervals of 5 seconds, 6 seconds and 7 seconds respectively. If they toll together for the first time at 9 AM in the morning, after what interval of time will they together ring again for the first time?

(a) After 30 seconds

(b) After 42 seconds

(c) After 35 seconds

(d) After 210 seconds

102.

For the question above, how many times would they ring, together in the next 1 hour?

(a) 17

(b) 18

(c) 19

(d) None of these

103.

A garrison has three kinds of soldiers. There are 66 soldiers of the first kind, 110 soldiers of the second kind and 242 soldiers of the third kind. It is desired to be arranging these soldiers in equal rows such that each row contains the same number of soldiers and there is only 1 kind of soldier in each row. What is the maximum number of soldiers who can be placed in each row?

(a) 11

(b) 1

(c) 22

(d) 33

104.

For the question above, what are the minimum number of rows that would be required to be formed?

(a) 11

(b) 19

(c) 18

(d) None of these

105.

A milkman produces three kinds of milk. On a particular day, he has 170 liters, 102 liters and 374 liters of the three kinds of milk. He wants to bottle them in bottles of equal sizes- so that each of the three varieties of milk would be completed bottled. How many bottle sizes are possible such that the bottle size in terms of liters is an integer?

(a) 1

(b) 2

(c) 4

(d) 34

106.

For the above question, what is the size of the largest bottle which can be used?

(a) 1

(b) 2

(c) 17

(d) 34

107.

For Question 105, what are the minimum number of bottles that would be required?

(a) 11

(b) 19

(c) 18

(d) None of these

108.

Find the number of zeroes at the end of 100!

(a) 20

(b) 23

(c) 24

(d) 25

109.

Find the number of zeroes at the end of 122!

(a) 20

(b) 23

(c) 24

(d) 28

110.

Find the number of zeroes at the end of 1400!

(a) 347

(b) 336

(c) 349

(d) 348

111.

Find the number of zeroes at the end of 380!

(a) 90

(b) 91

(c) 94

(d) 95

112.

Find the number of zeroes at the end of 72!

(a) 14

(b) 15

(c) 16

(d) 17

113.

The highest power of 3 that completely divides 40! is

(a) 18

(b) 15

(c) 16

(d) 17

114.

53!/3n is an integer. Find the highest possible value of n for this to be true.

(a) 19

(b) 21

(c) 23

(d) 24

115.

The highest power of 7 that completely divides 80! is:

(a) 12

(b) 13

(c) 14

(d) 15

116.

115!/7n is an integer. Find the highest possible value of n for this to be true.

(a) 15

(b) 17

(c) 16

(d) 18

117.

The highest power of 12 that completely divides 122! is:

(a) 54

(b) 56

(c) 57

(d) 58

118.

155!/20n is an integer. Find the highest possible value of n for this to be true.

(a) 77

(b) 38

(c) 75

(d) 37

119.

The minimum value of x so that x2/1024 is an integer is:

(a) 4

(b) 32

(c) 16

(d) 64

120.

Find the sum of all 2 digit natural numbers which leave a remainder of 3 when divided by 7.

(a) 650

(b) 663

(c) 676

(d) 702

121.

How many numbers between 1 and 200 are exactly divisible by exactly two of 3, 9 and 27?

(a) 14

(b) 15

(c) 16

(d) 17

122.

A number N is squared to give a value of S. The minimum value of N + S would happen when N is

(a) –0.3

(b) –0.5

(c) –0.7

(d) None of these

123.

L = x + y where x and y are prime numbers. Which of the following statement/s is/are true?

(i) The unit’s digit of L cannot be 5

(ii) The units digit of L cannot be 0.

(iii) L cannot be odd.

(a) All three

(b) Only iii

(c) only ii

(d) None

124.

XYZ is a 3 digit number such that when we calculate the difference between the two three digit numbers XYZYXZ the difference is exactly 90. How many possible values exist for the digits X and Y?

(a) 9

(b) 8

(c) 7

(d) 6

125.

What is the sum of all even numbers between 1 and 100 (both included)?

(a) 2450

(b) 2500

(c) 2600

(d) 2550

126.

The least number which can be added to 763 so that it is completely divisible by 57 is:

(a) 35

(b) 22

(c) 15

(d) 25

127.

The least number which can be subtracted from 763 so that it is completely divisible by 57 is:

(a) 35

(b) 22

(c) 15

(d) 25

128.

The least number which can be added to 8441 so that it is completely divisible by 57 is?

(a) 42

(b) 15

(c) 5

(d) 52

129.

The least number which can be subtracted from 8441 so that it is completely divisible by 57 is:

(a) 3

(b) 4

(c) 5

(d) 6

130.

Find the least number of 5 digits that is exactly divisible by 79

(a) 10003

(b) 10033

(c) 10043

(d) None of these

131.

Find the maximum number of 5 digits that is exactly divisible by 79:

(a) 99925

(b) 99935

(c) 99945

(d) 99955

132.

The nearest integer to 773 which is exactly divisible by 12 is:

(a) 768

(b) 772

(c) 776

(d) None of these

133.

A number when divided by 84 leaves a remainder of 57. What is the remainder when the same number is divided by 12?

(a) 7

(b) 8

(c) 9

(d) Cannot be determined

134.

A number when divided by 84 leaves a remainder of 57. What is the remainder when the same number is divided by 11?

(a) 2

(b) 7

(c) 8

(d) Cannot be determined

135.

511 and 667 when divided by the same number, leave the same remainder. How many numbers can be used as the divisor in order to make this occur?

(a) 14

(b) 12

(c) 10

(d) 8

136.

How many numbers between 200 and 400 are divisible by 13?

(a) 14

(b) 15

(c) 16

(d) 17

137.

A boy was trying to find 5/8th of a number. Unfortunately, he found out 8/5th of the number and realized that the difference between the answer he got and the correct answer is 39. What was the number?

(a) 38

(b) 39

(c) 40

(d) 52

138.

The sum of two numbers is equal to thrice their difference. If the smaller of the numbers is 10 find the other number.

(a) 15

(b) 20

(c) 40

(d) None of these

139.

411 + 412 + 413 + 414 + 415 is divisible by which of the following?

(a) 11

(b) 31

(c) 341

(d) All of the above

140.

The product of two numbers is 7168 and their HCF is 16. How many pairs of numbers are possible such that the above conditions are satisfied?

(a) 2

(b) 3

(c) 4

(d) 6

LEVEL OF DIFFICULTY (II)

1.

The arithmetic mean of two numbers is smaller by 24 than the larger of the two numbers and the GM of the same numbers exceeds by 12 the smaller of the numbers. Find the numbers.

(a) 6 and 54

(b) 8 and 56

(c) 12 and 60

(d) 7 and 55

2.

Find the number of numbers between 200 and 300, both included, which are not divisible by 2, 3, 4 and 5.

(a) 27

(b) 26

(c) 25

(d) 28

3.

Given x and n are integers, (15n3 + 6n2 + 5n + x)/n is not an integer for what condition?

(a) n is positive

(b) x is divisible by n

(c) x is not divisible by n

(d) (a) and (c)

4.

The unit digit in the expression 36234*33512*39180 – 5429*25123*31512 will be

(a) 8

(b) 0

(c) 6

(d) 5

5.

The difference of 1025 – 7 and 1024 + x is divisible by 3 for x = ?

(a) 3

(b) 2

(c) 4

(d) 6

6.

Find the value of x in = x.

(a) 1

(b) 3

(c) 6

(d) 12

(e) 9

7.

If a number is multiplied by 22 and the same number is added to it, then we get a number that is half the square of that number. Find the number

(a) 45

(b) 46

(c) 47

(d) data insufficient

8.

1255/311 + 848/1618 will give the digit at units place as

(a) 4

(b) 6

(c) 8

(d) 0

9.

The mean of 1, 2, 22…. 231 lies in between

(a) 224 to 225

(b) 225 to 226

(c) 226 to 227

(d) 229 to 230

10.

xy is a number that is divided by ab where xy < ab and gives a result 0.xyxyxy… then ab equals

(a) 11

(b) 33

(c) 99

(d) 66

11.

A number xy is multiplied by another number ab and the result comes as pqr, where r = 2y, q = 2(x + y) and p = 2x where x, y < 5, q π 0. The value of ab may be:

(a) 11

(b) 13

(c) 31

(d) 22

12.

[x] denotes the greatest integer value just below x and {x} its fractional value. The sum of [x]3 and {x}2 is –7.91. Find x.

(a) –2.03

(b) –1.97

(c) –2.97

(d) –1.7

13.

165 + 215 is divisible by

(a) 31

(b) 13

(c) 27

(d) 33

14.

If AB + XY = 1XP, where A π 0 and all the letters signify different digits from 0 to 9, then the value of A is:

(a) 6

(b) 7

(c) 9

(d) 8

Directions for questions 15 and 16: Find the possible integral values of x.

15.

|x – 3| + 2|x + 1| = 4

(a) 1

(b) –1

(c) 3

(d) 2

16.

x2 + |x – 1| = 1

(a) 1

(b) –1

(c) 0

(d) 1 or 0

17.

If 4n + 1 + x and 42nx are divisible by 5, n being an even integer, find the least value of x.

(a) 1

(b) 2

(c) 3

(d) 0

18.

If the sum of the numbers (a25)2 and a3 is divisible by 9, then which of the following may be a value for a ?

(a) 1

(b) 7

(c) 9

(d) There is no value

19.

If | x – 4 | + | y – 4 | = 4, then how many integer values can the set (x, y) have?

(a) Infinite

(b) 5

(c) 16

(d) 9

20.

[332/50] gives remainder and {.} denotes the fractional part of that. The fractional part is of the form (0 ◊ bx). The value of x could be

(a) 2

(b) 4

(c) 6

(d) 8

21.

The sum of two numbers is 20 and their geometric mean is 20% lower than their arithmetic mean. Find the ratio of the numbers.

(a) 4 : 1

(b) 9 : 1

(c) 1 : 1

(d) 17 : 3

22.

The highest power on 990 that will exactly divide 1090! is

(a) 101

(b) 100

(c) 108

(d) 109

23.

If 146! is divisible by 6n, then find the maximum value of n.

(a) 74

(b) 70

(c) 76

(d) 75

24.

The last two digits in the multiplication of 35 34 33 32 31 30 29 28 27 26 is

(a) 00

(b) 40

(c) 30

(d) 10

25.

The expression 333555 + 555333 is divisible by

(a) 2

(b) 3

(c) 37

(d) All of these

26.

[x] denotes the greatest integer value just below x and {x} its fractional value. The sum of [x]2 and {x}1 is 25.16. Find x.

(a) 5.16

(b) –4.84

(c) Both (a) and (b)

(d) 4.84

27.

If we add the square of the digit in the tens place of a positive two-digit number to the product of the digits of that number, we shall get 52, and if we add the square of the digit in the units place to the same product of the digits, we shall get 117. Find the two-digit number.

(a) 18

(b) 39

(c) 49

(d) 28

28.

Find two numbers such that their sum, their product and the differences of their squares are equal.

(a) and or and

(b) and or and

(c) and or and

(d) All of these

29.

The sum of the digits of a three-digit number is 17, and the sum of the squares of its digits is 109. If we subtract 495 from that number, we shall get a number consisting of the same digits written in the reverse order. Find the number.

(a) 773

(b) 863

(c) 683

(d) 944

30.

Find the number of zeros in the product: 11 × 22 × 33 × 44 × ...... 9898 × 9999 × 100100

(a) 1200

(b) 1300

(c) 1050

(d) 1225

31.

Find the pairs of natural numbers whose greatest common divisor is 5 and the least common multiple is 105.

(a) 5 and 105 or 15 and 35

(b) 6 and 105 or 16 and 35

(c) 5 and 15 or 15 and 135

(d) 5 and 20 or 15 and 35

32.

The denominator of an irreducible fraction is greater than the numerator by 2. If we reduce the numerator of the reciprocal fraction by 3 and subtract the given fraction from the resulting one, we get 1/15. Find the given fraction.

(a)

(b)

(c)

(d)

33.

A two-digit number exceeds by 19 the sum of the squares of its digits and by 44 the double product of its digits. Find the number.

(a) 72

(b) 62

(c) 22

(d) 12

34.

The sum of the squares of the digits constituting a two-digit positive number is 2.5 times as large as the sum of its digits and is larger by unity than the trebled product of its digits. Find the number.

(a) 13 and 31

(b) 12 and 21

(c) 22 and 33

(d) 14 and 41

35.

The units digit of a two-digit number is greater than its tens digit by 2, and the product of that number by the sum of its digits is 144. Find the number.

(a) 14

(b) 24

(c) 46

(d) 35

36.

Find the number of zeroes in the product: 5 × 10 × 25 × 40 × 50 × 55 × 65 × 125 × 80

(a) 8

(b) 9

(c) 12

(d) 13

37.

The highest power of 45 that will exactly divide 123! is

(a) 28

(b) 30

(c) 31

(d) 59

38.

Three numbers are such that the second is as much lesser than the third as the first is lesser than the second. If the product of the two smaller numbers is 85 and the product of two larger numbers is 115 find the middle number.

(a) 9

(b) 8

(c) 12

(d) 10

39.

Find the smallest natural number n such that n! is divisible by 990.

(a) 3

(b) 5

(c) 11

(d) 12

40.

is true only when

(a) x > 0, y > 0

(b) x > 0 and y < 0

(c) x < 0 and y > 0

(d) All of these

Directions for Questions 41 to 60: Read the instructions below and solve the questions based on this.

In an examination situation, always solve the following type of questions by substituting the given options, to arrive at the solution.

However, as you can see, there are no options given in the questions here since these are meant to be an exercise in equation writing (which I believe is a primary skill required to do well in aptitude exams testing mathematical aptitude). Indeed, if these questions had options for them, they would be rated as LOD 1 questions. But since the option-based solution technique is removed here, I have placed these in the LOD 2 category.

41.

Find the two-digit number that meets the following criteria. If the number in the units place exceeds, the number in its tens by 2 and the product of the required number with the sum of its digits is equal to 144.

42.

The product of the digits of a two-digit number is twice as large as the sum of its digits. If we subtract 27 from the required number, we get a number consisting of the same digits written in the reverse order. Find the number?

43.

The product of the digits of a two-digit number is one-third that number. If we add 18 to the required number, we get a number consisting of the same digits written in the reverse order. Find the number?

44.

The sum of the squares of the digits of a two-digit number is 13. If we subtract 9 from that number, we get a number consisting of the same digits written in the reverse order. Find the number?

45.

A two-digit number is thrice as large as the sum of its digits, and the square of that sum is equal to the trebled required number. Find the number?

46.

Find a two-digit number that exceeds by 12 the sum of the squares of its digits and by 16 the doubled product of its digits.

47.

The sum of the squares of the digits constituting a two-digit number is 10, and the product of the required number by the number consisting of the same digits written in the reverse order is 403. Find the 2 numbers that satisfy these conditions?

48.

If we divide a two-digit number by the sum of its digits, we get 4 as a quotient and 3 as a remainder. Now, if we divide that two-digit number by the product of its digits, we get 3 as a quotient and 5 as a remainder. Find the two-digit number.

49.

There is a natural number that becomes equal to the square of a natural number when 100 is added to it, and to the square of another natural number when 169 is added to it. Find the number?

50.

Find two natural numbers whose sum is 85 and whose least common multiple is 102.

51.

Find two-three digit numbers whose sum is a multiple of 504 and the quotient is a multiple of 6.

52.

The difference between the digits in a two-digit number is equal to 2, and the sum of the squares of the same digits is 52. Find all the possible numbers?

53.

If we divide a given two-digit number by the product of its digits, we obtain 3 as a quotient and 9 as a remainder. If we subtract the product of the digits constituting the number, from the square of the sum of its digits, we obtain the given number. Find the number.

54.

Find the three-digit number if it is known that the sum of its digits is 17 and the sum of the squares of its digits is 109. If we subtract 495 from this number, we obtain a number consisting of the same digits written in reverse order.

55.

The sum of the cubes of the digits constituting a two-digit number is 243 and the product of the sum of its digits by the product of its digits is 162. Find the two two-digit number?

56.

The difference between two numbers is 16. What can be said about the total numbers divisible by 7 that can lie in between these two numbers.

57.

Arrange the following in descending order:

1114, 110.109.108.107, 109.110.112.113

58.

If 3 £ x £ 5 and 4 £ y £ 7. Find the greatest value of xy and the least value of x/y.

59.

Which of these is greater:

(a) 200300 or 300200 or 400150

(b) 5100 and 2200

(c) 1020 and 4010

60.

The sum of the two numbers is equal to 15 and their arithmetic mean is 25 per cent greater than its geometric mean. Find the numbers.

61.

Define a number K such that it is the sum of the squares of the first M natural numbers.(i.e. K = 12 + 22 +….+ M2) where M < 55. How many values of M exist such that K is divisible by 4?

(a) 10

(b) 11

(c) 12

(d) None of these

62.

M is a two digit number which has the property that:

The product of factorials of it’s digits > sum of factorials of its digits

How many values of M exist?

(a) 56

(b) 64

(c) 63

(d) None of these

63.

A natural number when increased by 50% has its number of factors unchanged. However, when the value of the number is reduced by 75%, the number of factors is reduced by 66.66%. One such number could be:

(a) 32

(b) 84

(c) 126

(d) None of these

64.

Find the 28383rd term of the series: 123456789101112….

(a) 3

(b) 4

(c) 9

(d) 7

65.

If you form a subset of integers chosen from between 1 to 3000, such that no two integers add up to a multiple of nine, what can be the maximum number of elements in the subset. (Include both 1 and 3000.)

(a) 1668

(b) 1332

(c) 1333

(d) 1336

66.

The series of numbers (1,1/2,1/3,1/4 …………….1/1972) is taken. Now two numbers are taken from this series (the first two) say x, y. Then the operation x + y + x.y is performed to get a consolidated number. The process is repeated. What will be the value of the set after all the numbers are consolidated into one number.

(a) 1970

(b) 1971

(c) 1972

(d) None of these

67.

K is a three digit number such that the ratio of the number to the sum of its digits is least. What is the difference between the hundreds and the tens digits of K?

(a) 9

(b) 8

(c) 7

(d) None of these

68.

In Question 67, what can be said about the difference between the tens and the units digit?

(a) 0

(b) 1

(c) 2

(d) None of these

69.

For the above question, for how many values of K will the ratio be the highest?

(a) 9

(b) 8

(c) 7

(d) None of these

70.

A triangular number is defined as a number which has the property of being expressed as a sum of consecutive natural numbers starting with 1. How many triangular numbers less than 1000, have the property that they are the difference of squares of two consecutive natural numbers?

(a) 20

(b) 21

(c) 22

(d) 23

71.

x and y are two positive integers. Then what will be the sum of the coefficients of the expansion of the expression (x + y)44? Answer: 244

(a) 243

(b) 243 + 1

(c) 244

(d) 244 – 1

72.

What is the remainder when 9 + 92 + 93 + ….92n+1 is divided by 6?

(a) 1

(b) 2

(c) 3

(d) 4

73.

The remainder when the number 123456789101112 ……484950 is divided by 16 is:

(a) 3

(b) 4

(c) 5

(d) 6

74.

What is the highest power of 3 available in the expression 58! – 38!

(a) 17

(b) 18

(c) 19

(d) None of these

75.

Find the remainder when the number represented by 22334 raised to the power (12 + 22 + ..+ 662) is divided by 5?

(a) 2

(b) 4

(c) 1

(d) None of these

76.

What is the total number of divisors of the number 1233 × 3423 × 270 ?

(a) 4658.

(b) 9316

(c) 2744

(d) None of these

77.

For Question 76, which of the following will represent the sum of factors of the number (such that only odd factors are counted)?

(a)

(b) (334–1) × (1724 – 1)

(c)

(d) None of these

78.

What is the remainder when (1!)3 + (2!) 3 + (3!) 3 + (4!)3 +…..(1152!)3 is divided by 1152?

(a) 125

(b) 225

(c) 325

(d) 205

79.

A set S is formed by including some of the first One thousand natural numbers. S contains the maximum number of numbers such that they satisfy the following conditions:

1.No number of the set S is prime.

2.When the numbers of the set S are selected two at a time, we always see co prime numbers

What is the number of elements in the set S?

(a) 11

(b) 12

(c) 13

(d) 7

Find the last two digits of the following numbers

80.

101 × 102 × 103 × 197 × 198 × 199

(a) 54

(b) 74

(c) 64

(d) 84

81.

65 × 29 × 37 × 63 × 71 × 87

(a) 05

(b) 95

(c) 15

(d) 25

82.

65 × 29 × 37 × 63 × 71 × 87 × 85

(a) 25

(b) 35

(c) 75

(d) 85

83.

65 × 29 × 37 × 63 × 71 × 87 × 62

(a) 70

(b) 30

(c) 10

(d) 90

84.

75 × 35 × 47 × 63 × 71 × 87 × 82

(a) 50

(b) 70

(c) 30

(d) 90

85.

(201 × 202 × 203 × 204 × 246 × 247 × 248 × 249)2

(a) 36

(b) 56

(c) 76

(d) 16

86.

Find the remainder when 799 is divided by 2400.

(a) 1

(b) 343

(c) 49

(d) 7

87.

Find the remainder when (103 + 93)752 is divided by 123.

(a) 729

(b) 1000

(c) 752

(d) 1

88.

Arun, Bikas and Chetakar have a total of 80 coins among them. Arun triples the number of coins with the others by giving them some coins from his own collection. Next, Bikas repeats the same process. After this Bikas now has 20 coins. Find the number of coins he had at the beginning?

(a) 22

(b) 20

(c) 18

(d) 24

89.

The super computer at Ram Mohan Roy Seminary takes an input of a number N and a X where X is a factor of the number N. In a particular case N is equal to 83p 796161q and X is equal to 11 where 0 < p < q, find the sum of remainders when N is divided by (p + q) and p successively.

(a) 6

(b) 3

(c) 2

(d) 9

90.

On March 1st 2016, Sherry saved Re.1. Everyday starting from March 2nd 2016, he saved Re.1 more than the previous day. Find the first date after March 1st 2016 at the end of which his total savings will be a perfect square.

(a) 17th March 2016

(b) 18th April 2016

(c) 26th March 2016

(d) None of these

91.

What is the rightmost digit preceding the zeroes in the value of 2053?

(a) 2

(b) 8

(c) 1

(d) 4

92.

What is the remainder when 2(8!) – 21(6!) divides 14(7!) + 14(13!)?

(a) 1

(b) 7!

(c) 8!

(d) 9!

93.

How many integer values of x and y are there such that 4x + 7y = 3, while |x| < 500 and |y| < 500?

(a) 144

(b) 141

(c) 143

(d) 142

94.

If n = 1 + m, where m is the product of four consecutive positive integers, then which of the following is/are true?

(A) n is odd

(B) n is not a multiple of 3

(C) n is a perfect square

(a) All three

(b) A and B only

(c) A and C only

(d) None of these

95.

How many two-digit numbers less than or equal to 50, have the product of the factorials of their digits less than or equal to the sum of the factorials of their digits?

(a) 18

(b) 16

(c) 15

(d) None of these

96.

A candidate takes a test and attempts all the 100 questions in it. While any correct answer fetches 1 mark, wrong answers are penalised as follows; one-tenth of the questions carry 1/10 negative mark each, one-fifth of the questions carry 1/5 negative marks each and the rest of the questions carry ½ negative mark each. Unattempted questions carry no marks. What is the difference between the maximum and the minimum marks that he can score?

(a) 100

(b) 120

(c) 140

(d) None of these

Directions for Questions 97 to 99: A mock test is taken at Mindworkzz. The test paper comprises of questions in three levels of difficulty—LOD1, LOD2 and LOD 3.

The following table gives the details of the positive and negative marks attached to each question type:

Difficulty level

Positive marks for answering the question correctly

Negative marks for answering the question wrongly

LOD 1

4

2

LOD 2

3

1.5

LOD 3

2

1

The test had 200 questions with 80 on LOD 1 and 60 each on LOD 2 and LOD 3.

97.

If a student has solved 100 questions exactly and scored 120 marks, the maximum number of incorrect questions that he/she might have marked is:

(a) 44

(b) 56

(c) 60

(d) None of these

98.

If Amit attempted the least number of questions and got a total of 130 marks, and if it is known that he attempted at least one of every type, then the number of questions he must have attempted is:

(a) 34

(b) 35

(c) 36

(d) None of these

99.

In the above question, what is the least number of questions he might have got incorrect?

(a) 0

(b) 1

(c) 2

(d) None of these

100.

Amitabh has a certain number of toffees, such that if he distributes them amongst ten children he has nine left, if he distributes amongst 9 children he would have 8 left, if he distributes amongst 8 children he would have 7 left … and so on until if he distributes amongst 5 children he should have 4 left. What is the second lowest number of toffees he could have with him?

(a) 2519

(b) 7559

(c) 8249

(d) 5039

LEVEL OF DIFFICULTY (III)

1.

What two-digit number is less than the sum of the square of its digits by 11 and exceeds their doubled product by 5?

(a) 15, 95

(b) 95

(c) Both (a) and (b)

(d) 15, 95 and 12345

2.

Find the lower of the two successive natural numbers if the square of the sum of those numbers exceeds the sum of their squares by 112.

(a) 6

(b) 7

(c) 8

(d) 9

3.

First we increased the denominator of a positive fraction by 3 and then we decreased it by 5. The sum of the resulting fractions proves to be equal to 19/42. Find the denominator of the fraction if its numerator is 2.

(a) 7

(b) 8

(c) 12

(d) 9

4.

Find the last two digits of: 15 × 37 × 63 × 51 × 97 × 17.

(a) 35

(b) 45

(c) 55

(d) 85

5.

Let us consider a fraction whose denominator is smaller than the square of the numerator by unity. If we add 2 to the numerator and the denominator, the fraction will exceed 1/3. If we subtract 3 from the numerator and the denominator, the fraction will be positive but smaller than 1/10. Find the value.

(a)

(b)

(c)

(d)

6.

Find the sum of all three-digit numbers that give a remainder of 4 when they are divided by 5.

(a) 98,270

(b) 99,270

(c) 1,02,090

(d) 90,270

7.

Find the sum of all two-digit numbers that give a remainder of 3 when they are divided by 7.

(a) 686

(b) 676

(c) 666

(d) 656

8.

Find the sum of all odd three-digit numbers that are divisible by 5.

(a) 50,500

(b) 50,250

(c) 50,000

(d) 49,500

9.

The product of a two-digit number by a number consisting of the same digits written in the reverse order is equal to 2430. Find the lower number.

(a) 54

(b) 52

(c) 63

(d) 45

10.

Find the lowest of three numbers as described: If the cube of the first number exceeds their product by 2, the cube of the second number is smaller than their product by 3, and the cube of the third number exceeds their product by 3.

(a) 31/3

(b) 91/3

(c) 2

(d) Any of these

(e) None of these

11.

How many pairs of natural numbers are there the difference of whose squares is 45?

(a) 1

(b) 2

(c) 3

(d) 4

12.

Find all two-digit numbers such that the sum of the digits constituting the number is not less than 7; the sum of the squares of the digits is not greater than 30; the number consisting of the same digits written in the reverse order is not larger than half the given number.

(a) 52

(b) 51

(c) 49

(d) 53

13.

In a four-digit number, the sum of the digits in the thousands, hundreds and tens is equal to 14, and the sum of the digits in the units, tens and hundreds is equal to 15. Among all the numbers satisfying these conditions, find the number the sum of the squares of whose digits is the greatest.

(a) 2572

(b) 1863

(c) 2573

(d) None of these

14.

In a four-digit number, the sum of the digits in the thousands and tens is equal to 4, the sum of the digits in the hundreds and the units is 15, and the digit of the units exceeds by 7 the digit of the thousands. Among all the numbers satisfying these conditions, find the number the sum of the product of whose digit of the thousands by the digit of the units and the product of the digit of the hundreds by that of the tens assumes the least value.

(a) 4708

(b) 1738

(c) 2629

(d) 1812

15.

If we divide a two-digit number by a number consisting of the same digits written in the reverse order, we get 4 as a quotient and 15 as a remainder. If we subtract 1 from the given number, we get the sum of the squares of the digits constituting that number. Find the number.

(a) 71

(b) 83

(c) 99

(d) None of these

16.

Find the two-digit number the quotient of whose division by the product of its digits is equal to 8/3, and the difference between the required number and the number consisting of the same digits written in the reverse order is 18

(a) 86

(b) 42

(c) 75

(d) None of these

17.

Find the two-digit number if it is known that the ratio of the required number and the sum of its digits is 8 as also the quotient of the product of its digits and that of the sum is 14/9.

(a) 54

(b) 72

(c) 27

(d) 45

18.

If we divide the unknown two-digit number by the number consisting of the same digits written in the reverse order, we get 4 as a quotient and 3 as a remainder. If we divide the required number by the sum of its digits, we get 8 as a quotient and 7 as a remainder. Find the number.

(a) 81

(b) 91

(c) 71

(d) 72

19.

The last two-digits in the multiplication 122 × 123 × 125 × 127 × 129 will be

(a) 20

(b) 50

(c) 30

(d) 40

20.

The remainder obtained when 43101 + 23101 is divided by 66 is:

(a) 2

(b) 10

(c) 5

(d) 0

21.

The last three-digits of the multiplication 12345 × 54321 will be

(a) 865

(b) 745

(c) 845

(d) 945

22.

The sum of the digits of a three-digit number is 12. If we subtract 495 from the number consisting of the same digits written in reverse order, we shall get the required number. Find that three-digit number if the sum of all pairwise products of the digits constituting that number is 41.

(a) 156

(b) 237

(c) 197

(d) Both (a) and (b)

23.

A three-digit positive integer abc is such that a2 + b2 + c2 = 74. a is equal to the doubled sum of the digits in the tens and units places. Find the number if it is known that the difference between that number and the number written by the same digits in the reverse order is 495.

(a) 813

(b) 349

(c) 613

(d) 713

24.

Represent the number 1.25 as a product of three positive factors so that the product of the first factor by the square of the second is equal to 5 if we have to get the lowest possible sum of the three factors.

(a) x1 = 2.25, x2 = 5, x3 = 0.2

(b) x1 = 1.25, x2 = 4, x3 = 4.5

(c) x1 = 1.25, x2 = 2, x3 = 0.5

(d) x1 = 1.25, x2 = 4, x3 = 2

25.

Find a number x such that the sum of that number and its square is the least.

(a) –0.5

(b) 0.5

(c) –1.5

(d) 1.5

26.

When 22225555 + 55552222 is divided by 7, the remainder is

(a) 0

(b) 2

(c) 4

(d) 5

27.

If x is a number of five-digits which when divided by 8, 12, 15 and 20 leaves respectively 5, 9, 12 and 17 as remainders, then find x such that it is the lowest such number.

(a) 10017

(b) 10057

(c) 10097

(d) 10137

28.

32n – 1 is divisible by 2n + 3 for n =

(a) 1

(b) 2

(c) 3

(d) None of these

29.

10n – is divisible by 2n + 2 for what whole number value of n?

(a) 2

(b) 3

(c) 7

(d) None of these

30.

will leave a remainder:

(a) 4

(b) 7

(c) 1

(d) 2

31.

Find the remainder that the number 1989 1990 19923 gives when divided by 7.

(a) 0

(b) 1

(c) 5

(d) 2

32.

Find the remainder of 2100 when divided by 3.

(a) 3

(b) 0

(c) 1

(d) 2

33.

Find the remainder when the number 31989 is divided by 7.

(a) 1

(b) 5

(c) 6

(d) 4

34.

Find the last digit of the number 12 + 22 +…+ 992.

(a) 0

(b) 1

(c) 2

(d) 3

35.

Find gcd (2100 – 1, 2120 – 1).

(a) 220– 1

(b) 240 – 1

(c) 260 – 1

(d) 210–1

36.

Find the gcd (111…11 hundred ones ; 11…11 sixty ones).

(a) 111…forty ones

(b) 111…twenty five ones

(c) 111…twenty ones

(d) 111…sixty ones

37.

Find the last digit of the number 13 + 23+ 33 + 43… + 993.

(a) 0

(b) 1

(c) 2

(d) 5

38.

Find the GCD of the numbers 2n + 13 and n + 7.

(a) 1

(b) 2

(c) 3

(d) 4

39.

(a) 4

(b) 2

(c) 1

(d) 3

40.

The remainder when 1010 + 10100 + 101000 + … + 1010000000000 is divided by 7 is

(a) 0

(b) 1

(c) 2

(d) 5

41.

n is a number, such that 2n has 28 factors and 3n has 30 factors. 6n has.

(a) 35

(b) 32

(c) 28

(d) None of these

42.

Suppose the sum of n consecutive integers is x + (x + 1) + (x + 2) + (x + 3) + … + (x + (n – 1)) = 1000, then which of the following cannot be true about the number of terms n

(a) The number of terms can be 16

(b) The number of terms can be 5

(c) The number of terms can be 25

(d) The number of terms can be 20

43.

The remainder when 22 + 222 + 2222 + 22222 + …….(222…..49 twos)2 is divided by 9 is:

(a) 2

(b) 5

(c) 6

(d) 7

44.

N = 202 × 20002 × 200000002 × 20000000000000002 × 200000000….2 (31 zeroes) The sum of digits in this multiplication will be:

(a) 112

(b) 160

(c) 144

(d) Cannot be determined

45.

Twenty five sets of problems on Data Interpretation– one each for the DI sections of 25 CATALYST tests were prepared by the AMS research team. The DI section of each CATALYST contained 50 questions of which exactly 35 questions were unique, i.e. they had not been used in the DI section of any of the other 24 CATALYSTs. What could be the maximum possible number of questions prepared for the DI sections of all the 25 CATALYSTs put together?

(a) 1100

(b) 975

(c) 1070

(d) 1055

46.

In the above question, what could be the minimum possible number of questions prepared?

(a) 890

(b) 875

(c) 975

(d) None of these

Directions for Questions 47 to 49: At a particular time in the twenty first century there were seven bowlers in the Indian cricket team’s list of 16 players short listed to play the next world cup. Statisticians discovered that that if you looked at the number of wickets taken by any of the 7 bowlers of the current Indian cricket team, the number of wickets taken by them had a strange property. The numbers were such that for any team selection of 11 players (having 1 to 7 bowlers) by using the number of wickets taken by each bowler and attaching coefficients of +1, 0, or –1 to each value available and adding the resultant values, any number from 1 to 1093, both included could be formed. If we denote W1, W2, W3, W4, W5, W6 and W7 as the 7 values in the ascending order what could be the answer to the following questions:

47.

Find the value of W1 + 2W2 + 3W3 + 4W4 + 5W5 + 6W6.

(a) 2005

(b) 1995

(c) 1985

(d) None of these

48.

Find the index of the largest power of 3 contained in the product W1 W2 W3 W4 W5 W6 W7.

(a) 15

(b) 10

(c) 21

(d) 6

49.

If the sum of the seven coefficients is 0, find the smallest number that can be obtained.

(a) – 1067

(b) – 729

(c) – 1040

(d) – 1053

Directions for Questions 50 and 51: Answer these questions on the basis of the information given below.

In the ancient game of Honololo the task involves solving a puzzle asked by the chief of the tribe. Anybody answering the puzzle correctly is given the hand of the most beautiful maiden of the tribe. Unfortunately, for the youth of the tribe, solving the puzzle is not a cakewalk since the chief is the greatest mathematician of the tribe.

In one such competition the chief called everyone to attention and announced openly:

“A three-digit number ‘mnp’ is a perfect square and the number of factors it has is also a perfect square. It is also known that the digits m, n and p are all distinct. Now answer my questions and win the maiden’s hand.”

50.

If (m + n + p) is also a perfect square, what is the number of factors of the six-digit number mnpmnp?

(a) 32

(b) 72

(c) 48

(d) Cannot be determined

51.

If the fourth power of the product of the digits of the number mnp is not divisible by 5, what is the number of factors of the nine-digit number, mnpmnpmnp?

(a) 32

(b) 72

(c) 48

(d) Cannot be determined

52.

In a cricket tournament organised by the ICC, a total of 15 teams participated. Australia, as usual won the tournament by scoring the maximum number of points. The tournament is organised as a single round robin tournament—where each team plays with every other team exactly once. 3 points are awarded for a win, 2 points are awarded for a tie/washed out match and 1 point is awarded for a loss. Zimbabwe had the lowest score (in terms of points) at the end of the tournament. Zimbabwe scored a total of 21 points. All the 15 national teams got a distinct score (in terms of points scored). It is also known that at least one match played by the Australian team was tied/washed out. Which of the following is always true for the Australian team?

(a) It had at least two ties/washouts.

(b) It had a maximum of 3 losses.

(c) It had a maximum of 9 wins.

(d) All of the above.

53.

What is the remainder when 1281000 is divided by 153?

(a) 103

(b) 145

(c) 118

(d) 52

54.

Find the remainder when 505152 is divided by 11.

(a) 6

(b) 4

(c) 7

(d) 3

55.

Find the remainder when 323334 is divided by 11.

(a) 5

(b) 4

(c) 10

(d) 1

56.

Find the remainder when 307287 is divided by 11.

(a) 5

(b) 9

(c) 6

(d) 3

57.

Find the remainder when 505652 is divided by 11.

(a) 7

(b) 5

(c) 9

(d) 10

58.

Find the remainder when 333435 is divided by 7.

(a) 5

(b) 4

(c) 6

(d) 2

59.

Let Sm denote the sum of the squares of the first m natural numbers. For how many values of m < 100, is Sm a multiple of 4?

(a) 50

(b) 25

(c) 36

(d) 24

60.

For the above question, for how many values will the sum of cubes of the first m natural numbers be a multiple of 5 (if m < 50)?

(a) 20

(b) 21

(c) 22

(d) None of these

61.

How many integer values of x and y satisfy the expression 4x + 7y = 3 where |x| < 1000 and |y| < 1000?

(a) 284

(b) 285

(c) 286

(d) None of these

ANSWER KEY

Level of Difficulty (I)

1. (a)

2. (a)

3. (b)

4. (b)

5. (a)

6. (c)

7. (d)

8. (d)

9. (d)

10. (d)

11. (a)

12. (c)

13. (d)

14. (d)

15. (b)

16. (c)

17. (b)

18. (d)

19. (a)

20. (b)

21. (d)

22. (a)

23. (b)

24. (c)

25. (d)

26. (d)

27. (c)

28. (d)

29. (b)

30. (d)

31. (a)

32. (b)

33. (c)

34. (d)

35. (c)

36. (b)

37. (d)

38. (d)

39. (b)

40. (b)

41. (a)

42. (a)

43. (a)

44. (d)

45. (c)

46. (d)

47. (b)

48. (c)

49. (c)

50. (d)

51. (a)

52. (a)

53. (b)

54. (c)

55. (d)

56. (b)

57. (a)

58. (b)

59. (d)

60. (d)

61. (a)

62. (a) Æ (21)12

(b) Æ (0.8)3

63. (b)

64.

1. Æ 0, 2, 4, 6, 8

2. Æ 1, 4, 7

3. Æ 0, 4, 8

4. Æ 0, 5

5. Æ 4

6. Æ 7

7. Æ 0

65.

LCM Æ 17010

HCF Æ 27

LCM Æ 780

HCF Æ 29

LCM Æ 245700

HCF Æ 30

66. (b)

67. (a)

68. (c)

69. (c)

70. (c)

71. (a)

72. (d)

73. (b)

74. (b)

75. (a)

76. (b)

77. (d)

78. (a)

79. (a)

80. (b)

81. (c)

82. (b)

83. (c)

84. (c)

85. (c)

86. (a)

87. (c)

88. (b)

89. (b)

90. (c)

91. (d)

92. (d)

93. (a)

94. (c)

95. (c)

96. (b)

97. (a)

98. (c)

99. (c)

100. (d)

101. (d)

102. (a)

103. (c)

104. (b)

105. (c)

106. (d)

107. (b)

108. (c)

109. (d)

110. (c)

111. (c)

112. (c)

113. (a)

114. (c)

115. (a)

116. (d)

117. (d)

118. (b)

119. (b)

120. (c)

121. (b)

122. (b)

123. (d)

124. (a)

125. (d)

126. (a)

127. (b)

128. (d)

129. (c)

130. (b)

131. (b)

132. (a)

133. (c)

134. (d)

135. (b)

136. (b)

137. (c)

138. (b)

139. (d)

140. (a)

Level of Difficulty (II)

1. (a)

2. (b)

3. (c)

4. (b)

5. (b)

6. (b)

7. (b)

8. (d)

9. (c)

10. (c)

11. (d)

12. (d)

13. (d)

14. (c)

15. (b)

16. (d)

17. (a)

18. (d)

19. (c)

20. (a)

21. (a)

22. (c)

23. (b)

24. (a)

25. (d)

26. (c)

27. (c)

28. (d)

29. (b)

30. (b)

31. (a)

32. (b)

33. (a)

34. (a)

35. (b)

36. (b)

37. (a)

38. (d)

39. (c)

40. (a)

41. (24)

42. (63)

43. (24)

44. (32)

45. (27)

46. (64)

47. (13, 31)

48. (23)

49. (1056)

50. (51, 34)

51. (144, 864)

52. (46, 64)

53. (63)

54. (863)

55. (36, 63)

56.

may be 2 or 3 depending upon the numbers

57.

1114 > 109.110.112.113 > 110.109.108.107

58.

greatest Æ 35 least 3/7

59. (a) 200300

(b) 5100

(c) 1020

60. (12, 3)

61. (c)

62. (c)

63. (b)

64. (a)

65. (d)

66. (c)

67. (b)

68. (a)

69. (a)

70. (b)

71. (c)

72. (c)

73. (d)

74. (a)

75. (b)

76. (d)

77. (a)

78. (b)

79. (b)

80. (c)

81. (b)

82. (c)

83. (d)

84. (a)

85. (c)

86. (b)

87. (d)

88. (b)

89. (d)

90. (d)

91. (a)

92. (b)

93. (c)

94. (a)

95. (a)

96. (c)

97. (b)

98. (a)

99. (a)

100. (d)

Level of Difficulty (III)

1. (a)

2. (b)

3. (d)

4. (a)

5. (b)

6. (b)

7. (b)

8. (d)

9. (d)

10. (a)

11. (c)

12. (a)

13. (d)

14. (b)

15. (d)

16. (d)

17. (b)

18. (c)

19. (b)

20. (d)

21. (b)

22. (d)

23. (a)

24. (c)

25. (a)

26. (a)

27. (d)

28. (d)

29. (d)

30. (a)

31. (d)

32. (c)

33. (c)

34. (a)

35. (a)

36. (c)

37. (a)

38. (a)

39. (a)

40. (d)

41. (a)

42. (d)

43. (c)

44. (b)

45. (d)

46. (a)

47. (a)

48. (c)

49. (c)

50. (d)

51. (d)

52. (b)

53. (d)

54. (a)

55. (c)

56. (a)

57. (b)

58. (d)

59. (d)

60. (d)

61. (b)

Solutions and Shortcuts

Level of Difficulty (I)

1.The units digit in this case would obviously be ‘0’ because the given expression has a pair of 2 and 5 in it’s prime factors.

2.When you read the sentence “when the digits are reversed, the number decreases by 54, you should automatically get two reactions going in your mind.

(i)The difference between the digits would be 54/9 = 6.

(ii)Since the number ‘decreases’- the tens digit of the number would be larger than the units digit.

Also, since we know that the sum of the digits is 10, we get that the digits must be 8 and 2 and the number must be 82. Thus, the changed number is 28.

3.The two numbers should be factors of 405. A factor search will yield the factors. (look only for 2 digit factors of 405 with sum of digits between 1 to 19).

Also 405 = 5 × 34. Hence: 15 × 27

45 × 9 are the only two options.

From these factors pairs only the second pair gives us the desired result.

i.e. Number × sum of digits = 405.

Hence, the answer is 45.

4.You can solve this question by using options. It can be seen that Option (b) 12,3 fits the situation perfectly as their Arithmetic mean = 7.5 and their geometric mean = 6 and the geometric mean is 20% less than the arithmetic mean

5.Two more than half of 1/3rd of 96 = 18. Also since we are given that the difference between the AM and GM is 18, it means that the GM must be an integer. From amongst the options, only option (a) gives us a GM which is an integer. Thus, checking for option 1, we get the GM=7 and AM=18.

6.For the number A381 to be divisible by 11, the sum of the even placed digits and the odds placed digits should be either 0 or a multiple of 11. This means that (A + 8) – (3 + 1) should be a multiple of 11 – as it is not possible to make it zero. Thus, the smallest value that A can take (and in fact the only value it can take) is 7. Option (c) is correct.

7.For 381A to be divisible by 9, the sum of the digits 3 + 8 + 1 + A should be divisible by 9. For that to happen A should be 6. Option (d) is correct.

8.96 when divided by 8, would give a remainder of 1. Hence, the required answer would be 2.

9.LCM of 5, 15 and 20 = 60. HCF of 5, 15 and 20 = 5. The required ratio is 60:5 = 12:1

10.LCM of 5/2, 8/9 and 11/14 would be given by: (LCM of numerators)/(HCF of denominators)

= 440/1 = 440

11.Only the first option can be verified to be true in this case. If A is even, 3A would always be divisible by 6 as it would be divisible by both 2 and 3. Options b and c can be seen to be incorrect by assuming the value of A as 4.

12.The essence of this question is in the fact that the last digit of the number is 0. Naturally, the number is necessarily divisible by 2,5 and 10. Only 4 does not necessarily divide it.

13.B would necessarily be even- as the possible values of B for the three digit number 15B to be divisible by 6 are 0 and 6. Also, the condition stated in option (c) is also seen to be true in this case – as both 0 and 6 are divisible by 6. Thus, option (d) is correct.

14.For the GCD take the least powers of all common prime factors.

Thus, the required answer would be 23 × 3

15.The units digit would be given by 5 + 6 + 9 (numbers ending in 5 and 6 would always end in 5 and 6 irrespective of the power and 354 will give a units digit equivalent to 34n+2 which would give us a unit digit of 32 i.e.9)

16.The respective units digits for the three parts of the expression would be:

5 + 9 + 2 =16 Æ required answer is 6. Option (c) is correct.

17.The respective units digits for the six parts of the expression would be:

1 + 4 + 7 + 6 + 5 + 6 =29 Æ required answer is 9. Option (b) is correct.

18.The respective units digits for the six parts of the expression would be:

1 × 4 × 7 × 6 × 5 × 6 Æ required answer is 0. Option (d) is correct.

19.The number of zeroes would be given by adding the quotients when we successively divide 1090 by 5:

1090/5 + 218/5 + 43/5 + 8/5 = 218 + 43 + 8 + 1 = 270. Option (a) is correct.

20.The number of 5’s in 146! can be got by [146/5] + [29/5] + [5/5]= 29+5+1 = 35

21.1420 = 142 × 10 = 22 × 711 × 51.

Thus, the number of factors of the number would be (2 + 1) (1 + 1) (1 + 1) = 3 × 2 × 2 = 12.

Option (d) is correct.

22.(x2 – 5x + 6) = (x – 2)(x – 3)

& (x2 – 7x + 10) = (x – 5)(x – 2)

Required HCF = (x – 2); required LCM = (x – 2)(x – 3)(x – 5).

Option (a) is correct.

23.Since both P and Q are prime numbers, the number of factors would be (1 + 1)(1 + 1) = 4.

24.Since both P and Q are prime numbers, the number of factors would be (2 + 1)(1 + 1) = 6.

25.Since both P and Q are prime numbers, the number of factors would be (3 + 1)(2 + 1) = 12.

26.The sides of the pentagon being 1422, 1737, 2160, 2214 and 2358, the least difference between any two numbers is 54. Hence, the correct answer will be a factor of 54.

Further, since there are some odd numbers in the list, the answer should be an odd factor of 54.

Hence, check with 27, 9 and 3 in that order. You will get 9 as the HCF.

27.The HCF of 576 and 448 is 32. Hence, each section should have 32 children. The number of sections would be given by: 576/32 + 448/32 = 18 + 14 = 32. Option (b) is correct.

28.The HCF of the given numbers is 31 and hence the number of bottles required would be:

403/31 + 465/31 + 496/31 = 13 + 15 + 16 = 44. Option (d) is correct.

29.The LCM of the 4 numbers is 612. The highest 4 digit number which would be a common multiple of all these 4 numbers is 9792. Hence, the correct answer is 9793.

30.The LCM of 16, 18 and 20 is 720. The numbers which would give a remainder of 4, when divided by 16, 18 and 20 would be given by the series:

724, 1444, 2164, 2884 and so on. Checking each of these numbers for divisibility by 7, it can be seen that 2884 is the least number in the series that is divisible by 7 and hence is the correct answer. Option (d) is correct.

31.They will ring together again after a time which would be the LCM of 6, 8, 12 and 18. The required LCM = 72. Hence, they would ring together after 72 seconds. Option (a) is correct.

32.720/72 = 10 times. Option (b) is correct.

33.5 × 7 × 6 = 0. Option (c) is correct.

34.All these numbers can be verified to not be perfect squares. Option (d) is correct.

35.A perfect square can never end in an odd number of zeroes. Option (c) is correct.

36.It is obvious that the LCM of 5,12,18 and 20 would never be a multiple of 9. At the same time it has to be a multiple of each of 3, 8 and 5. Option (b) is correct.

37.720 = 24 × 32 × 51 . Number of factors = 5 × 3 × 2 = 30. Option (d) is correct.

38.16 – x2 = (4 – x) (4 + x) and x2 + x – 6 = (x + 3)(x – 2)

The required LCM = (4 – x)(4 + x) (x + 3) (x – 2).

Option (d) is correct.

39.x2 – 4 = (x – 2) (x + 2) and x2 + x – 6 = (x + 3)(x – 2)

GCD or HCF of these expressions = (x – 2).

Option (b) is correct.

40.If A is not divisible by 3, it is obvious that 2A would also not be divisible by 3, as 2A would have no ‘3’ in it.

41.9100/8 = (8 + 1)100/8 Æ Since this is of the form (a + 1)n/a, the Remainder = 1. Option (a) is correct.

42.21000 /3 is of the form (a)EVEN POWER/(a + 1). The remainder = 1 in this case as the power is even. Option (a) is correct.

43.The condition for the product to be the greatest is if the two terms are equal. Thus, the break up in option (a) would give us the highest product of the two parts. Option (a) is correct.

44.50/5 =10, 10/5 =2.

Thus, the required answer would be 10 + 2 = 12. Option (d) is correct.

45.Checking each of the options it can be seen that the value in option (c)[viz: 1362480]is divisible by 24.

46.Any number divisible by 88, has to be necessarily divisible by 11, 2, 4, 8, 44 and 22. Thus, each of the first three options is correct.

47.10800 = 108 × 100 = 33 × 24 × 52 .

The number of divisors would be: (3 + 1) (4 + 1) (2 + 1) = 4 × 5 × 3 = 60 divisors. Option (b) is correct.

48.The GCD (also known as HCF) would be got by multiplying the least powers of all common factors of the two polynomials. The common factors are (x + 3) – least power 1, and (x + 1) – least power 2. Thus, the answer would be (x + 3) (x + 1)2. Option (c) is correct.

49.For the LCM of polynomials write down the highest powers of all available factors of all the polynomials.

The correct answer would be (x + 3) (3x + 4) (4x2 - 1)

50.Three consecutive natural numbers, starting with an even number would always have at least three 2’s as their prime factors and also would have at least one multiple of 3 in them. Thus, 6, 12 and 24 would each divide the product.

51.When the birds sat one on a branch, there was one extra bird. When they sat 2 to a branch one branch was extra.

To find the number of branches, go through options. Checking option (a), if there were 3 branches, there would be 4 birds. (this would leave one bird without branch as per the question.)

When 4 birds would sit 2 to a branch there would be 1 branch free (as per the question). Hence, the answer (a) is correct.

52.The number would either be (3n + 1)2 or (3n + 2)2. In the expansion of each of these the only term which would not be divisible by 3 would be the square of 1 and 2 respectively. When divided by 3, both of these give 1 as remainder.

53.The given expression can be written as:

53 × 33 × 32 × 72/52 × 72 × 34 = 53 × 35 × 72/52 × 72 × 34 = 15. Option (b) is correct.

54.D = 1.44, C = 0.09, B = 0.16, while the value of A is negative.

Thus, D > B > C > A is the required order. Option (c) is correct.

55.The upper limit for x + y = 4 + 3 = 7. The lower limit of xy = 2 – 3 = –1. Required ratio = 7/–1 = –7.

Option (d) is correct.

56.For the sum of squares of digits to be 13, it is obvious that the digits should be 2 and 3. So the number can only be 23 or 32. Further, the number being referred to has to be 32 since the reduction of 9, reverses the digits.

57.trying the value in the options you get that the product of 54 × 45 = 2430. Option (a) is correct.

58.Option (b) can be verified to be true as the LCM of 90 and 24 is indeed 360.

59.The pairs given in option (d) 78 and 13 and 26 and 39 meet both the conditions of LCM of 78 and HCF of 13. Option (d) is correct.

60.Solve using options. Option (d) 51 and 34 satisfies the required conditions.

61.282 – 272 = 55 and so also 82 – 32 = 55. Option (a) is correct.

62.(a)2112 = (213)4

Since 213 > 54, 2112 > 544.

(b)(0.4)4 = (4/10)4 = 1024/10000 = 0.1024.

(0.8)3 = (8/10)3 = 512/1000 = 0.512

Hence, (0.8)3 > (0.4)4.

63.This is never possible.

64.1.c = 0, 2, 4, 6 or 8 would make 38c as even and hence divisible by 2.

2.c = 1, 4 or 7 are possible values to make 38c divisible by 3.

3.c = 0, 4 or 8 would make the number end in 80, 84 or 88 and would hence be divisible by 4.

4.c = 0 or 5 would make the number 380 or 385 – in which case it would be divisible by 5.

5.For the number to become divisible by 6, it should be even and divisible by 3. From the values 1, 4 and 7 which make the number divisible by 3, we only have c = 4 making it even. Thus, c = 4.

6.For the number to be divisible by 9, 3 + 8 + c should be a multiple of 9. c = 7 is the only value of c which can make the number divisible by 9.

7.Obviously c = 0 is the correct answer.

65.Use the standard process to solve for LCM and HCF.

66.For 34x43 to be ending in 7, x has to be 3 (as 43 = 4n + 3). Option (b) is correct.

Solutions for 67 & 68:

The given condition says that Pen

Also, since the least cost of the three is `12, if we allocate a minimum of 12 to each we use up 36 out of the 41 available. The remaining 5 can be distributed as 0,1,4 or 0,2 and 3 giving possible values of Case 1: 12,13 and 16 or Case 2: 12,14 and 15.

67.In both cases, the cost of the pen is 12.

68.If the cost of the eraser is not divisible by 4, it means that Case 2 holds true. For this case, the cost of the pencil is 14.

69.Amrit would place eight oranges in the basket (as there are eight 1’s).

For the mangoes, he would place six mangoes (number of 2’s) and remove four mangoes (number of 4’s) from the basket. Thus, there would be 2 mangoes and 8 oranges in the basket.

A total of 8 – 2 = 6 extra oranges in the basket. Option (c) is correct.

70.Solve using trial and error – Option (c) fits the situation as if we start with 15 mangoes, the following structure would take place:

Start with 15 mangoes Æ First watchman takes 1/3rd + 1 more = 5 + 1 = 6 mangoes Æ 9 mangoes left.

Second watchman takes Æ 1/3rd + 1 more = 3 + 1 = 4 mangoes Æ 5 mangoes left.

Third watchman takes Æ 1/5th + 4 more = 1 + 4 = 5 mangoes Æ 0 mangoes left.

71.The last 3 digits of the number would determine the remainder when it is divided by 8. The number upto the 120th digit would be 1234567891011… 646. 646 divided by 8 gives us a remainder of 6.

72.There would be multiple ways of scoring 34.5 marks. Think about this as follows:

If he solves 80 and gets all 80 correct, he would end up scoring 80 marks.

With every question that would go wrong his score would fall down by: 1.5 marks (he would lose the 1 mark he is gaining and further attract a penalty of 0.5 marks).

Also, for every question he does not attempt his score would fall down by: 1.25 marks (he would lose the 1 mark he is gaining and further attract a penalty of 0.25 marks).

Thus, his score would drop @ 1.5 and @1.25 marks for every wrong and every unattempted question respectively.

Also, to get a total of 34.5 marks overall he has to lose 45.5 marks.

There are many possible combinations of non attempts and wrongs through which he can possibly lose 45.5 marks—for example:

17 wrongs (loses 25.5 marks) and 16 non-attempts (loses 20 marks)

12 wrongs (loses 18 marks) and 22 non attempts (loses 27.5 marks)

Hence, we cannot answer this question uniquely and the answer is Option (d).

73.Continuing the thought process for the previous question our thinking would go as follows:

10 questions unanswered Æ loses 12.5 marks

To lose another 33 marks he needs to get 22 incorrects.

Thus, the number of corrects would be 80 – 10 – 22= 48. Option (b) is correct.

74.3M + 4G + 5W = 750(i)

6M + 9G + 10W = 1580(ii)

Adding the two equations we get:

9M + 13G + 15W = 2330(iii)

Dividing this expression by 3 we get:

3M + 4.33G + 5W =776.666(iv)

(iv) - (i) Æ 0.33 G = 26.666 Æ G = 80

Now, if we look at the equation (i) and multiply it by 2, we get: 6M + 8G + 10W = 1500. If we subtract the cost of 4 guavas from this we would get:

6M + 4G + 10W = 1500 – 320 = 1180

Option (b) is correct.

75.If you try a value of M as 5, N would become ¼. It can be seen that 51/4 (which would be the value of MN) would be around 1.4 and hence, less than 2.

If you try for possible values of MN by increasing the value of M, you would get 61/5, 71/6, 81/7, 91/8 and so on. In each of these cases you can clearly see that the value of MN would always be getting consecutively smaller than the previous value.

If you tried to go for values of M such that they are lower than 5, you would get the following values for MN:

41/3, 31/2 and the last value would be 21/1 . In this case, we can clearly see that the value of the expression MN is increasing. However, it ends at the value of 2 (for 21/1) and hence that is the maximum value that MN can take. Option (a) is correct.

76.(7M + 9G + 19W) – (4M + 6G + 16W) = 120. Hence, 1M + 1G + 1W = 40

77.The cost of a mango cannot be uniquely determined here because we have only 2 equations between 3 variables, and there is no way to eliminate one variable.

78.Since the value of y varies between –8 to 2, it is evident that if we take a very small value for y, say 0.0000000000000000000000001, and we take normal integral values for x and z, the expression xz/y would become either positive or negative infinity (depending on how you manage the signs of the numbers x, y and z).

79.Ties < Trousers < Shirts. Since each of the three is minimum 11, the total would be a minimum of 33 (for all 3). The remaining 5 need to be distributed amongst ties, trousers and shirts so that they can maintain the inequality Ties < Trousers < Shirts

This can be achieved with 11 ties, and the remaining 27 pieces of clothing distributed between trousers and shirts such that the shirts are greater than the trousers.

This can be done in at least 2 ways: 12 trousers and 15 shirts; 13 trousers and 14 shirts.

If you try to go for 12 ties, the remaining 26 pieces of clothing need to be distributed amongst shirts and trousers such that the shirts are greater than the trousers and both are greater than 12.

With only 26 pieces of clothing to be distributed between shirts and trousers this is not possible. Hence, the number of ties has to be exactly 11. Option (a) is correct.

80.The number of shirts would be at least 14 as the two distributions possible are: 11, 12, 15 and 11, 13, 14. Option (b) is correct.

81.The LCM of 12, 15, 18 and 20 is 180. Thus, the least number would be 184.

Option (c) is correct.

82.2800 = 20 × 20 × 7. Thus, we need to multiply or divide with 7 in order to make it a perfect square.

83.The answer is given by = 32.

This can be experimentally verified as 302 = 900, 312 = 961 and 322 = 1024. Hence, 1024 is the required answer. Option (c) is correct.

84.First find the LCM of 6, 9, 15 and 18. Their LCM = 18 × 5 = 90.

The series of numbers which would leave a remainder of 4 when divided by 6, 9, 15 and 18 would be given by:

LCM + 4; 2 × LCM + 4; 3 × LCM + 4; 4 × LCM + 4; 5 × LCM + 4 and so on .

Thus, this series would be:

94, 184, 274, 364, 454….

The other constraint in the problem is to find a number which also has the property of being divisible by 7. Checking each of the numbers in the series above for their divisibility by 7, we see that 364 is the least value which is also divisible by 7. Option (c) is correct.

85.LCM of 2, 3, 4, 5 and 6 = 6 × 5 × 2 =60 (Refer to the shortcut process for LCM given in the chapter notes).

Thus, the series 61, 121, 181 etc would give us a remainder 1 when divided by 2, 3, 4, 5 and 6.

The least 3 digit number in this series is 121. Option (c) is correct.

86.70 = 2 × 5 × 7; 245 = 5 × 7 × 7.

HCF = 5 × 7 = 35. Option (a) is correct.

87.7056 is the closest perfect square below 7147. Hence, 7147 – 7056 = 91 is the required answer. Option (c) is correct.

88.If you check each of the options, you can clearly see that the number 3600 is divisible by each of these numbers. Option (b) is correct.

Alternately, you can also think about this question as:

The LCM of 6, 8 and 15 = 120. Thus, we need to look for a perfect square in the series of multiples of 120. 120, 240, 360, 480, 600, 720…., the first number which is a perfect square is: 3600.

89.30492 = 22 × 32 × 71 × 112.

For a number to be a perfect square each of the prime factors in the standard form of the number needs to be raised to an even power. Thus, we need to multiply or divide the number by 7 so that we either make it: 22 × 32 ×72 × 112 (if we multiply the number by 7) or

We make it: 22 × 32 × 112. (if we divide the number by 7).

Option (b) is correct.

90.88 × 113 = 9944 is the greatest 4 digit number exactly divisible by 88. Option (c) is correct.

91.3/4th of 116 = ¾ × 116 = 87

4/5th of 45 = 4/5 × 45 = 36.

Required difference = 51.

Option (d) is correct.

92.The correct arrangement would be 75 plants in a row and 75 rows since 5625 is the square of 75.

93.9EVEN POWER × 74n+1 Æ 1 × 7 = 7 as the units digit of the multiplication.

Option (a) is correct.

94.It can be seen that for 40 and 80 the number of factors are 8 and 10 respectively. Thus option (c) satisfies the condition.

95.In order to solve this question you need to realize that remainders of 1, 3, 4 and 5 in the case of 3, 5, 6 and 7 respectively, means remainders of –2 in each case. In order to find the number which leaves remainder –2 when divided by these numbers you need to first find the LCM of 3, 5, 6 and 7 and subtract 2 from them. Since the LCM is 210, the first such number which satisfies this condition is 208. However, the question has asked us to find the largest such number below 4000. So you need to look at multiples of the LCM and subtract 2. The required number is 3990 – 2 = 3988

96.The number would be given by the (LCM of 2, 3 and 4) +1 Æ which is 12 + 1 = 13. Option (b) is correct.

97.The number would be given by the 2 × (LCM of 2, 3 and 4) +1 Æ which is 24 + 1 = 25. Option (a) is correct.

98.In order to solve this you need to find the last 2 digit number in the series got by the logic:

(LCM of 2, 3, 4) + 1; 2 × (LCM of 2, 3, 4) + 1; 3 × (LCM of 2, 3, 4) + 1 …

i.e. you need to find the last 2 digit number in the series:

13, 25, 37, 49….

In order to do so, you can do one of the following:

(a)Complete the series by writing the next numbers as:

61, 73, 85, 97 to see that 97 is the required answer.

(b)Complete the series by adding a larger multiple of 12 so that you reach closer to 100 faster.

Thus, if you have seen 13, 25, 37,,,,, you can straightaway add any multiple of 12 to get a number close to 100 in the series in one jump.

Thus, if you were to add 12 × 4 = 48 to 37 you would reach a value of 85 (and because you have added a multiple of 12 to 37 you can be sure that 85 would also be on the same series.

Thus, the thinking in this case would go as follows:

13, 25, 37, …, 85, 97. Hence, the number is 97.

If you look at the two processes above- it would seem that there is not much difference between the two, but the real difference would be seen and felt if you would try to solve a question which might have asked you to find the last 3 digit number in the series. (as you would see in the next question). In such a case, getting to the number would be much faster if you use a multiple of 12 to jump ahead on the series rather than writing each number one by one.

(c)For the third way of solving this, you can see that all the numbers in the series:

13, 25, 37… are of the form 12n + 1. Thus, you are required to find a number which is of the form 12n + 1 and is just below 100.

For this purpose, you can try to first see what is the remainder when 100 is divided by 12.

Since the remainder is 4, you can realize that the number 100 is a number of the form 12n + 4.

Obviously then, if 100 is of the form 12n + 4, the largest 12n + 1 number just below 100 would occur at a value which would be 3 less than 100. (This occurs because the distance between 12n + 4 and 12n + 1 on the number line is 3.)

Thus the answer is 100 – 3 = 97.

Hence, Option (c) is correct.

99.In order to solve this you need to find the last 3 digit number in the series got by the logic:

(LCM of 2, 3, 4) +1; 2 × (LCM of 2, 3, 4) +1; 3 × (LCM of 2, 3, 4) + 1 …

i.e. you need to find the last 3 digit number in the series:

13, 25, 37, 49….

In order to do so, you can do one of the following:

(a)Try to complete the series by writing the next numbers as:

61, 73, 85, 97, 109… However, you can easily see that this process would be unnecessarily too long and hence infeasible to solve this question.

(b)Complete the series by adding a larger multiple of 12 so that you reach closer to 1000 faster.

This is what we were hinting at in the previous question. If we use a multiple of 12 to write a number which will come later in the series, then we can reach close to 1000 in a few steps. Some of the ways of doing this are shown below:

(i)13, 25, 37, …….. 997 (we add 12 × 80 = 960 to 37 to get to 37 + 960 = 997 which can be seen as the last 3 digit number as the next number would cross 1000).

(ii)13, 25, 37, …… (add 600)….637, …(add 120)… 757,,,,, (add 120),,,, 877, ….(add 120)….997. This is the required answer.

(iii)13, 25, 37, ……(add 120)….157, …(add 120)…277……(add 120)…..397….(add 120)……517…. (add 120)…637……(add 120)…757,,,,, (add 120),,,, 877, ….(add 120)….997. This is the required answer.

What you need to notice is that all the processes shown above are correct. So while one of them might be more efficient than the other, as far as you ensure that you add a number which is a multiple of 12(the common difference) you would always be correct.

(c)Of course you can also do this by using remainders. For this, you can see that all the numbers in the series:

13, 25, 37….are of the form 12n + 1. Thus, you are required to find a number which is of the form 12n + 1 and is just below 1000.

For this purpose, you can try to first see what is the remainder when 1000 is divided by 12.

Since the remainder is 4, you can realize that the number 1000 is a number of the form 12n + 4.

Obviously then, if 1000 is of the form 12n + 4, the largest 12n + 1 number just below 1000 would occur at a value which would be 3 less than 1000. (This occurs because the distance between 12n + 4 and 12n + 1 on the number line is 3.)

Thus the answer is 1000 – 3 = 997.

Hence, Option (c) is correct.

100.The logic of this question is that the frog can never reach point C if it makes an odd number of jumps. Since, the question has asked us to find out in how many ways can the frog reach point C in exactly 7 jumps, the answer would naturally be 0. Option (d) is correct.

101.They would ring together again after a time interval which would be the LCM of 5, 6 and 7. Since the LCM is 210, option (d) is the correct answer.

102.Since they would ring together every 210 seconds, their ringing together would happen at time intervals denoted by the following series- 210, 420, 630, 840, 1050, 1260, 1470, 1680, 1890, 2100, 2310, 2520, 2730, 2940, 3150, 3360, 3570 – a total of 17 times. This answer can also be calculated by taking the quotient of 3600/210 = 17. Option (a) is correct.

103.The maximum number of soldiers would be given by the HCF of 66, 110 and 242. The HCF of these numbers can be found to be 22 and hence, option (c) is correct.

104.The minimum number of rows would happen when the number of soldiers in each row is the maximum. Since, the HCF is 22 the number of soldiers in each row is 22. Then the total number of rows would be given by:

66/22 + 110/22 + 242/22 = 3 + 5 + 11 = 19 rows. Option (b) is correct.

105.The Number of bottle sizes possible would be given by the number of factors of the HCF of 170, 102 and 374. Since, the HCF of these numbers is 34, the bottle sizes that are possible would be the divisors of 34 which are 1 liter, 2 liters, 17 liters and 34 liters respectively. Thus, a total of 4 bottle sizes are possible. Option (c) is correct.

106.The size of the largest bottle that can be used is obviously 34 liters (HCF of 170, 102 and 374). Option (d) is correct.

107.The minimum number of bottles required would be: 170/34 + 102/34 + 374/34 = 5 + 3 + 11 = 19. Option (b) is correct.

108.The answer would be given by the quotients of 100/5 + 100/25 = 20 + 4 = 24. Option (c) is correct.

The logic of how to think about Questions 108 to 118 has been given in the theory in the chapter. Please have a relook at that in case you have doubts about any of the solutions till Question 118.

109.24 + 4 = 28. Option (d) is correct.

110.280 + 56 + 11 + 2 = 349. Option (c) is correct.

111.76 + 15 + 3 = 94. Option (c) is correct.

112.14 + 2 = 16. Option (c) is correct.

113.13 + 4 + 1 = 18. Option (a) is correct.

114.17 + 5 + 1 = 23. Option (c) is correct.

115.11 + 1 = 12. Option (a) is correct.

116.16 + 2 = 18. Option (d) is correct.

117.The number of 3’s in 122! = 40 +13 + 4 + 1 = 58. The number of 2’s in 122! = 61 + 30 + 15 + 7 + 3 + 1 = 117. The number of 22s is hence equal to the quotient of 117/2 = 58. We have to choose the lower one between 58 and 58. Since both are equal, 58 would be the correct answer. Hence, Option (d) is correct.

118.The power of 20 which would divide 155! would be given by the power of 5’s which would divide 155! since 20 = 22 × 5 and the number of 22s in any factorial would always be greater than the number of 5s in the factorial. 31 + 6 + 1 = 38. Option (b) is correct.

119.1024 = 210. Hence, x has to be a number with power of 2 greater than or equal to 5. Since, we are asked for the minimum value, it must be 5. Thus, option (b) is correct.

120.The two digit numbers that would leave a remainder of 3 when divided by 7 would be the numbers 10, 17, 24, 31, 38, 45, …94. The sum of these numbers would be given by the formula

(number of numbers × average of the numbers) = There are 13 numbers in the series and their average is 52. Thus, the required answer is 13 × 52 = 676. Option (c) is correct.

(Note the logic used here is that of sum of an Arithmetic Progression and is explained in details in the next chapter).

121.All numbers divisible by 27 would also be divisible by 3 and 9. Numbers divisible by 9 but not by 27 would be divisible by 3 and 9 only and need to be counted to give us our answer.

The numbers which satisfy the given condition are: 9, 18, 36, 45, 63, 72, 90, 99, 117, 126, 144, 153, 171, 180 and 198. There are 15 such numbers.

Alternately, you could also think of this as:

Between 1 to 200 there are 22 multiples of 9. But not all these 22 have to be counted as multiples of 27 need to be excluded from the count. There are 7 multiples of 27 between 1 and 200. Thus, the answer would be given by 22 – 7 =15. Option (b) is correct.

122.The required minimum happens when we use (–0.5) as the value of N. (–0.5)2 + (–0.5) = 0.25 – 0.5 = –0.25 is the least possible value for the sum of any number and it’s square. Option (b) is correct.

123.Each of the statements are false as we can have the sum of 2 prime numbers ending in 5, 0 and the sum can also be odd. Option (d) is correct.

124.This occurs for values such as: 103 – 013; 213 – 123; 324 – 234 etc where it can be seen that the value of X is 1 more than Y. The possible pairs of values for X and Y are: 1, 0; 2, 1; 3, 2…9, 8 – a total of nine pairs of values. Option (a) is correct.

125.The required sum would be given by the formula n(n + 1) for the first n even numbers. In this case it would be 50 × 51 = 2550. Option (d) is correct.

126.763/57 leaves a remainder of 22 when it is divided by 57. Thus, if we were to add 35 to this number the number we obtain would be completely divisible by 57. Option (a) is correct.

127.Since, 763/57 leaves a remainder of 22, we would need to subtract 22 from 763 in order to get a number divisible by 57. Option (b) is correct.

128.8441/57 leaves a remainder of 5. Thus, if we were to add 52 to this number the number we obtain would be completely divisible by 57. Option (d) is correct.

129.Since, 8441/57 leaves a remainder of 5. We would need to subtract 5 from 8441 in order to get a number divisible by 57. Option (c) is correct.

130.10000 divided by 79 leaves a remainder of 46. Hence, if we were to add 33 to 10000 we would get a number divisible by 79. The correct answer is 10033. Option (b) is correct.

131.100000 divided by 79 leaves a remainder of 65. Hence, if we were to subtract 65 from 100000 we would get a number divisible by 79. The correct answer is 99935. Option (b) is correct.

132.It can be seen that in the multiples of 12, the number closest to 773 is 768. Option (a) is correct.

133.Since 12 is a divisor of 84, the required remainder would be got by dividing 57 by 12. The required answer is 9. Option (c) is correct

134.Since 11 does not divide 84, there are many possible answers for this question and hence we cannot determine one unique value for the answer. Option (d) is thus correct.

135.The numbers that can do so are going to be factors of the difference between 511 and 667 i.e. 156. The factors of 156 are 1,2,3,4,6, 12,13, 26, 39, 52,78,156. There are 12 such numbers. Option (b) is correct.

136.The multiples of 13 between 200 and 400 would be represented by the series:

208, 221, 234, 247, 260, 273, 286, 299, 312, 325, 338, 351, 364, 377 and 390

There are a total of 15 numbers in the above series. Option (b) is correct.

Note: The above series is an Arithmetic Progression. The process of finding the number of terms in an Arithmetic Progression are defined in the chapter on Progressions.

137.8n/5 – 5n/8 = 39n/40 = 39. Solve for n to get the value of n = 40. Option (c) is correct.

138.x + y = 3(xy) Æ 2x = 4y. If we take y as 10, we would get the value of x as 20. Option (b) is correct.

139.411 + 412 + 413 + 414 + 415 = . 411 (1 + 41 + 42 + 43 + 44) = 411 × 341. The factors of 341 are:

1, 11, 31 and 341. Thus, we can see that the values in each of the three options would divide the expression. 411 + 412 + 413 + 414 + 415 . Thus, option (d) is correct.

140.Since the numbers have their HCF as 16,both the numbers have to be multiples of 16 (i.e. 24).

7168 = 210 × 71

In order to visualise the required possible pairs of numbers we need to look at the prime factors of 7168 in the following fashion:

7168 = 210 × 71 = (24 × 24) × 22 × 71 = (16 × 16) × 2 × 2 × 7 It is then a matter of distributing the 2 extra twos and the 1 extra seven in 22 × 71 between the two numbers given by 16 and 16 inside the bracket. The possible pairs are:

32 × 224; 64 × 112; 16 × 448. Thus there are 3 distinct pairs of numbers which are multiples of 16 and whose product is 7168. However, out of these the pair 32 × 224 has it’s HCF as 32 and hence does not satisfy the given conditions. Thus there are two pairs of numbers that would satisfy the condition that their HCF is 16 and their product is 7168. Option (a) is correct.

Level of Difficulty (II)

1.If a and b are two numbers, then their Arithmetic mean is given by (a + b)/2 while their geometric mean is given by (ab)0.5. Using the options to meet the conditions we can see that for the numbers in the first option (6 and 54) the AM being 30, is 24 less than the larger number while the GM being 18, is 12 more than the smaller number. Option (a) is correct.

2.Use the principal of counting given in the theory of the chapter. Start with 101 numbers (i.e. all numbers between 200 and 300 both included) and subtract the number of numbers which are divisible by 2 (viz. [(300 – 200)/2] + 1 = 51 numbers), the number of numbers which are divisible by 3 but not by 2 (Note: This would be given by the number of terms in the series 201, 207, … 297. This series has 17 terms) and the number of numbers which are divisible by 5 but not by 2 and 3. (The numbers are 205, 215, 235, 245, 265, 275, 295. A total of 7 numbers)

Thus, the required answer is given by 101 – 51 – 17 – 7 = 26. Option (b) is correct.

3.Since 15n3, 6n2 and 5n would all be divisible by n, the condition for the expression to not be divisible by n would be if x is not divisible by n. Option (c) is correct.

4.It can be seen that the first expression is larger than the second one. Hence, the required answer would be given by the (units digit of the first expression – units digit of the second expression) = 6 – 0 = 6. Option (b) is correct.

5.Suppose you were to solve the same question for 103 – 7 and 102 + x.

103 – 7 = 993 and 102 + x = 100 + x.

Difference = 993 – x

For 104 – 7 and 103 + x

The difference would be 9993 – (1000 + x)

= (8993 – x)

For 105 – 7 and 103 + x

Difference: 99993 – (10000 + x) = 89993 – x

You should realize that the difference for the given question would be 8999 . . . . . 9 3 – x. For this difference to be divisible by 3, x must be 2 (since that is the only option which will give you a sum of digits divisible by 3.)

6.The value of x should be such that the left hand side after completely removing the square root signs should be an integer. For this to happen, first of all the square root of 3x should be an integer. Only 3 and 12 from the options satisfy this requirement. If we try to put x as 12, we get the square root of 3x as 6. Then the next point at which we need to remove the square root sign would be 12+2(6) = 24 whose square root would be an irrational number. This leaves us with only 1 possible value (x = 3). Checking for this value of x we can see that the expression is satisfied as LHS = RHS.

7.If the number is n, we will get that 22n + n = 23n is half the square of the number n. Thus, we have

n2 = 46 n Æ n = 46

8.1255/311 = 344.455 Æ 4 as units place.

Similarly, 848/1618= 272 Æ 6 as the units place.

Hence, 0 is the answer.

9.1 + 2 + 22 + . . . + 231 = 232 – 1

Hence, the average will be: = 227 – 1/25 which lies between 226 and 227.

Hence the answer will be (c).

10.The denominator 99 has the property that the decimals it gives rise to are of the form 0.xyxyxy. This question is based on this property of 99. Option c is correct.

11.The value of b has to be 2 since, r = 2y. Hence, option d is the only choice.

12.For [x]3 + {x}2 to give – 7.91,

[x]3 should give – 8 (hence, [x] should be –2]

Further, {x}2 should be + 0.09.

Both these conditions are satisfied by –1.7.

Hence option (d) is correct.

13.165 + 215 = 220+ 215 = 215 (25 + 1) Æ Hence, is divisible by 33.

14.The interpretation of the situation AB + XY = 1XP is that the tens digit in XY is repeated in the value of the solution (i.e. 1XP). Thus for instance if X was 2, it would mean we are adding a 2 digit number AB to a number in the 20’s to get a number in the 120’s. This can only happen if AB is in the 90’s which means that A is 9.

15.|x – 3| + 2 |x + 1| = 4 can happen under three broad conditions.

(a)When 2|x + 1| = 0, then |x – 3| should be equal to 4.

Putting x = – 1, both these conditions are satisfied.

(b)When 2|x + 1| = 2, x should be 0, then |x – 3| should also be 2. This does not happen.

(c)When 2 |x + 1| = 4, x should be + 1 or – 3, in either case |x – 3| which should be zero does not give the desired value.

16.At a value of x = 0 we can see that the expression x2 + |x – 1| = 1 Æ 0 + 1 = 1. Hence, x = 0 satisfies the given expression. Also at x = 1, we get 1 + 0 = 1. Option (d) is correct.

17.4n+1 represents an odd power of 4 (and hence would end in 4). Similarly, 42n represents an even power of 4 (and hence would end in 6). Thus, the least number ‘x’ that would make both 4n+1 + x and 42nx divisible by 5 would be for x = 1.

18.Check for each value of the options to see that the expression does not become divisible by 9 for any of the initial options. Thus, there is no value that satisfies the divisibility by 9 case.

19.The expression would have solutions based on a structure of:

4 + 0; 3 + 1; 2 + 2; 1 + 3 or 0 + 4.

There will be 2*1 = 2 solutions for 4 + 0 as in this case x can take the values of 8 and 0, while y can take a value of 4;

Similarly there would be 2*2 = 4 solutions for 3 + 1 as in this case x can take the values of 7 or 1, while y can take a value of 5 or 3;

Thus, the total number of solutions can be visualised as:

2 (for 4 + 0) + 4 (for 3 + 1) + 4 (for 2 + 2) + 4 (for 1 + 3) + 2 (for (0 + 2) = 16 solutions for the set (x, y) where both x and y are integers.

20.The numerator of 332/50 would be a number that would end in 1. Consequently, the decimal of the form .bx would always give us a value of x as 2.

21.If we assume the numbers as 16 and 4 based on 4:1 (in option a), the AM would be 10 and the GM = 8 a difference of 20% as stipulated in the question. Option (a) is correct.

22.990 = 11 × 32 × 2 × 5. The highest power of 990 which would divide 1090! would be the power of 11 available in 990. This is given by [1090/11] + [1090/121] = 99 + 9 = 108

23.For finding the highest power of 6 that divides 146!, we need to get the number of 3’s that would divide 146!. The same can be got by: [146/3] + [48/3] + [16/3] + [5/3] = 70.

24.There would be two fives and more than two twos in the prime factors of the numbers in the multiplication. Thus, we would get a total of 2 zeroes.

25.Both 333555 and 555333 are divisible by 3,37 and 111. Further, the sum of the two would be an even number and hence divisible by 2. Thus, all the four options divide the given number.

26.Both the values of options a and b satisfy the given expression. As for 5.16, the value of [x]2 = 25 and the value of {x} = 0.16. Thus, [x]2 + {x}1 = 25.16

Similarly for a value of x = –4.84, the value of [x] = –5 and hence [x]2 = 25 and the value of {x} = 0.16. Thus, [x]2 + {x}1 = 25.16

27.The given conditions can be seen to be true for the number 49. Option (c) is correct.

28.Solve this question through options. Also realize that a × b = a + b only occurs for the situation 2 × 2 = 2 + 2. Hence, clearly the answer has to be none of these.

29.863 satisfies each of the conditions and can be spotted through checking of the options.

30.The number of zeroes would be given by counting the number of 5’s. The relevant numbers for counting the number of 5’s in the product would be given by:

55; 1010, 1515, 2020, 2525…and so on till 100100

The number of 5’s in these values would be given by:

(5 + 10 + 15 + 20 + 50 + 30 + 35 + 40 + 45 + 100 + 55 + 60 + 65 + 70 + 150 + 80 + 85 + 90 + 95 + 200)

This can also be written as:

(5 + 10 + 15 + 20 + 25 + 30 + 35 + 40 + 45 + 50 + 55 + 60 + 65 + 70 + 75 + 80 + 85 + 90 + 95 + 100) + (25 + 50 + 75 + 100)

= 1050 + 250 = 1300

31.Option (a) is correct as the LCM of 5 and 105 is 105 and their HCF is 5. Also for the pair of values, 15 and 35 the HCF is 5 and the LCM is 105.

32.Solve using options. Using option b = 3/5 and performing the given operation we get:

2/3 – 3/5 = (10 – 9)/15 = 1/15. Option (b) is hence correct.

33.Both the conditions are satisfied for option (a) = 72 as the number 72 exceeds the sum of squares of the digits by 19 and also 72 exceeds the doubled product of it’s digits by 44.

34.Solve by checking the given options. 31 and 13 are possible values of the number as defined by the problem.

35.The given conditions are satisfied for the number 24.

36.The number of 2’s in the given expression is lower than the number of 5’s. The number of 2’s in the product is 9 and hence that is the number of zeroes.

37.45 = 32 × 5. Hence, we need to count the number of 32’s and 5’s that can be made out of 123!.

Number of 3’s = 41 + 13 + 4 + 1 = 59 Æ Number of 32’s = 29

Number of 5’s = 24 + 4 = 28.

The required answer is the lower of the two (viz. 28 and 29). Hence, option (a) 28 is correct.

38.The first sentence means that the numbers are in an arithmetic progression. From the statements and a little bit of visualization, you can see that 8.5, 10 and 11.5 can be the three values we are looking for – and hence the middle value is 10.

39.990 = 11 × 32 × 5 × 2. For n! to be divisible by 990, the value of n! should have an 11 in it. Since, 11 itself is a prime number, hence the value of n should be at least 11.

40.For the expression to hold true, x and y should both be positive.

41.Since, we are not given options here we should go ahead by looking within the factors of 144 (especially the two digit ones)

The relevant factors are 72,48, 36, 24, 18 and 12. Thinking this way creates an option for you where there is none available and from this list of numbers you can easily identify 24 as the required answer.

42–46.Write simple equations for each of the questions and solve.

47.Since the sum of squares of the digits of the two digit number is 10, the only possibility of the numbers are 31 and 13.

48.If the number is ‘ab’ we have the following equations:

(10a + b) = 4 (a + b) + 3 Æ 6a – 3b = 3

(10a + b) = 3 (a × b) + 5.

Obviously we would need to solve these two equations in order to get the values of the digits a and b respectively. However, it might not be a very prudent decision to try to follow this process- as it might turn out to be too cumbersome.

A better approach to think here is:

From the first statement we know that the number is of the form 4n + 3. Thus, the number has to be a term in the series 11, 15, 19, 23, 27…

Also from the second statement we know that the number must be a 3n + 5 number.

Thus, the numbers could be 11, 14, 17, 20, 23….

Common terms of the above two series would be probable values of the number.

It can be seen that the common terms in the two series are: 11, 23, 35, 47, 59, 71, 83 and 95. One of these numbers has to be the number we are looking for.

If we try these values one by one, we can easily seen that the value of the two digit number should be 23 since Æ 23/ (2 + 3) Æ Quotient as 4 and remainder = 3.

Similarly, if we look at the other condition given in the problem we would get the following-

23/6 Æ quotient as 3 and remainder = 5.

Thus, the value of the missing number would be 23.

49.We can see from the description that the number (say X) must be such that X + 100 and X + 169 both must be perfect squares. Thus we are looking for two perfect squares which are 69 apart from each other. This would happen for 342 and 352 since their difference would be (35 – 34)(35 + 34) = 69.

50.Since their least common multiple is 102, we need to look for two factors of 102 such that they add up to 85. 51 and 34 can be easily spotted as the numbers.

51.If one number is x, the other should be 6x or 12x or 18x or 24x and so on. Also, their sum should be either 504 or 1008 or 1512. (Note: the next multiple of 504 = 2016 cannot be the sum of two three digit numbers.

52.Obviously 46 and 64 are the possible numbers.

53.The key here is to look for numbers which are more than three times but less than four times the product of their digits. Also, the product of the digits should be greater than 9 so as to leave a remainder of 9 when the number is divided by the product of it’s digits.

In the 10s, 20s and 30s there is no number which gives a quotient of 3 when it is divided by the product of it’s digits. In the 40s, 43 is the only number which has a quotient of 3 when divided by 12 (product of it’s digits). But 43/12 does not give us a remainder of 9 as required.

In the 50s the number 53 divided by 15 leaves a remainder of 8, while in the 60s, 63 divided by 18 gives us a remainder of 9 as required.

54.The first thing to use while solving this question is to look at the information that the sum of squares of the three digits is 109. A little bit of trial and error shows us that this can only occur if the digits are 8, 6 and 3. Using the other information we get that the number must be 863 since, 863 – 495 = 368.

55.It is obvious that the only condition where the cubes of 3 numbers add up to 243 is when we add the cubes of 3 and 6. Hence, the numbers possible are 36 and 63.

56.There would definitely be two numbers and in case we take the first number as 7n – 1, there would be three numbers – (as can be seen when we take the first number as 27 and the other number is 43).

57.Between 1114, 110 × 109 × 108 × 107, 109 × 110 × 112 × 113.

It can be easily seen that

111 × 111 × 111 × 111 > 110 × 109 × 108 × 107

also 109 × 110 × 112 ×113 > 109 × 110 × 108 × 107

Further the product 111 × 111 × 111 × 111 > 109 × 110 × 112 × 113 ( since, the sum of the parts of the product are equal on the LHS and the RHS and the numbers on the LHS are closer to each other than the numbers on the RHS.

58.Both x and y should be highest for xy to be maximum. Similarly x should be minimum and y should be maximum for x/y to be minimum.

59.200300 = (2006)50

300200 = (3004)50

400150 = (4003)50

Hence 200300 is greater.

61.The sum of squares of the first n natural numbers is given by n(n + 1)(2n + 1)/6.

For this number to be divisible by 4, the product of n(n + 1)(2n + 1) should be a multiple of 8. Out of n, (n + 1) and (2n + 1) only one of n or (n + 1) can be even and (2n + 1) would always be odd.

Thus, either n or (n + 1) should be a multiple of 8.

This happens if we use n = 7, 8, 15, 16, 23, 24, 31, 32, 39, 40, 47, 48. Hence, 12 such numbers.

62.In the 20s the numbers are: 23 to 29

In the 30s the numbers are: 32 to 39

Subsequently the numbers are 42 to 49, 52 to 59, 62 to 69, 72 to 79, 82 to 89 and 92 to 99.

A total of 63 numbers.

63.You need to solve this question using trial and error.

For 32 (option 1):

32 = 25. Hence 6 factors. On increasing by 50%, 48 = 24 × 31 has 10 factors. Thus the number of factors is increasing when the number is increased by 50% which is not what the question is defining for the number. Hence, 32 is not the correct answer.

Checking for option (b) 84.

84 = 22 × 31 × 71 Æ (2 + 1) (1 + 1) (1 + 1) = 12 factors

On increasing by 50% Æ 126 = 21 × 32 × 71 Æ (1 + 1) (2 + 1) (1 + 1) = 12 factors. (no change in number of factors).

Second Condition: When the value of the number is reduced by 75% Æ 84 would become 21 (31 × 71) and the number of factors would be 2 × 2 = 4 – a reduction of 66.66% in the number of factors.

64.There will be 9 single digit numbers using 9 digits, 90 two digit numbers using 180 digits, 900 three digit numbers using 2700 digits. Thus, when the number 999 would be written, a total of 2889 digits would have been used up. Thus, we would need to look for the 25494th digit when we write all 4 digit numbers. Since, 25494/4 = 6373.5 we can conclude that the first 6373 four digit numbers would be used up for writing the first 25492 digits. The second digit of the 6374th four digit number would be the required answer. Since, the 6374th four digit number is 7373, the required digit is 3.

65.In order to solve this question, think of the numbers grouped in groups of 9 as:

{1, 2, 3, 4, 5, 6, 7, 8, 9} {10, 11, 12…..18} and so on till {2989, 2990…2997} – A total of 333 complete sets. From each set we can take 4 numbers giving us a total of 333 × 4 = 1332 numbers.

Apart from this, we can also take exactly 1 multiple of 9 (any one) and also the last 3 numbers viz 2998, 2999 and 3000. Thus, there would be a total of 1332 + 4 = 1336 numbers.

66.It can be seen that for only 2 numbers (1 and ½) the consolidated number would be 1 + ½ + ½ = 2

For 3 numbers, (1, ½ , 1/3) the number would be 3. Thus, for the given series the consolidated number would be 1972.

67.The value of K would be 199 and hence, the required difference is 9 – 1 = 8

68.9 – 9 = 0 would be the difference between the units and the tens digits.

69.The highest ratio would be a ratio of 100 in the numbers, 100, 200, 300, 400, 500, 600, 700, 800 and 900. Thus a total of 9 numbers.

70.Basically every odd triangular number would have this property, that it is the difference of squares of two consecutive natural numbers. Thus, we need to find the number of triangular numbers that are odd.

3, 15, 21, 45, 55, 91, 105, 153, 171, 231, 253, 325, 351, 435, 465, 561, 595, 703, 741, 861, 903 – A total of 21 numbers.

71.The coefficients would be 44C0, 44C1, 44C2 and so on till 44C44. The sum of these coefficients would be 244 (since the value of nC0 + nC1 + nC2 +…nCn = 2n)

72.The remainder of each power of 9 when divided by 6 would be 3. Thus, for (2n+1) powers of 9, there would be an odd number of 3’s. Hence, the remainder would be 3.

73.The remainder when a number is divided by 16 is given by the remainder of the last 4 digits divided by 16 (because 10000 is a multiple of 16. This principle is very similar in logic to why we look at last 2 digits for divisibility by 4 and the last 3 digits for divisibility by 8). Thus, the required answer would be the remainder of 4950/16 which is 6.

74.58! – 38! = 38! (58 × 57 × 56 × 55 × …. × 39 – 1) Æ 38! (3n – 1) since the expression inside the bracket would be a 3n – 1 kind of number. Thus, the number of 3’s would depend only on the number of 3’s in 38! Æ 12 + 4 + 1 = 17.

75.The given expression can be seen as (22334ODD POWER)/5, since the sum of 12 + 22 + 32 + 42 + … + 662 can be seen to be an odd number. The remainder would always be 4 in such a case.

76.1233 × 3423 × 270 = 2159 × 333 × 1723. The number of factors would be 160 × 34 × 24 = 130560. Thus, option (d) is correct.

77.Option (a) is correct.

78.1152 = 2 × 2 × 2 × 2 × 2 × 2 × 2 × 3 × 3 = 27 × 32. Essentially every number starting from 4!3 would be divisible perfectly by 1152 since each number after that would have at least 7 twos and 2 threes.

Thus, the required remainder is got by the first three terms:

(1 + 8 + 216)/1152 = 225/1152 gives us 225 as the required remainder.

79.We can take only perfect squares of odd numbers and one perfect square of an even number. Thus, for instance we can take numbers like 1, 4, 9, 25, 49, 121, 169, 289, 361, 529, 841 and 961. A total of 12 such numbers can be taken.

80.(101 × 102 × 103 × 197 × 198 × 199)/100 Æ [1 × 2 × 3 × (–3) × (–2) × (–1)]/100 Æ –36 as remainder Æ remainder is 64.

81.[65 × 29 × 37 × 63 × 71 × 87]/100 Æ [–35 × 29 × 37 × –37 × –29 × –13]/100 Æ [35 × 29 × 37 × 37 × 29 × 13]/100 = [1015 × 1369 × 377]/100 Æ 15 × 69 × 77/100 Æ remainder as 95.

82.[65 × 29 × 37 × 63 × 71 × 87 × 85]/100 Æ [–35 × 29 × 37 × –37 × –29 × –13 × –15]/100 Æ [35 × 29 × 37 × 37 × 29 × 13 × –15]/100 = [1015 × 1369 × 377 × –15]/100 Æ [15 × 69 × 77 × – 15]/100 Æ remainder as 75.

83.[65 × 29 × 37 × 63 × 71 × 87 × 62]/100 Æ [–35 × 29 × 37 × –37 × –29 × –13 × 62]/100 Æ [35 × 29 × 37 × 37 × 29 × 13 × 62]/100 = [1015 × 1369 × 377 × 62]/100 Æ [15 × 69 × 77 × 62]/100 = [1035 × 4774]/100 Æ 35 ×74/100 Æ remainder as 90.

84.[75 × 35 × 47 × 63 × 71 × 87 × 82]/100 = [3 × 35 × 47 × 63 × 71 × 87 × 41]/2 Æ remainder = 1.

Hence, required remainder = 1 x 50 = 50.

85.For this question we need to find the remainder of:

(201 × 202 × 203 × 204 × 246 × 247 × 248 × 249) × (201 × 202 × 203 × 204 × 246 × 247 × 248 × 249) divided by 100.

(201 × 202 × 203 × 204 × 246 × 247 × 248 × 249) × (201 × 202 × 203 × 204 × 246 × 247 × 248 × 249)/100 = (201 × 101 × 203 × 102 × 246 × 247 × 248 × 249) × (201 × 202 × 203 × 204 × 246 × 247 × 248 × 249)/25

Æ (1 × 1 × 3 × 2 × –4 × –3 × –2 × –1) × (1 × 2 × 3 × 4 × –4 × –3 × –2 × –1)/25 = 144 × 576/25 Æ (19 × 1)/25 = remainder 19.

19 × 4 = 76 is the actual remainder (since we divided by 4 during the process of finding the remainder.

86.74/2400 gives us a remainder of 1. Thus, the remainder of 799/2400 would depend on the remainder of 73/2400 Æ remainder = 343.

87.The numerator can be written as (1729)752/1728 Æ remainder as 1.

88.Bikas’s movement in terms of the number of coins would be:

B Æ 3B (when Arun triples everyone’s coins) Æ B.

Think of this as: When Bikas triples everyone’s coins, and is left with 20 it means that the other 3 have 60 coins after their coins are tripled. This means that before the tripling by Bikas, the other three must have had 20 coins—meaning Bikas must have had 60 coins.

But 60 = 3B Æ B = 20.

89.For 83p796161q to be a multiple of 11 (here X is 11) we should have the difference between the sum of odd placed digits and even placed digits should be 0 or a multiple of 11.

(8 + p + 9 + 1 + 1) – (3 + 7 + 6 + 6 + q) = (19 + p) – (22 + q). For this difference to be 0, p should be 3 more than q which cannot occur since 0 < p < q.

The only way in which (19 + p) – (22 + q) can be a multiple of 11 is if we target a value of –11 for the expression. One such possibility is if we take p as 1 and q as 9.

The number would be 8317961619. On successive division by (p + q) =10 and 1 the sum of remainders would be 9.

90.n(n + 1)/2 should be a perfect square. The first value of n when this occurs would be for n = 8. Thus, on the 8th of March the required condition would come true.

91.We have to find the unit’s digit of 253 Æ 24n+1 Æ 2 as the units digit.

92.[7! (14 + 14 × 13 × 12 × 11 × 10 × 9 × 8)]/[7! (16 – 3)] = [(14 + 14 × 13 × 12 × 11 × 10 × 9 × 8)]/[(13)] Æ remainder 1.

Hence, the original remainder must be 7! (because for the sake of simplification of the numbers in the question we have cut the 7! From the numerator and the denominator in the first step.

93.x = 6 and y = 3 is one pair of values where the given condition is met.

After that you should be able to spot that if you were to increase x by 7, y would also increase by 4. The number of such pairs would depend on how many terms are there in the series

–498, –491, ….. –1, 6, 13, 20, … 489, 496. The series has 994/7 + 1 = 143 terms and hence there would be 143 pairs of values for (x, y) which would satisfy the equation.

94.All three conditions can be seen to be true.

95.Product of factorials < Sum of factorials would occur for any number that has either 0 or 1 in it.

The required numbers uptil and including 50 are: 10 to 19, 20, 21, 30, 31, 40, 41, 50. Besides for the number 22, the product of factorials of the digits would be equal to the sum of factorials of the digits. Thus a total of 18 numbers.

96.The maximum marks he can score is: 100 (if he gets all correct).

The minimum marks he can score would be given by: 10 × (–0.1) + 20 × (–0.2) + 70 × (–0.5) = – 40.

The difference between the two values would be 100 – (–40) = 140 marks.

Logic for Questions 97 to 99:

If a student solved 200 questions and got everything correct he would score a total of 620 marks. By getting a LOD 1 question wrong he would lose 4+2=6 marks, while by not solving an LOD 1 question he would lose 4 marks.

Similarly for LOD 2 questions, Loss of marks = 4.5 (for wrong answers) and loss of marks= 3 (for not solved)

Similarly for LOD 3 questions, Loss of marks = 3 (for wrong answers) and loss of marks = 2 (for not solved)

Since, he has got 120 marks from 100 questions solved he has to lose 500 marks (from the maximum possible total of 620) by combining to lose marks through 100 questions not solved and some questions wrong.

97.It can be seen through a little bit of trial and error with the options, that if he got 44 questions of LOD 1 correct and 56 questions of LOD 3 wrong he would end up scoring 44 × 4 – 56 × 1 = 176 – 56 = 120. In such a case he would have got the maximum possible incorrects with the given score.

98.32 × 4 + 1 × 3 – 1 × 1 = 130 (in this case he has solved 32 corrects from LOD 1, 1 correct from LOD 2 and 1 incorrect from LOD 3). Thus, a total of 34 attempts.

99.In the above case he gets 1 question incorrect. However, he can also get 130 marks by 30 × 4 + 2 × 3 + 2 × 2 where he gets 30 LOD 1 questions correct, and 2 questions correct each from LOD 2 and LOD 3).

The least number of incorrects would be 0.

100.The least number would be (LCM of 10, 9, 8, 7, 6 and 5 – 1) = 2519. The second least number = 2520 × 2 –1 = 5039.

Hints

Level of Difficulty (III)

1–5.Solve through options.

6.Use AP with first term 104 and last term 999 and common difference 5.

7.Find the first 2 digit number which gives a remainder of 3 when divided by 7 and then find the largest such number (10 and 94 respectively). Use Arithmetic Progression formulae to add the numbers.

8.Use AP with first term 105 and last term 995 and common difference is 10.

10.The cubes of the numbers are x – 3, x + 2 and x + 3. Use options and you will see that (a) is the answer.

11.(x2 y2) = 45. i.e. (xy)(x + y) = 45. The factors of 45 possible are, 15, 3; 9, 5 and 45, 1.

Hence, the numbers are 9 and 6, or 7 and 2 or 23 and 22.

12–18.Use options to check the given conditions.

19.The answer will be 50 since, 125*122 will give 50 as the last two digits.

20.The remainder theorem is to be used.

43101 + 23101 = (43 + 23) (. . . . .)

Hence, when divided by 66 the remainder will be zero.

21.The unit’s digit will be 1 × 5 = 5 (no carry over.) The tens digit will be ( 4*1 + 5*2) = 4 (carry over 1). The hundreds digit will be (3*1 + 4*2 + 5*1) = 6 + 1 (carried over) = 7. Hence, answer is 745.

22–25.Use options to solve.

26.Use the rule of indices and remainder theorem.

27.Options are not provided as it is an LOD 3 question. If they were there you should have used options.

28.Use trial and error

29.Use options.

30.Use remainder theorem and look at patterns by applying the rules of indices.

We get the value as:

Æ Looking at the pattern we will get 4 as the final remainder.

31.Use the remainder theorem and get the remainder as: 1 × 2 × 4 × 4 × 4/7 = 128/7 Æ 2 is the remainder. 32.

32.2100/3 = (24)25/3 Æ 1.

33.Use the remainder theorem and try finding the patterns.

34.Find the last digit of the number got by adding 12 + 22 + …92 (you will get 5 here). Then multiply by 10 to get zero as the answer.

35.(2100 – 1) and (2120 – 1) will yield the GCD as 220 – 1. (This has been explained in the theory of GCDs).

36.The GCDs of 100 ones and 60 ones will be twenty ones because 20 is the GCD of sixty and Hundred.

39.

But 43/7 gives us a remainder of 1.

Hence we need to convert 43232 into 43n + x (Think why!!)

Here again, we will be more interested in finding the value of x rather than n, since the remainder only depends on the value of x.

[Concept Note: When we start to write as the remainder.

43232 in the form 43 × 43 × 43 …… n times × 4x we are not bothered about how many times we can write 43 since it will continuously give us 1 every time as the remainder.